You are on page 1of 297

For more high yield stuff for the board exams please check our facebook page;

https://www.facebook.com/USMLEgoodies/
And for watching a wide variety of medical lecture videos please check our youtube channel;
https://www.youtube.com/channel/UCB18YOYrrzR6asXcXNGtKPQ/videos?view_as=public
Thanks

BOOKCHAPTER

Frontmatter
AmberS.TullyMDandJamesS.StuddifordMD,FACP
USMLEImagesfortheBoards,iii

USMLEImagesfortheBoards:AComprehensiveImageBasedReview

USMLEImagesfortheBoards
AComprehensiveImageBasedReview
AmberS.Tully,MD,AssociateStaff,DepartmentofFamilyMedicine,ClevelandClinic,
Cleveland,Ohio
JamesS.Studdiford,MD,FACP,AssociateProfessor,DepartmentofFamilyandCommunity
Medicine,JeffersonMedicalCollege,ThomasJeffersonUniversity,Philadelphia,Pennsylvania

Copyright2015Elsevier,Inc.Allrightsreserved.

BOOKCHAPTER

Copyright
AmberS.TullyMDandJamesS.StuddifordMD,FACP
USMLEImagesfortheBoards,iviv

1600JohnF.KennedyBlvd.
Ste1800
Philadelphia,PA191032899
USMLEIMAGESFORTHEBOARDS:ACOMPREHENSIVEIMAGEBASEDREVIEW
ISBN:9781455709038
Copyright2013bySaunders,animprintofElsevierInc.
Nopartofthispublicationmaybereproducedortransmittedinanyformorbyanymeans,electronic
ormechanical,includingphotocopying,recording,oranyinformationstorageandretrievalsystem,
withoutpermissioninwritingfromthepublisher.Detailsonhowtoseekpermission,further
informationaboutthePublisher'spermissionspoliciesandourarrangementswithorganizations
suchastheCopyrightClearanceCenterandtheCopyrightLicensingAgency,canbefoundatour
website:www.elsevier.com/permissions(http://www.elsevier.com/permissions).
Thisbookandtheindividualcontributionscontainedinitareprotectedundercopyrightbythe
Publisher(otherthanasmaybenotedherein).
Knowledgeandbestpracticeinthisfieldareconstantlychanging.Asnewresearchand
experiencebroadenourunderstanding,changesinresearchmethods,professionalpractices,or
medicaltreatmentmaybecomenecessary.
Practitionersandresearchersmustalwaysrelyontheirownexperienceandknowledgein
evaluatingandusinganyinformation,methods,compounds,orexperimentsdescribedherein.
Inusingsuchinformationormethodstheyshouldbemindfuloftheirownsafetyandthesafety
ofothers,includingpartiesforwhomtheyhaveaprofessionalresponsibility.

Withrespecttoanydrugorpharmaceuticalproductsidentified,readersareadvisedtocheck
themostcurrentinformationprovided(i)onproceduresfeaturedor(ii)bythemanufacturerof
eachproducttobeadministered,toverifytherecommendeddoseorformula,themethodand
durationofadministration,andcontraindications.Itistheresponsibilityofpractitioners,
relyingontheirownexperienceandknowledgeoftheirpatients,tomakediagnoses,to
determinedosagesandthebesttreatmentforeachindividualpatient,andtotakeall
appropriatesafetyprecautions.
Tothefullestextentofthelaw,neitherthePublishernortheauthors,contributors,oreditors,
assumeanyliabilityforanyinjuryand/ordamagetopersonsorpropertyasamatterof
productsliability,negligenceorotherwise,orfromanyuseoroperationofanymethods,
products,instructions,orideascontainedinthematerialherein.
Notices
InternationalStandardBookNumber
9781455709038
LibraryofCongressCataloginginPublicationData
Tully,Amber.
USMLEimagesfortheboards:animagebasedreview/AmberTully,JamesS.Studdiford.
p.cm.
Includesbibliographicalreferencesandindex.
ISBN9781455709038(pbk.:alk.paper)
I.Studdiford,JamesS.II.Title.
[DNLM:1.ClinicalMedicineExaminationQuestions.WB18.2]
LCclassificationnotassigned
616.0076dc23
2012001897
ContentStrategyDirector:MadeleneHyde
SeniorContentStrategist:JamesMerritt
SeniorContentCoordinator:MargaretNelson

PublishingServicesManager:PatriciaTannian/HemamaliniRajendrababu
SeniorProjectManager:SarahWunderly
ProjectManager:PrathibhaMehta
SeniorBookDesigner:LouisForgione
PrintedinChina
Lastdigitistheprintnumber:987654321

Copyright2015Elsevier,Inc.Allrightsreserved.

BOOKCHAPTER

Dedication
AmberS.TullyMDandJamesS.StuddifordMD,FACP
USMLEImagesfortheBoards,vv

Theboardsbookisdedicatedtoourstudentsandresidents,past,present,andfuture,whoserve
asourgreatestteachers.

Copyright2015Elsevier,Inc.Allrightsreserved.

BOOKCHAPTER

AbouttheAuthors
AmberS.TullyMDandJamesS.StuddifordMD,FACP
USMLEImagesfortheBoards,viivii

AmberStonehouseTully,MD,isAssociateStaffintheDepartmentofFamilyMedicineatthe
ClevelandClinic.Shehasaspecialinterestindermatology,plasticsurgery,physicaldiagnosis,and
clinicalimages.Whileamedicalstudentshecodevelopedamedicalimagingdatabase,Jefferson
ClinicalImages(JCI),whichcurrentlyholdsover1200images,boardsreviewquestions,clinical
cases,andlectures.Dr.Tullyenjoysmedicalwritingandhaspublishedseveralpapersinpeer
reviewedjournals.SheandDr.StuddifordcreatedTopDoc,amedicalquiziPhoneappreleasedin
2010.Dr.Tullyhasreceivedseveralleadershipawards,includingtheFamilyMedicineEducation
ConsortiumEmergingLeaderAward,AAFP/BristolMyersSquibbAwardforExcellenceinGraduate
MedicalEducation,theWilliamPotterMemorialPrizeinClinicalMedicine,theArthurKrieger
MemorialPrizeinFamilyMedicine,theDrs.RonaldR.andHenryPohlPrizeforProfessionalism,
andtheJ.SavacoolPrizeinMedicalEthics.
JamesS.Studdiford,MD,FACP,isanAssociateProfessorofFamilyandCommunityMedicine
atJeffersonMedicalCollegeofThomasJeffersonUniversity.Hehasaspecialinterestinmedical
photographyhefoundedanddevelopedamedicalimagingdatabase,JeffersonClinicalImages
(JCI),inconjunctionwithAcademicInformationServicesandResearchofScottMemoriallibraryat
ThomasJeffersonUniversity.TheJCIdatabaseincludeswriteupsforeachimageandexceeds1000
entries.DoctorStuddifordalsohasanactiveinterestinmedicalstudenteducation.Hehasreceived
severalawardsforteaching,includingSeniorClassPortrait,HumanisminMedicineAward,andThe
LindbackAwardforDistinguishedTeachinginaClinicalScience.HehasalsodeliveredTheOathof
HippocratesforthegraduatingseniorclassofJeffersonMedicalCollegeonseveraloccasions.

Copyright2015Elsevier,Inc.Allrightsreserved.

BOOKCHAPTER

ContributorsandConsultants
CindyFindenMD,StevenG.FindenMD,DDS,GregaryD.MarhefkaMD,FACC,JeffreyMathewsMD,Kathryn
P.TrayesMD,AbelMorrisBumgarnerMD,JessicaHamiltonMD,MPH,JosephMegaMD,MPH,Jolion
McGreevyMD,MBE,MPH,SamanthaA.SmithMD,JoshuaH.BarashMD,GeoffreyD.MillsMD,PhD,LouisR.
PetroneMD,GeorgeValkoMD,ArmondeBaghdanianMD,ArthurBaghdanianMD,VictorA.DiazMD,Alison
GrantMD,MohitGuptaMD,AnneNasonHackmanMD,ThomasM.KennedyMD,DailaPravsMD,Trisha
SchimekMD,MSPH,KevinCharlesScottMDandPriyaSharma
USMLEImagesfortheBoards,ixx

SENIORCONTRIBUTORS
CindyFinden,MD
AssistantClinicalProfessorofRadiology,DepartmentofRadiology,ThomasJefferson
UniversityHospital,Philadelphia,Pennsylvania
StevenG.Finden,MD,DDS
AssistantProfessorofRadiology,DivisionofNeuroradiology/ENT,ThomasJefferson
UniversityHospital,Philadelphia,Pennsylvania
GregaryD.Marhefka,MD,FACC
AssistantProfessorofMedicine,DepartmentofInternalMedicine,DivisionofCardiology,
JeffersonHeartInstitute,Philadelphia,Pennsylvania
JeffreyMathews,MD
Resident,DepartmentofRadiology,ThomasJeffersonUniversityHospital,Philadelphia,
Pennsylvania
KathrynP.Trayes,MD
AssistantProfessor,DepartmentofFamilyandCommunityMedicine,JeffersonMedical
College,ThomasJeffersonUniversity,Philadelphia,Pennsylvania
Contributors
AbelMorrisBumgarner,MD
JeffersonMedicalCollege,ThomasJeffersonUniversity,Philadelphia,Pennsylvania
JessicaHamilton,MD,MPH
KaiserPermanenteEastBayOB/GYNResidencyProgram,Oakland,California

JosephMega,MD,MPH
DepartmentofFamilyMedicine,ContraCostaRegionalMedicalCenter,Martinez,California
JolionMcGreevy,MD,MBE,MPH
Resident,DepartmentofEmergencyMedicine,BostonMedicalCenter,Boston,Massachusetts
SamanthaA.Smith,MD
Resident,DepartmentofFamilyMedicine,YorkHospital,York,Pennsylvania
SENIORCONSULTANTS
JoshuaH.Barash,MD
AssistantProfessor,DepartmentofFamilyandCommunityMedicine,JeffersonMedical
College,ThomasJeffersonUniversity,Philadelphia,Pennsylvania
GeoffreyD.Mills,MD,PhD
AssistantProfessor,DepartmentofFamilyandCommunityMedicineandDepartmentof
Physiology,AssistantResidencyProgramDirector,JeffersonMedicalCollege,Thomas
JeffersonUniversity,Philadelphia,Pennsylvania
LouisR.Petrone,MD
ClinicalAssistantProfessor,DepartmentofFamilyandCommunityMedicine,Jefferson
MedicalCollege,ThomasJeffersonUniversity,Philadelphia,Pennsylvania
GeorgeValko,MD
GustaveandVallaAmsterdamProfessorofFamilyandCommunityMedicine,ViceChairfor
ClinicalPrograms,DepartmentofFamilyandCommunityMedicine,JeffersonMedicalCollege,
ThomasJeffersonUniversity,Philadelphia,Pennsylvania
Consultants
ArmondeBaghdanian,MD
JeffersonMedicalCollege,ThomasJeffersonUniversity,Philadelphia,Pennsylvania
ArthurBaghdanian,MD
JeffersonMedicalCollege,ThomasJeffersonUniversity,Philadelphia,Pennsylvania
VictorA.Diaz,MD
AssistantProfessor,AssistantMedicalDirector,Director,QualityImprovement,Jefferson
FamilyMedicineAssociates,Philadelphia,Pennsylvania
AlisonGrant,MD
Resident,CrozerKeystoneFamilyMedicine,Springfield,Pennsylvania
MohitGupta,MD
JeffersonMedicalCollege,ThomasJeffersonUniversity,Philadelphia,Pennsylvania

AnneNasonHackman,MD
Instructor,DepartmentofFamilyandCommunityMedicine,ThomasJeffersonUniversity
Hospital,Philadelphia,Pennsylvania
ThomasM.Kennedy,MD
JeffersonMedicalCollege,ThomasJeffersonUniversity,Philadelphia,Pennsylvania
DailaPravs,MD
Instructor,DepartmentofFamilyandCommunityMedicine,JeffersonMedicalCollege,
ThomasJeffersonUniversity,Philadelphia,Pennsylvania
TrishaSchimek,MD,MSPH
JeffersonMedicalCollege,ThomasJeffersonUniversity,Philadelphia,Pennsylvania
KevinCharlesScott,MD
InstructorandPrimaryCareResearchFellow,DepartmentofFamilyandCommunity
Medicine,JeffersonMedicalCollege,ThomasJeffersonUniversity,Philadelphia,Pennsylvania
PriyaSharma
ThirdYearMedicalStudent,JeffersonMedicalCollege,ThomasJeffersonUniversity,
Philadelphia,Pennsylvania

Copyright2015Elsevier,Inc.Allrightsreserved.

BOOKCHAPTER

BoardImages
AmberS.TullyMDandJamesS.StuddifordMD,FACP
USMLEImagesfortheBoards,Section1,33

Notes

1.A45yearoldwomanpresentstoclinicwithanewpruriticrashontheflexorsurfaceofher
leftforearm.HermedicalhistoryissignificantforWilsondisease,andshewasrecentlystarted
onpenicillamine,acopperchelator.OnexamshehasKayserFleischerringsinbotheyesand
purple,polyangularpapulesonherleftforearm.Oncloserinspection,finewhitestreakscover
thesurfaceofthepapules.Aclinicaldiagnosisoflichenplanusismade.Penicillamineis
discontinued.Whatisthebesttreatmentoptionatthistime?
A.Oralsteroids
B.Oralantihistamine
C.Topicalsteroids
D.Azathioprine
E.BandConly

2.A36yearoldmanpresentstotheclinicwithapruriticrashonhisscalp,neck,andback.His
socialhistoryissignificantforintravenous(IV)druguse.Onexam,violaceous,polygonal
papulesarenotedinclustersonthescalp,neck,andback.Whitestriaearenotedonthe
lesionalsurfaces.Onbuccalmucosatherearetender,dendritic,lacy,whitelesionsbilaterally.
Whichofthefollowingis/aretrueregardinghisdiagnosis?
A.CanbeassociatedwithchronicHCV
B.Diseaseistreatedwithoralsteroids
C.Thecoursecanbechronic
D.Diseaseistreatedwithtopicalsteroids
E.Alloftheabove
1.TheanswerisE:BandConly.Thispatienthaslichenplanus(LP)asacutaneous
inflammatoryreactiontopenicillamine.ClinicalidentifiersforLPincludetheFourPs(purple,
polygonal,pruritic,papules),aswellasWickhamstriae,thefinewhitelinescoveringthe
papules.OtherdrugexposuresthathavebeenlinkedwithLPincludegold,chloroquine,and
methyldopa.Thisisalocalrash,sothebestinitialtreatmentwouldbetopicalsteroidandoral
antihistamineforrelieffromitching(E).Oralsteroidsandmorepotentimmunemodulators
(azathioprine,cyclosporine)shouldbeusedwithmoregeneralized,systemicLP.
2.TheanswerisE:Alloftheabove.Thispatienthasoralandcutaneouslichenplanus
(LP)mostlikelyrelatedtochronicHCVstatus.Biopsiesondermalandorallesionsreveal
mononuclearcellsatthedermoepidermaljunctionandaTcellmediatedcytotoxicreaction
againstkeratinocytes.HCVisthoughttotriggerLPviadermalandmucousmembrane
replication.LPusuallyaffectsflexorsurfacessuchasthewrists,pretibialshafts,scalp,trunk,
andglanspenis,andmayalsoinvolvethebuccalmucosa,tongue,andlips.Thispatienthas
localcutaneouslesionsthatcouldbetreatedwithtopicalsteroids.However,duetohisbuccal
mucosaLPheshouldbestartedonashortcourseoforalprednisone(E).Antihistamines
shouldalsobeusedtotreatpruritis.ThecourseofLPislargelyunpredictable,rangingfrom
spontaneousremissiontochroniceruption.

LichenPlanus

Lichenplanusisacommon,distinctinflammatorydisorderthataffectstheskin,mucous
membranes,nails,andhair.Lesionsappearassymmetric,grouped,erythematoustoviolaceous,flat
topped,polygonalpapules.Closeinspectionofthelesionswithahandlensandafterapplicationof
mineraloilwillrevealfinewhitelines(Wickhamstriae).Etiologyofthedermatosisisdiverseand
includesdrugs,metals,andinfections(especiallyhepatitisC),whichresultinalterationsofcell
mediatedimmunity.Womenareaffectedmoreoftenthanmen,andthetypicalageofonsetis
between30and60.TraumamaycausetheKoebnerphenomenonandlineararrangements.
Treatmentiswithtopicalandsystemiccorticosteroidsorcyclosporine.
FourPs:purple,polygonal,pruritic,papules
Cutaneouslesions:distributedtypicallyonflexuralaspectsofarmsandlegsbutmaybecome
generalizedsymptomispruritis
Orallesions:milkywhitereticulatedpapules,whichmaybecomeerosiveorulcerate
symptomispain
Notes

3.A47yearoldmanpresentswithapainlessenlargingmassonthebackofhisrightupperleg.
Hehasnotendernessoverthearea,andthemassisfirmonpalpation.Abiopsyrevealsa
leiomyosarcoma.Allofthefollowingaretrueexcept:
A.Thistumorgrowsbydirectlocalextension.
B.Thistumorisderivedfrommaturefatcells.
C.Thistumorisderivedfromembryonicmesoderm.
D.Treatmentoptionsareguidedbybiopsyresults.

E.Themainprognosticfactorsaretumorgradeandtumorsize.
3.TheanswerisB:Thistumorisderivedfrommaturefatcells.Tumorsormaturefat
cells(B)areseeninbenignlipomasratherthansarcomas.Thismanhasaleiomyosarcoma,
whichisamalignantcancerderivedfromembryonicmesoderm(C).Thistumorgrowsby
directlocalextension(A).Treatmentoptionsareguidedbybiopsyresults(D)andthemain
prognosticfactorsaretumorgradeandtumorsize(E).Thusalloftheabovearetrueof
sarcomasexcept(B).

Sarcoma

Asarcomaisamalignantneoplasmthatarisesfrommesenchymalembryoniccellsandaffects
connectivetissuecellssuchasbone,cartilage,muscles,bloodvessels,orfatcells.Sarcomascanbe
dividedintotwogroups:thosederivedfromboneandthosederivedfromsofttissue.Thesetumors
areoftenhighlyaggressive,andbiopsyisrequiredforidentificationofcancercellsandtoguide
treatment.Imagingsuchasultrasound,computedtomography(CT),ormagneticresonanceimaging
(MRI)maybeusefulbeforebiopsyisperformed.Treatmentoptionsincludesurgicalexcision,
radiation,andchemotherapy.Inadultsthemostcommonhistopathologicsubtypesareliposarcoma
andleiomyosarcoma,andthemostcommonsitesoforiginarethethigh,buttock,andgroin.In
children,smallcellsarcomas(e.g.,Ewingsarcoma,embryonalrhabdomyosarcoma,andprimitive
neuroectodermaltumor)aremostcommon.

KaposiSarcoma
Kaposisarcoma(KS)istheonlysarcomathatisassociatedwithavirus.
Growsbydirectlocalextension
Softtissuesarcomathatoftenpresentsaspainless,enlargingmass

Mainprognosticfactorsaregradeandtumorsize.
Radiationexposureisariskfactorforsofttissuesarcomas.
Notes

4.A33yearoldmancomestotheclinicinthewinterwithcomplaintsofdry,itchyskinonhis
forearms(shownabove)andeyelids,aswellasinthecreasesofhiselbowsandknees.Hispast
medicalhistoryissignificantformild,persistentasthmaandseasonalallergies.Hesaysthat
sincechildhoodthisproblemhasoccurredannuallyinwinter,whenhisasthmaisalsoworse
thanusual.Duringthevisitheisvisiblyuncomfortableandoftenscratcheshisarms.Whichof
thefollowingshouldbeconsideredwhenusingatopicalcorticosteroidtotreatthiscondition?
A.Topicalcorticosteroidsarecontraindicatedinthesettingofsecondarystaphylococcal
infection.
B.Skinbleachingcanoccurwithchronicuse.
C.Applicationneartheeyeincreasesriskforocularsideeffects.
D.Applicationoveralargesurfaceareacancausesystemiccorticosteroideffects.
E.Alloftheabove
5.

A27yearoldmancomplainsofanitchyrashonhisface.Therashappearedjustafterhe
returnedfromadayatthebeach.Herecentlyboughtanewsunscreenthathefirstusedtwo
weeksago.Heusedthesamesunscreenonhisfaceduringhismostrecenttriptothebeach.He
hashadsimilarrashesthroughouthislife.Hehasahistoryofhayfeverandmildasthma.Other
thandiscontinuinguseofthesunscreen,whichofthefollowingpreventivemeasuresshouldbe
recommended?
A.Limitthenumberofshowersperday
B.Useadryingsoap
C.Takehotbaths
D.Applytopicalcorticosteroiddaily
E.Applyantibacterialointmentdaily
4.TheanswerisE:Alloftheabove.Thispatienthasatopicdermatitis(eczema).The
classictriadofatopicdiseaseincludesasthma,allergicrhinitis,andatopicdermatitis.Topical
corticosteroidsareusedtotreatsubacute,scalylesionsorchronic,dry,lichenifiedlesionsof
atopicdermatitis.Theymust,however,beusedwithcautionforallofthereasonslistedabove(
E).Patientswitheczemaareatanincreasedriskofdevelopingasecondaryinfection,
commonlyduetostaphylococcus,whichisacontraindicationtotopicalcorticosteroids.
Chronicuseoftopicalcorticosteroidsmaycauseskinbleaching,andtheriskofsystemicside
effectsincreaseswithapplicationoveralargesurfaceareaorinsmallerindividuals.Useonthe
eyelidsisnotrecommendedduetotheriskofdevelopingcataractsorglaucoma.
5.TheanswerisA:Limitthenumberofshowersperday.Preventionofatopic
dermatitis(eczema)includesavoidanceofknowntriggers.Inthiscase,sunscreentriggeredthe
mostrecentreactionhowever,otheragentsmayhavebeenassociatedwiththepatient'spast
rashes.Patientsshouldnottakehotbaths(C)ormultipleshowersperday(A),andthey

shouldavoiddryingsoaps(B).Topicalsteroidscanbeusedforsubacuteandchronicreactions
butnotasapreventivemeasure(D).Steroidsshouldbeusedwithcautionontheface.Eczema
putspatientsatincreasedriskofsecondarybacterialinfection,butprophylaxiswithatopical
antibacterialointmentistypicallynotindicated(E).

AtopicDermatitis(Eczema)
Atopicdermatitisisaninflammatoryskindisease.Itstemsfromamixtureofgeneticsusceptibility,
defectsintheinnateimmunesystem,andincreasedimmunologicresponsestoallergens.Most
patientsshowsignsofskindiseasebeforeage5.Lesionshaveadifferentappearancebasedonthe
duration(acuteversussubacuteversuschronic)andtheageofthepatient.Youngpatientsoftenhave
arashonthefaceandextensorsurfaces,whileadultshavearashontheflexuralsurfacesofthe
elbowsandknees.
Commonlyassociatedwithapersonalandfamilyhistoryofasthmaandallergicrhinitis
(atopictriad)
Decreasedskinbarrierprotectionleadstoanincreasedriskofsecondarybacterialinfection.
Preventivetreatmentincludesmoisturizersandavoidanceofknowntriggers.
Subacuteorchroniclesionsaretreatedwithtopicalcorticosteroids.
6.

A20yearoldmanpresentswitha0.5cmflatnonpalpablehyperpigmentedlesiononhis
trunk.Howwouldyouclassifyhisprimaryskinlesion?
A.Macule
B.Patch

C.Plaque
D.Papule
E.Lichenification
Notes

7.A75yearoldwomanpresentswithseverallesionsonthedorsalsurfaceofbothofherhands.
Thesehyperpigmentedlesionsareflatandnonpalpable,havedistinctborders,andrangefrom
0.1to0.8cminsize.Whichofthefollowingbestclassifiesthisprimaryskinlesion?
A.Patch
B.Plaque
C.Papule
D.Macule
E.Nodule
6.TheanswerisA:Macule.Thispatienthasamacule(A)becauseitisaflatlesionless
than10mmindiameter.Apatch(B)isalargernonpalpablelesiongreaterthan10mm.
Plaques(C)areelevatedpalpablelesionsgreaterthan1cm.Papules(D)arepalpablelesions
lessthan5mm.Lichenification(E)isasecondaryskinlesioncharacterizedbyepidermal
thickening.Visibleandpalpableskinthickeningisoftenpresentwithaccentuatedskin
markings.
7.TheanswerisD:Macule.Thispatienthasseveralmacules(D)onthedorsalsurfaceof
herhands,likelyrepresentingbenignlesionscalledsolarlentigines.Thetermmaculeisusedto
classifyanyflatlesionoflessthan10mmindiameterthatisevenwiththesurfaceof
surroundingskinanddiffersincolorfromthesurroundingskinormucousmembrane.

Maculesmaybehyperpigmented,hypopigmented,ordepigmented.Patches(A)arelarger,flat
nonpalpablelesionsmeasuringgreaterthan10mminsize.Plaques(B)areelevatedpalpable
lesionsgreaterthan10mm.Papules(C)arepalpablelesionslessthan5mm.Anodule(E)is
asolid,round,orellipsoidalpalpablelesionwithadiameterlargerthan5mm.Nodulesare
differentiatedfrompapulesandplaquesbydepthofinvolvementand/orsubstantive
palpability.

Macule

Amaculeisaprimaryskinlesiondefinedbyanonpalpablechangeinsurfacecolorwithoutelevation
ordepressionthatisgenerallylessthan10mminsize.
Flat,nonpalpable,lessthan10mm
Primaryskinlesion
Examplesincludevitiligo,tineaversicolor,cafaulaitspots,mongolianspots,freckles.
Notes

8.A69yearoldmanpresentswithseveralexophytic,brownpapulesandplaquesthatappear
asthoughtheycouldbescrapedoff.Whattypicalfindingwouldyouexpecttoseeonhistology?
A.Islandsofproliferatingepitheliumresemblingthebasallayeroftheepidermis
B.Hyperplasiaofbenign,basaloidepidermalcellswithhornpseudocysts
C.Focalincreaseinmelanocytes
D.Intraepidermalatypiaoverasundamageddermis
E.Intraepidermalatypicalkeratinocyteswithpenetrationofthebasementmembrane
9.A74yearoldmancomestoyou,hisnewprimarycarephysician,afterbeingurgedbyhis
granddaughtertogetsomespotsonhisbacklookedat.Physicalexamdemonstratesnumerous
stuckon,waxy,verrucouspapulesandplaquesofvarioussizesandcolors.Whichofthe
followingiscorrectregardingthisman'scondition?
A.Thesegrowthsoriginateinkeratinocytes.
B.Similartowarts,theselesionsareviralinorigin.
C.Theselesions,ifleftuntreated,mayprogresstomelanoma.
D.Histologicexaminationwoulddemonstrateafocalincreaseinmelanocytes.
E.Thesegrowthscouldhavebeenavoidedifadequatepreventivemeasureshadbeen
taken.
8.TheanswerisB:Hyperplasiaofbenign,basaloidepidermalcellswithhorn
pseudocysts.Thesewarty,brownlesionswithastuckonappearanceareseborrheic
keratoses.Biopsywouldshowhyperplasiaofbenign,basaloidepidermalcellswithhorn
pseudocysts(B).Hornpseudocystsarevirtuallypathognomonic.Islandsofproliferating
epitheliumresemblingthebasallayeroftheepidermis(A)isthehistologicfindinginbasal
cellcarcinoma.Afocalincreaseinmelanocytes(C)isseeninlentigo.Intraepidermalatypia
overasundamageddermis(D)isseeninactinickeratosis.Intraepidermalatypical
keratinocyteswithpenetrationofthebasementmembrane(E)isindicativeofsquamouscell
carcinoma.
9.TheanswerisA:Thesegrowthsoriginateinkeratinocytes.Thesewartybrown
lesionsareseborrheickeratosesandoriginateinkeratinocytes,hencethename(A).While
theselesionsareoftenreferredtoasseborrheicwarts,thereisnotaviralassociation(B).

Theselesionsarebenigninnatureanddonotcommonlyprogresstomelanoma(C).Afocal
increaseinmelanocytesisseeninlentigo(D).Whilepreventivemeasuressuchassunscreen
useandprotectiveclothingareusefultopreventmalignantskinlesions,seborrheickeratoses,
commonlyreferredtoasagebarnacles,aresimplyaproductofaging.

SeborrheicKeratosis

Seborrheickeratosesareraised3to20mmlesionswithastuckonappearance.Theycanbeflator
raisedandtypicallyhaveavelvetyorwartysurface.Althoughthelesionsareoftenhyperpigmented,
theyrangeincolorfromlighttantoblack.Thesebenignepidermalgrowthsarearesultof
proliferationofimmaturekeratinocytes.Theytendtodevelopafterage50.Atypicallesionscanbe
biopsiedtoruleoutcancer.Shavebiopsyrevealshorncystswhicharevirtuallypathognomonic.
Usuallynotreatmentisnecessary,butsurgicalexcisionisanoptionforcosmeticreasons.
Increasedincidencewithincreasingage
Commonbenignskingrowth
Biopsyshowshorncysts
Notreatmentnecessary
Notes

10.Onaroutinewellchildexamofan8yearoldgirl,thepatient'smotherraisesconcernfor
herdaughter'ssusceptibilitytoskincancerduetothepatient'sfaircomplexionwithmany
frecklesonherfaceandextremities.Thepatient'smothernotesthatherdaughter'sskinburns
easilywhenoutinthesun.Mostofthefamilyisunabletotanwithsunexposure.Herfamily
historyissignificantforamaternalgrandmotherwithbasalcellcarcinomaandpaternalaunt
withmelanoma.Onexamthepatientisahealthy,faircomplexionedgirlwithlightblueirides
andareasofdenseephelides(freckles)onherface,shoulders,arms,andlegs.Youcounselthe
patientandhermotherofherriskofskincancer.Whichofthefollowingprovide(s)effective
preventionforskincancerinthispatient?
A.Avoidexcessivesunandultraviolet(UV)lightexposure.
B.Usetitaniumdioxideandzincoxidesunscreens.
C.Usehydroquinonetobleachfreckles.
D.Useskincoveringclothingandhats.
E.Alloftheabove
F.OnlyA,B,andD
10.TheanswerisF:OnlyA,B,andD.Thispatientexhibitsthephenotypeofthosewith
thehighestlifetimeriskofskincancer(melanoma,basalcellcarcinoma,squamouscell
carcinoma):fairskinwithanabundanceoffreckles,lightiridesusuallywithblondorredhair,
unabletotan,andeasilysunburned.ExposuretoultravioletA(UVA)andultravioletB(UVB)
lightposesahigherriskintheseindividuals.FrequentUVlightexposureandblistering
childhoodsunburnsputtheseindividualsathigherriskformelanoma.ChronicUVlight
exposurecorrelatesmorewithahigherriskofbasalcellcarcinomaandsquamouscell
carcinoma.ParamounttoskincancerpreventionisavoidingandblockingexposuretoUVAand
UVBlight(F).Hydroquinonehasbeenshowntobleachfreckles,butitoffersnoadded
protectionfromUVlight.

Ephelis(freckle)

Anephelisiscommonlyknownasafreckleandfirstappearsduringchildhoodinfairskinned
individuals,presumablydevelopingaftersunexposure.Whensunexposureisdiscontinued,the
ephelideswilltypicallyfadeormayevendisappear.Theyrepresentanincreaseinmelanin
productioninresponsetoUVradiationexposure.Itmaybedifficulttodistinguishclinicallybetween
anephelisandsolarlentigo.
Commononthecentralfaceandfirstnotedinchildhood
Moreprominentafterexposuretosunlight,fadingaftercessationofsunlightexposure
Familialinheritanceandmorecommonlyseeninfairskinnedindividuals
Notes

11.A13yearoldgirlpresentstoyourclinicwiththechiefcomplaintofseveralredlumpson
herback,arms,legs,andforeheadwhichhavebeengrowinginnumberfor2months.Sheadds
thatshehashadcoffeecoloredmarksonherskinsinceshewasatoddler.Herpastmedical
historyissignificantforalearningdisabilityandrecentonsetofpuberty.Onphysicalexamthe
patientisTannerstage2inbreastandpubichairdevelopment.Shehaseightcafaulaitspots
onhertrunk,numerousneurofibromasoverherbody,andaxillaryfreckling.Sheisdiagnosed
withneurofibromatosis1(NF1).Whichofthefollowingcanbeusedregardingmanagementof
thiscondition?
A.Imagingofthebrain,orbits,chest,spine
B.Eyeexam
C.Geneticcounseling
D.Surgicalremovalofdermalneurofibromas
E.Spineexam
F.Alloftheabove
11.TheanswerisF:Alloftheabove.Thispatientexhibitsthreeofthediagnosticcriteria
forNF1:1)Sixormorecafaulaitspots2)twoormoreneurofibromas3)frecklingin
axillary/inguinalregion4)opticgliomas5)twoormoreirishamaratomas(Lischnodules)6)
bonylesions(sphenoiddysplasia/pseudoarthroses)7)firstdegreerelativewithNF1.Onlytwo
ofthesecriteriaareneededfordiagnosisofNF1.PatientsshouldbereferredtoNFspecialists
whocanuseimaging,usuallyinthesettingofneurologicdeficits,toassessforgliomas,
sphenoiddysplasia,pseudoarthroses,andspinallesions.Anannualophthalmologicalexamis
neededtoassessforopticpathwaygliomasand,toalesserextent,Lischnodules.Thespine
shouldalsobeevaluatedregularlyforsignsofscoliosis.Dermalneurofibromascanbe
surgicallyremovedforcosmeticreasonsorformalignantdegeneration.FinallyallNFpatients
shouldreceivegeneticcounseling,asthisisanautosomaldominantdisorderwith50%chance
ofinheritanceintheiroffspring(F).

Neurofibromatosis

Thispatienthasmultipleskincolored,softpapulesandpedunculatednodulesonherabdomen.
Theseneurofibromatouslesionsareseeninassociationwithneurofibromatosis(NF),anautosomal
dominantdiseasecharacterizedbychangesintheskin,nervoussystem,endocrineglands,andbones.
Centralnervoussystem(CNS)involvementoccursin10%ofpatientswithNFandconsistsofbenign
lesionsincludingacousticneuromas,meningiomas,opticglioma,andinsomecasesastrocytomas.
Thusneurologicsignsandsymptomsshouldbeapproachedwithahighindexofsuspicion.
Pheochromocytomasoccurin1%ofpatients,causingseverehypertension.Notecafaulait(CAL)
maculesonabdomen,alsoaskinfindingassociatedwithNF.
NF1:twoormoreofthefollowing:sixCALspots(>15mminadults),twoneurofibromas,
frecklingintheaxilaoringuinalarea,opticglioma,twoLischnodules,osseousabnormality,
andafirstdegreerelativewithNF1
NF2:bilateralacousticneuromas,afirstdegreerelativewithNF2,andbilateralacoustic
neuromasoranytwoofthefollowing:neurofibromas,meningiomas,gliomas,orschwannoma
NF2featuresincludeseizures,skinnodules,andCALspots
Notes

12.A34yearoldwomanpresentswiththinwalled,fluidfilledblistersoverhertrunkand
extremities.Theblisterseasilyrupture.Hertemperatureonarrivalis102F,andherblood
pressureis85/50.Younotepeelingskinoverherpalmsandsoles.Thewomanwastreated3
weeksagoforaurinarytractinfection.Thewomantakesoralcontraceptivepills,butshetakes
noothermedications.Herlastmenstrualperiodjustended,andsheusedanew
superabsorbentbrandoftampons.Whatisthecauseofthissyndrome?
A.Bacterialreleasedexotoxins
B.Bacterialreleasedendotoxins
C.Medicationreaction
D.Postviralsyndrome

E.Antibodiesdirectedagainstdesmogleinmolecules
13.A33yearoldwomanpresentstotheemergencydepartment(ED)withsuddenonsetfever,
chills,vomiting,diarrhea,muscleaches,andadiffuserash.Youareincrediblybusyandhave
nothadachancetoseethispatientwhenthenursealertsyouthatshehasdevelopedsevere
hypotensionandisbeginningtodecompensate.Youstartsupportivetherapyandbegintotake
ahistory.Shecogentlyasksyouifyouthinkthishasanythingtodowiththespongethatshe
usesforcontraception.Shehadplacedthespongeaweekagoandrecallsthatsheforgottotake
itout.Whichofthefollowingisthemostlikelycauseofhersymptoms?
A.Bacterialendotoxins
B.Viralsyndrome
C.Allergicreaction
D.Bacterialexotoxins
E.Autoimmunereaction
12.TheanswerisA:Bacterialreleasedexotoxins.Thiswomanhastoxicshock
syndrome(TSS),likelyduetoheruseofsuperabsorbenttampons.TSSiscausedbybacterial
exotoxins(A).MosttypicallytheexotoxinsarereleasedbyStaphylococcusaureus,butcan
alsobeduetogroupAstreptococcus.Theseexotoxinscausedetachmentwithintheepidermal
layer.Bacterialendotoxins(B)aremostoftenassociatedwithgramnegativebacteria,which
donotplayaroleinTSS.Medications(C)andareactiontoaviralinfection(D),suchas
HSV,arecommoncausesofStevensJohnsonsyndrome(SJS),whichclassicallyinvolvedsthe
mucousmembranesandoralmucosa.Antibodiesdirectedagainstdesmogleinmolecules(E)
causespemphigusvulgaris.
13.TheanswerisD:Bacterialexotoxins.ThispatientislikelysufferingfromTSSdueto
theuseofanintravaginalcontraceptivedevice(sponge,anolderformofcontraception).
Menstruatingwomen,womenusingintravaginalcontraceptivedevices,peoplewhohave
undergonenasalsurgery,andpersonswithpostoperativestaphylococcalwoundinfectionsare
allatriskforTSS.TSSiscausedbybacterialexotoxins(D).Fever,chills,vomiting,diarrhea,
andrapid,severehypotensionarecharacteristicoftheinitialcourseofTSS.Desquamation,
particularlyonthepalmsandsoles,canoccur12weeksafteronsetoftheillness.Bacterial
endotoxins(A)aremostoftenassociatedwithgramnegativebacteria,whichdonotplaya
roleinTSS.Viralsyndromes(B)andallergicreactions(C)arecommoncausesofSJS,which
classicallyinvolvesthemucousmembranesandoralmucosa.TSSisnotduetoanautoimmune
reaction(E).

ToxicShock(StaphScaldedSkin)

Toxicshocksyndrome(TSS)isarare,lifethreateningcomplicationofbacterialinfectionthathas
beenmostoftenassociatedwiththeuseofsuperabsorbenttampons.OftenTSSresultsfromtoxins
producedbyS.aureus,butitmayalsobeproducedbytoxinsproducedbygroupAstreptococcus.S.
aureusreleasesepidermolyticexotoxinsAandB,whichcausedetachmentwithintheepidermal
layer.Theseexotoxinsareproteasesthatcleavedesmoglein1,whichnormallyholdsthegranulosum
andspinosumlayerstogether.TSSpresentswiththinwalled,fluidfilledblistersthateasilyrupture.
Symptomsmayalsoincludefever,lowbloodpressure,vomiting,diarrhea,orarash,whichmaylead
todesquamation(especiallyofthepalmsandsoles).
Causedbyexotoxins
Detachmentwithinepidermallayer
Treatwithantibioticsandsupportivecare
Thinwalledblistersanddesquamationofskin(particularlyhandsandsoles)

Notes

14.A54yearoldmanisintheclinicforanannualvisit.Duringthephysicalexamination,he
inquiresaboutmultipleredentitiespresentonhischest,upperarms,andback,asshown
above.Thelesionsarecherryredand1mmindiameter.Theyhavebeenthereforafewyears
accordingtothepatient.Hehasnootherhealthcomplaintsandnosignificantpastmedical
history.Whatisthebestnextstepinworkupforthispatient?
A.Investigationforavisceraladenocarcinoma
B.Biopsiesofthelesionstoruleoutmalignancy
C.Mohsprocedureforremoval
D.Electivelaserremovalorelectrocoagulation
E.Widelocalexcision
14.TheanswerisD:Electivelaserremovalorelectrocoagulation.Thispatienthasa
normalfindingofcherryangiomas,whichrepresentbenignvascularlesionsthataremore
commonwithincreasingage.Thehistologyofthelesionsconsistsofdilatedcapillariesandan
edematousstromawithhomogenizationofcollagen.Theyareextremelycommonintheelderly
andhavenoclinicalsignificance(A,B).However,lesionsthatgeneratecosmeticconcernmay
betreatedwithlasertherapyorelectrocoagulation(D).Thesefindingsdonotwarrantmore
invasivetreatment(C,E).

CherryHemangioma

Cherryhemangiomas(orcherryangiomas)arecommon,asymptomatic,domedvascularlesionsthat
varyincolorfrombrightredtoviolaceousorblack.Theyarefoundtypicallyonthetrunk,appearing
aroundage30andincreasinginnumberswithage.Theyhavenoclinicalsignificancebutmayposea
cosmeticconcern.Ifdesired,theycanbetreatedwithlaserremovalorelectrocoagulation.
Oftenlessthan3mmindiametersomesmallerlesionsareonlydistinguishedfrom
petechiaebytheirpermanence.
Histologyconsistingofnumerousdilatedcapillarieslinedbyflattenedendothelium,
edematousstroma,andhomogenizedcollagen
Numbersincreasewithagebuthavenoclinicalsignificance.
Notes

15.A12yearoldimmigrantboyfromBrazilpresentstotheclinicwith5daysofcough,
conjunctivitis,fever,andrunnynose.Hismotherbroughthimintoday,alarmedbyarashthat
beganonhisheadandprogressedoverhisfaceandneckdownhisarmstohishands.His
medicalchartissignificantonlyforabsentMMRandTdapvaccinations.Onphysicalexama
diffuseerythematous,maculopapularrashisnotedonhisface,neck,chest,andupper
extremities.Onthebuccalmucosaoftheinnercheeks,bluishwhitespotsarenotedon

erythematousbasesbilaterallyatthelevelofthefirstmolars.Whichis/arethebesttreatment
option(s)atthistime?
A.Antiviraltherapy
B.VitaminA
C.Supportivecare
D.PenicillinG
E.Alloftheabove
16.A3yearoldgirlwhohasnovaccinationrecordspresentstoanemergencySaturdayclinic
withafever,constantcough,runnynose,conjunctivitis,diarrhea,andvomitingfor3days.On
examthepatientisfebrileandappearsacutelysickwithcervicalandoccipital
lymphadenopathy.Onoralexambluishwhitespotssurroundedbyredarenotedonthebuccal
mucosaoftheinnercheeks.Aclinicaldiagnosisofmeaslesismade.Supportivecareisbegun.
Thenextdaythepatientdevelopsamaculopapularrashonherforehead,whicheruptsdown
herface,neck,trunk,andreachesherpalmsandsoles.Indiscussingthecourseofthedisease
youtellhermotherthatintermsofacutecomplications,otitismedia(AOM)isthemostlikely,
andpneumoniaisthemostserious.Whichofthefollowingis/areknowncomplicationsofher
disease?
A.Subacutesclerosingpanencephalitis
B.Diarrhea
C.Death
D.Alloftheabove
15.TheanswerisC:Supportivecare.Thispatientwithmeaslesvirushasthe
characteristicexanthem(maculopapularrashstartingontheforeheadandprogressingdown
theface,neck,trunk,andextremities)after24daysofcough,coryza,andfever.Koplikspots
(bluishwhitespotsonred)developonthebuccalmucosapriortotheonsetoftherashandare
pathognomonicformeasles.MeaslesIgMantibodyisconfirmatory.Thebesttreatmentinthe
immunocompetentpatientissupportivecare:hydration,antipyretics,andmaintenanceof
oxygenation(C).VitaminAisusefulifthepatienthassymptomsofdeficiency,malabsorption,
malnutrition,orimmunosuppression.Ribavirinantiviralcouldbeusedwithan
immunocompromisedpatient.Antibioticsarenotindicated.
16.TheanswerisD:Alloftheabove.Thispatienthasmeaslesduetolackofproper
vaccination.Koplikspots(bluishwhitespotsonredonthebuccalmucosa)occurring14days
beforetheexanthemarepathognomonicformeasles.Acutelytheleadingcauseofdeathis
pneumonia.AOMisthemostcommoncomplication.Diarrheacanleadtodehydration.
Subacutesclerosingpanencephalitisisarare,chronicformofmeaslesinfectionintheCNS,

causingfatalneurodegeneration713yearsfollowinginitialinfection(D).Allofthese
complicationscanbeavoidedwithproperMMRvaccination.Sideeffectsofvaccination(fever,
injectionsiterash,andrarelyfebrileseizureandthrombocytopenia)areminimalconsidering
theriskofcomplicationswithmeaslesinfection.

KoplikSpots

Koplikspotsemerge1to2daysbeforethecharacteristicerythematousmaculopapularrash.They
firstappearas1to3mmbluepapuleswithgraywhitecentersonthebuccalmucosaandare
pathognomonicformeaslesinfection.
Pathognomonicformeaslesinfection
Appearas1to3mmbluepapuleswithgraywhitecentersonthebuccalmucosa
Precedetheappearanceofthemaculopapularrashby1to2days
Notes

17.A43yearoldmanpresentswithcomplaintsoflesionsaroundhiseyes.Henotesthatthe
lesionshavebeenpresentforacoupleofyearsandhavenotchangedinsize,shape,orcolor.
Younotethatthelesionsare1mm,raisedandnonfluidfilled.Whatprimaryskinlesiondoes
thispatienthave?
A.Macule
B.Papule
C.Patch
D.Plaque
E.Nodule
18.A10yearoldgirlwithnosignificantpastmedicalhistoryisbroughttoherpediatriciandue
toseveralitchylesionsonherabdomenandbuttocks.Shenotesthattheyhavebeenpresentfor
thepastcoupleofmonthsandseemtoappearandresolveindifferentspots.Shedeniesany
recenttravelorsickcontactsbutdoesnotethatshehasacatwhosleepswithheratnight.On
physicalexam,thelesionsare4mm,raised,erythematousandnonfluidfilled.Howwould
youcharacterizethispatient'sprimaryskinlesion?
A.Macule
B.Patch
C.Plaque
D.Nodule
E.Papule
17.TheanswerisB:Papule.Thispatienthasapapule(B)becauseitisraised,solid,and
lessthan5mm.Macules(A)areflatlesionslessthan10mmindiameter.Apatch(C)isa
largernonpalpablelesiongreaterthan10mm.Plaques(D)areelevatedplateaulikepalpable
lesionsgreaterthan1cm.Nodules(E)aremorphologicallysimilartopapules,buttheyare
greaterthan1cminbothwidthanddepth.Theyaremostfrequentlycenteredinthedermisor
subcutaneousfat.
18.TheanswerisE:Papule.Thispatientpresentswithseveralpruritic,erythematous
papules(E)likelysecondarytofleabitesfromhercat.Apapuleisasolid,elevatedlesionless
than5mminwhichasignificantportionprojectsabovetheplaneofthesurroundingskin.A
macule(A)isanyflatlesionoflessthan10mmindiameterthatisevenwiththesurfaceof
surroundingskinanddiffersincolorfromthesurroundingskinormucousmembrane.Patches

(B)arelarger,flatnonpalpablelesionsmeasuringgreaterthan10mminsize.Plaques(C)
areelevatedpalpablelesionsgreaterthan10mm.Anodule(D)isasolid,round,orellipsoidal
palpablelesionwithadiameterlargerthan5mm.Nodulesaredifferentiatedfrompapulesand
plaquesbydepthofinvolvementand/orsubstantivepalpability.

Papule

Apapuleisaprimaryskinlesiondefinedbyapalpable,solid,wellcircumscribedelevationofskin
measuringlessthan5mmindiameter.
Palpablelesionlessthan5mm
Primaryskinlesion
Maybeassociatedwithsecondaryfeaturessuchascrustsorscales
Notes

19.A50yearoldmanwithahistoryofatopicdermatitispresentstotheclinicwithgolden

crusted,erythematous,weepinglesionsonhisleftarm.Heisafebrileandotherwisehealthy.
Gramstainofthefluidrevealsgrampositivecocciinchains,andcultureoftheweepingarea
growsgroupAstreptococciafter24hours.Whatisthenextappropriatestepintheclinical
managementofthispatient?
A.IVpenicillinandvancomycin
B.WarmcompressesandIVpenicillin
C.Secondgenerationcephalosporin
D.Topicalmupirocinointmentafterremovalofdirt,crusts,anddebriswithsoapand
water
E.Prophylacticampicillinforallclosecontacts
20.A17yearoldwomanpresentstotheclinicwith3daysofaperioral,crusty,yellowrash.She
saysthatcrustingstartedatacrackatthecornerofhermouthandgraduallygrewtoherchin.
Onexamahoneycrustedrashisnotedatthechinandcornersofthemouthwithsomesmall
unrupturedvesiclesonerythematousbases.Aclinicaldiagnosisofimpetigoismade.Whichof
thefollowingis/aretrueregardingthiscondition?
A.Highlycontagious,capableofselfinoculation
B.Canstartatsiteofskininjuryoronintactskin
C.RepresentsaninfectionwithS.aureusand/orgroupAstreptococcus
D.Treatmentistopicalmupirocinointment
E.Alloftheabove
19.TheanswerisD:Topicalmupirocinointmentafterremovalofdirt,crusts,and
debriswithsoapandwater.ThispatienthasimpetigocausedbygroupAstreptococci
basedonclinicalfindingsandconfirmedwithGramstainandculture.Impetigoisasuperficial
skininfectionthatveryrarelyhassystemiceffectsandcanbeeffectivelytreatedwitheither
topicalmupirocinointmentororalpenicillin(D).Removalofcrustsanddebriswithsoapand
waterisahelpfuladjunct.Impetigoisaverycontagious,autoinoculableinfection.Clinical
differentiationbetweenstreptococcalandstaphylococcallesionsisdifficult,andoldererosions
maycontainbothtypesofbacteria.IVantibioticsandprophylaxisofclosecontactsarenot
warranted.
20.TheanswerisE:Alloftheabove.Thispatienthasnonbullousimpetigo,whichis
diagnosedclinicallyassmallvesiclesonaredbase.Asvesiclesrupture,anadherentyellow
brown,honeycoloredcrustforms.Locallyadditionalsitesofclusteredvesicleswillappearas
thisdermalstaphylococcaland/orgroupAstreptococcalinfectiongrows.Areasofskinaround
themouthandnoseareusualsites.Althoughnotnecessary,skininjuryaswellaspreexisting

lesionssuchaseczemaoracoldsorecanleadtoimpetigowiththeintroductionofinfectious
bacteria.Impetigoishighlycontagiousandmostprominentinyoungerchildren.Treatmentof
thislocalsuperficialinfectioninvolvesisolationandavoidanceofphysicalcontactandtopical
mupirocinointment(E).Systemicantibioticsshouldbeusedonlyforextensivecasesof
impetigo.

Impetigo

Impetigoiscausedbyskininfectionwitheitherstreptococciorstaphylococciandappearsascrusted
superficialerosionsthatcontainpurulentmaterialandruptureeasily.Thelesionsmayappearas
macules,vesicles,bullae,pustules,andhoneycoloredgummycrusts.Impetigoisahighlycontagious
infectionandindividualscanoftenautoinoculatethemselves.Thefaceandotherexposedareasare
mostcommonlyaffected.Gramstainandcultureconfirmthediagnosis.
Streptococcalandstaphylococcalcausesofimpetigoareclinicallyindistinguishable,except
forbullousimpetigo.
Preexistinglesionslikescabies,zoster,oreczemacanpredisposepatientstotheinfection.
Gramstainandcultureconfirmthediagnosis.
Notes

21.A42yearoldAfricanAmericanmanpresentstotheofficeforaroutinecheckup.On
examination,hehasdarkenedskinonthebackofhisneck(shownabove),axilla,andelbows,
whichhestateshasbeenthereforafewmonths.Heismildlyoverweight,butotherwisehehas
nosignificantpastmedicalhistoryandpresentswithnoothermedicalcomplaints.Whatisthe
bestnextstepinthispatient'songoingtherapy?
A.Skinbiopsy
B.Fastingglucoselevel
C.CA125,CEA,andCA199tumormarkersforunderlyingneoplasm
D.CATscanoftheabdomenandpelvis
E.Reassurancewithnofurtherworkup
22.A65yearoldAfricanAmericanmanstatesthatmultipleareasofhisskinhavesuddenly
becomedarker.Healsocomplainsofintermittentepigastricpainandaninabilitytofinishhis
mealsoverthelastfewweeks.Hesayshefeelsfullwithminimalfoodintakeandnotesthat
hisclothesseemlooser.HisfastingglucoseandhemoglobinA1carenormal.Whatisthenext
beststep?
A.Topicalretinoid
B.Upperendoscopy
C.Skinbiopsy
D.Recommenddietandexercise
E.Reassurancewithnofurtherworkup
21.TheanswerisB:Fastingglucoselevel.Thispatient'sfindingsrepresenttheclassic
presentationforacanthosisnigricans,whichconsistsofhyperkeratosisandhyperpigmentation
oftheneck,axilla,groin,andotherskinfolds,especiallyonflexuralsurfaces.Thisconditionis
mostcommonlyassociatedwithtype2diabetesmellitusandinsulinresistance.Thereforethe
bestinitialcourseofactiontotakewiththispatientistoobtainafastingglucoselevel(B).
Althoughacanthosisnigricanshasalsobeenseeninpatientswithadenocarcinomasofthe
gastrointestinal(GI)andgenitourinarytract(C,D),especiallygastriccarcinomas,thisisnot
themostlikelyetiologyofthecondition.Typicallycasesassociatedwithcarcinomashavea
rapidonset,andthepatientinthiscasehasaninsidiousonsetofthedisorder.Askinbiopsy(A
)willshowtypicaldermatopathologyfindingsofhyperkeratosis,epidermalpapillomatosis,

slightacanthosis,andpossibleincreasedmelaninpigmentation,butitwillnotprovide
diagnosticvalueinrevealingtheunderlyingdisorder.ChoiceEisincorrectbecauseadiabetes
workupisindicatedinpatientswhopresentwiththisclinicalfinding.
22.TheanswerisB:Upperendoscopy.Acanthosisnigricanswithrapidonsetraises
suspicionformalignancy,mostcommonlygastriccarcinoma.Althoughmalignancyisarare
causeofacanthosisnigricans,rapidonsetofskinchanges,coupledwiththepatient'ssymptoms
ofabdominalpain,earlysatiety,andweightloss,warrantinvestigationforgastriccancerwith
upperendoscopy(B).Acanthosisnigricanslesionssecondarytoinsulinresistance(themost
commoncause)andmalignancyareindistinguishable,soskinbiopsy(C)willnothelpto
identifytheunderlyingcause.Atopicalmedicationtoimprovethecosmeticappearanceofthe
acanthosisnigricans(A)andinitiationofadietandexerciseprogram(D)maybe
appropriateifseriouscausesofacanthosisnigricansareruledouthowever,thepatient's
symptomsplustherapidonsetoftheacanthosisnigricansmakesthesechoices,aswellas
reassurance(E),inappropriateatthistime.

AcanthosisNigricans

Acanthosisnigricansisacutaneousmarkercommonlyassociatedwithinsulinresistanceandless
frequentlywithgeneticdisordersormalignancy.Itischaracterizedbysymmetric,hyperpigmented
plaquesthattypicallyappearonskinfolds,especiallytheflexuralareas.
Acanthosisnigricansmostoftenhasaninsidiousonsetandsignifiesinsulinresistancefrom
anycause,commonlytype2diabetesmellitus.
Diagnosticworkupinitiallyincludestestingfordiabetesmellitusandmetabolicsyndrome.
Imagingtestsandendoscopyshouldbeusedifthereisacuteonsettoruleoutmalignancy.
Treatmentistargetedtotheunderlyingcause,ifidentified.Topicalkeratolyticortopical
retinoidsmayimproveacanthosisnigricans,ascanweightlossinobesepatients.
Notes

23.A4yearoldgirlpresentswitharedpurple,raisedvascularplaqueoverherleftfourth
digit.Thelesionappearedacoupleweeksaftershewasborn.Itgrewrapidlyforaperiodof
time,buthasbeenstableforaboutayear.Whatisthenextstepintreatment?
A.Surgicalexcisionandbiopsy
B.Lasersurgery
C.Cryosurgery
D.Systemicglucocorticoids
E.Notreatment
24.Afirsttimemotherbringsher1montholdinfanttoherpediatricianconcernedbya
growingplaqueonherleftbuttock.Hermomcommentsthattherewasaredmarkpresentat
birthbutisconcernedthatthismarkhasnotresolvedandhasinfactcontinuedtogrowand
nowdemonstratesaraised,vascularappearance.Whichofthefollowingisthebestadvicefor
thisconcernedmother?
A.Advisethemothertotakeherinfanttoadermatologisttoremovethislesionbefore24
monthsofage.
B.Reassurancethatmostoftheselesionsresolvespontaneously
C.Biopsyofthelesiontoruleoutamalignantvascularneoplasm
D.Reassurancethatwhilethislesionwilllikelybepermanentitisnotmalignantin
nature
E.Advisetheuseoftopicalcorticosteroidstospeedresolution.
23.TheanswerisE:Notreatment.Thislesionisastrawberryhemangioma(alsocalledan
infantilehemangiomaoracapillaryhemangioma).Thebesttreatmentfortheselesionsisto
letthemresolvespontaneously(E).About60%resolvebyage5,and90%aregonebyage9.

Treatmentisonlynecessaryifthelesionblocksvision,isinthewayofthenostrils,orif
ulcerationoccurs.Ifsurgeryisnecessary,lasersurgery(B)orcryosurgery(C)maybe
indicated.Surgicalexcisionandbiopsy(A)isnotindicatedinthiscaseasthediagnosiscanbe
madeclinically.Systemicglucocorticoids(D)maybeinitiatediftreatmentisindicated,but
spontaneousresolutiongenerallygivesthebestresults.
24.TheanswerisB:Reassurancethatmostoftheselesionsresolve
spontaneously.Thislesionismostlikelyastrawberryhemangiomaandwillresolve
spontaneously(B).Biopsy(C)ortreatment(D)isnotnecessary,andclinicaldiagnosisis
usuallysufficient.Topicalcorticosteroidshavenotbeendemonstratedtobebeneficial(E),
andreferraltoadermatologistisnotindicatedatthistime.

StrawberryHemangioma

Strawberryhemangiomas(alsocalledcapillaryhemangiomasorinfantilehemangiomas)are
superficialvasculartumorswhichappearasraised,red,lumpyareasoffleshanywhereonthebody,
althoughthemajorityoccurontheheadorneck.Thesehemangiomasareclonalproliferationsof
endothelialcells.Theyusuallyappearabout14weeksafterbirthandmaygrowrapidlybefore
stoppingandthenslowlyfading.About60%resolvebyage5,andabout90%aregonebyage9.
Womenaremoreoftenaffectedthanmales.Theincidenceisincreasedinpreterminfants.No
treatmentisnecessaryunlessthelesionblocksvisionorisinthewayofthenostrils.
Rapidgrowthphasefollowedbyspontaneousinvolution
Increasedincidenceinwomenandinpreterminfants
Resolvespontaneously
Interventionnecessaryifulcerationoccursorifthelesioncausesfunctionalimpairment
Notes

25.A40yearoldmancomestothecliniccomplainingofpainatthebaseofhisrightgreattoe,
whichhasbeengoingonsincehewokeup.Hehaspainwhileambulatingandhadtrouble
puttingonhisshoes.Hehashypertensioncontrolledonhydrochlorothiazide(HCTZ)and
lisinoprilandtakesatorvastatinforhyperlipidemia.Hedrinks12beerseachnightafter
dinner.OnexaminationhehasaBMIof30andstablevitalsigns.Hehasswellingand
erythemaatthebaseoftherighthallux,andhepullshisfootawaybrisklyinpainwhenthis
areaispalpated.Needleaspirationoftheareaisperformed,andbloodworkisdrawnforserum
uricacid.Whatchange(s)maybehelpfulinthispatienttopreventfuturegoutyattacks?
A.Weightloss
B.Cessationofalcoholconsumption
C.Discontinuationofthediuretic
D.Stayingwellhydrated
E.Alloftheabove
26.A70yearoldAfricanAmericanwomanpresentstoclinicwithapainfulbumponher
finger.Herpastmedicalhistoryissignificantforalcoholabuse.Onexamthereisaninflamed
Heberdennodeonherrightfirstdistalinterphalangeal(DIP)jointthatisextremelytenderto
palpation.Labsrevealelevatederythrocytesedimentationrate(ESR)andelevateduricacid
levels.Aspiratefromthenoduledemonstratesnegativelybirefringentneedleshapedcrystals
andawhitebloodcount(WBC)of12,000103/L.Adiagnosisofacutegoutyattackismade.
Whatis/areanappropriatetreatment(s)ofgout?
A.Colchicine
B.Cessationofalcoholconsumption
C.Nonsteroidalantiinflammatorydrug(NSAID)

D.Corticosteroidinjection
E.Alloftheabove
25.TheanswerisE:Alloftheabove.Thispatientissufferingfromacutegoutyarthritis.
Themostcommonlyaffectedlocationisthefirstmetatarsophalangealjoint,andtreatmentof
anacuteattackisaimedatanalgesiaandantiinflammatorymeasures.Needleaspirationof
synovialfluidfromtheaffectedjointwillhelptoruleoutothercauses(e.g.,septicarthritis)and
showthecharacteristicmonosodiumuratecrystalsoncompensatedpolarizedlightmicroscopy.
Thecrystalsareneedleshapedandappearyellowwhenparalleltotheaxisofrotationofthe
compensatorandbluewhenperpendicular.Whileallopurinoltherapyintheintercriticalperiod
betweenattacksmaylessenmorbidityfromgout,issueslikeobesity,hyperlipidemia,diabetes,
alcoholism,smoking,andhydrationtakepriority.Lifestylechangescanhaveapositiveeffect
onapatient'sprognosisafterthefirstattack(E),becauseallthesefactorsaffectpurineand/or
uricacidmetabolisminthebody.
26.TheanswerisE:Alloftheabove.Thispatientissufferingfromanacutetophaceous
goutyarthritis.FirstlinetreatmentincludesNSAIDs(indomethacin,naproxen).Intraarticular
steroidinjectionscanoffermoreimmediatereliefandhavenosystemicsideeffects.Colchicine
candecreaseinflammationifusedearlyinagoutyattack.Bothallopurinolandcessationof
alcoholareeffectiveaslongtermtherapiesinpatientswithhyperuricemiaandrecurrentgouty
attacks(E).

Gout

Goutischaracterizedbyagroupofheterogeneousconditionsthatresultinthedepositionof
monosodiumuratecrystalsinthesynovialfluid,usuallywithassociatedhyperuricemia.Itistypically
characterizedbyfourstages:asymptomatichyperuricemia,acutegoutyarthritis,intercriticalgout
(betweenattacks),andchronictophaceousgout.LeukocytosisandelevatedESRareoftenpresent
duringacuteattacks.Treatmentrevolvesaroundanalgesiaandreductionininflammationinthe
acutesettingwithNSAIDs,colchicine,andcorticosteroids.Chronictreatmentincludesxanthine
oxidaseinhibitorslikeallopurinol.

Acutegoutyarthritistypicallypresentsinthemiddleyearsandaffectsasinglejointinthe
lowerextremities,mostcommonlythefirstmetatarsophalangealjoint.
Affectedjointsareerythematousandextremelytendertopalpation.
Chronictophaceousgoutrarelygrantspatientsperiodswithoutsymptoms.Uratecrystalscan
befoundinthesofttissues,cartilage,andtendons.
Notes

27.A45yearoldmanpresentswithsymmetriclesionsoverhisknees.Thelesionsappearasa
silveryscaleonanerythematousbase.Themanreportsthatheisotherwisehealthy.Younote
thatslightscratchingofthescalylesionsresultsinpunctatebleedingpoints.Whatisthemost
likelycauseoftheselesions?
A.Trauma
B.Lichensimplexchronicus
C.Seborrheicdermatitis
D.Atopicdermatitis
E.Psoriasis
28.A38yearoldmanpresentscomplainingofarashonhislegandchronicpaininhisfingers
andlowerbackforthepast6months.Hehasnosignificantmedicalhistorybutdoesnotethat
hisfathersufferedfromsomethingsimilar.Onphysicalexamyounotelesionswithasilvery
scaleonanerythematousbasethroughoutthepatient'sleftlegandwithpassivemotionofthe
DIPjointsofhisrighthandaswellasmildtendernesstopalpationatthesacroiliacjoint.You

alsoidentifythatthepatienthaspittingonthenailsofhisfingers.Whatwouldbethenextbest
stepindiagnosingthispatient'sdisease?
A.Skinbiopsyoflesionsonleg
B.MRIoflumbarspine
C.Nofurthertests,thediagnosiscanbemadesolelyonclinicalfindings
D.CheckforhumanleukocyteantigenB27(HLAB27)
E.Checkuricacidlevelinurine
27.TheanswerisE:Psoriasis.Psoriaticlesionstypicallyoccuronextensorsurfacesand
oftenexhibitKoebnerphenomenonandAuspitzsign(slightscratchingofthescalylesions
resultsinpunctatebleedingpoints).Althoughbonyprominencesmaybedamagedduring
trauma(A),adescriptionofasilverscaleonanerythematousbaseisaclassicdescriptionof
psoriasis.Lichensimplexchronicus(B)occursduetohabituallyscratchinganareaofskin.
Theskineventuallythickensanddarkens.Thesechangesarecalledlichenification.Seborrheic
dermatitis(C)causesredorgoldenscalypatchesofskinaroundareaswithoilyskin,suchas
theears,eyebrows,scalp,andnasolabialfold.Atopicdermatitis(D)typicallyaffectsflexor
surfacesandcanleadtolichenification.
28.TheanswerisC:Nofurthertests,thediagnosiscanbemadesolelyonclinical
findings.Thispatienthasthesignsandsymptomsofpsoriaticarthritis,aninflammatory
arthritisthatcharacteristicallyoccursinindividualswithpsoriasis.Adiagnosiscanbemade
basedonclinicalfindings(C)ofpsoriasisandarthropathyalongwithnailchangesranging
frompitting,horizontalridging,onycholysis,yellowishdiscoloration,ordystrophic
hyperkeratosis.Askinbiopsy(A)maybehelpfulifoneisunsureofthepsoriasisonthis
patient'sleg.However,itwouldnothelptocharacterizehisarthropathy.AnMRIofthelumbar
spine(B)mayshownonspecificchangesthatwouldnotbeusefulforadiagnosis.Although
HLAB27(D)isfoundin50%70%ofpatientswithaxialdisease,itwouldnotbeusefultorule
outpsoriaticarthritisinthiscase.Uricacid(E)levelsmaybeelevatedinthepresenceof
extensivepsoriasis.However,thisisanonspecificfindingandwouldnothelpdiagnosisthis
patient'sdisease.

Psoriasis

Psoriasisisacommonchronicskindisordertypicallycharacterizedbyinflamed,edematousskin
lesionscoveredwithasilverywhitescale.Plaquepsoriasisisthemostcommontypeandis
characterizedbysymmetricpatchesonthescalp,trunk,andlimbs.Themostcommonsitesarethe
extensorsurfacesofthelimbs.Guttatepsoriasisischaracterizedbysmallreddotsandfrequently
appearsafteranupperrespiratoryinfection.Nailpsoriasismaycausepitsinthenails,whichmay
becomeyellowandthickened,eventuallyseparatingfromthenailbed.Psoriaticarthritisaffects
approximately10%ofthosewithskinsymptoms.Thearthritisisusuallyinthehands,feet,andlarger
joints.Itproducesstiffness,pain,andprogressivejointdamage.Psoriaticlesionsmaydemonstrate
Auspitzsign(slightscratchingofthescalylesionsrevealingpunctatebleedingpoints)orKoebner
phenomenon(lesionsoccurringinareasofirritation/scratching,suchasthepantline).Thediagnosis
isusuallyclinical.Ifthereisaquestionablediagnosis,apunchbiopsymaybeperformed.Treatment
optionsincludePUVA,topicalretinoids,andcytotoxicagentssuchasmethotrexateandcyclosporine.
Extensorsurfaces
Silverscalesonanerythematousbase
Symmetric
Auspitzsign
Koebnerphenomenon
AbnormalTlymphocytefunction
Notes

29.A65yearoldmanpresentswithathickdomeshapedlesiononhisscalp.Thelesionis
blueblack,7mminsize,andhenotedthatthelesionsometimesbleedswhenheirritatesthe
area.Whatisthemostlikelydiagnosis?
A.Benignnevi
B.Basalcellcarcinoma
C.Squamouscellcarcinoma
D.Melanoma
E.Actinickeratosis
30.A40yearoldmanpresentswithablack,raisedlesiononhisupperback.Thepatient
noticedthatthelesionhasalmostdoubledinsizeoverthepast6months.Onphysicalexamthe
lesionis1cm,nontender,andasymmetricwithirregularborders.Whatisthecorrectwayto
characterizethislesion?
A.Nodularmelanoma
B.Superficialspreadingmelanoma
C.Lentigomaligna
D.Acrallentiginousmelanoma
E.Actinickeratosis
29.TheanswerisD:Melanoma.Thispatienthasfindingsofmelanoma(D)asindicated
bythesize,color,border,andasymmetry.Thelesion'scharacteristicsareconcerningfora
malignantprocessratherthanabenignnevi(A).Basalcellcarcinoma(B)isthemost
commonskincancer.Theyareoftenslowlygrowing,raisedpapularlesionsthatrarely
metastasize.Basalcellcarcinomasmaybetranslucentorpearlywithrolledbordersor
telangiectasia.Squamouscellcarcinoma(C)isthesecondmostcommonskincancerand
usuallyoccursinsunexposedareas.Itischaracterizedbyred,scalyskinthatmayulcerate.
Actinickeratosis(E)isarough,scaly,darkbrownorpinkpatchthatappearsafteryearsofsun
exposure.Asmallnumberofactinickeratoseseventuallydevelopintosquamouscell
carcinomas.

30.TheanswerisA:Nodularmelanoma.Thislesionisanodularmelanoma(A),the
secondmostcommonmelanomasubtype,accountingfor15%30%ofallmelanomas.Nodular
melanomasareremarkableforrapidevolution,oftenarisingoverseveralweekstomonths,and
typicallyappearasuniformlydarkblackbluelesionswithirregularbordersandmeasuring
over6mminsize.Superficialspreadingmelanomas(B)arethemostcommonsubtypeand
accountforaround70%ofallcutaneousmelanomas.Classically,thesesubtypesappearwith
asymmetry,irregularscallopedborders,andoftenvariousshadesofcolor.Theydonotproject
prominentlyfromtheskinandareoftennonpalpable.Lentigomaligna(C)displays
asymmetry,poorlydefinedirregularborders,andisgenerallyaflat,slowlyenlarging,brown,
frecklelikemacule.Acrallentiginousmelanoma(D)onlyconstitutes2%8%ofallmelanomas
inCaucasiansbutupto60%72%inAfricanAmericans.Themostcommonsitesarethesole,
palm,andsubunguallocations,andlesionscanappearbrown,black,tan,orred.Actinic
keratosis(E)occursinfairskinnedolderindividualswithextensivesunexposure.These
lesionsaretypically26mminsizeandappearaserythematous,flat,rough,orscalypapules.

Melanoma

Melanomaisamalignanttumorofmelanocytesfoundpredominantlyintheskin.Itislesscommon
thanmanyothertypesofskincancer,butaccountsfor75%ofskincancerrelateddeaths.Thereare
fourmaintypesofmelanomas.Superficialspreadingmelanomasarethemostcommonandare
usuallyflatandirregularinshapeandcolor.Nodularmelanomasusuallystartasaraisedarea.
Lentigomalignamelanomasusuallyoccurintheelderlyonsundamagedareassuchastheface,
neck,andarms.Acrallentiginousmelanomasusuallyoccuronthepalms,soles,orunderthenails
andaremorecommoninAfricanAmericans.
Treatmentincludessurgicalremovalofthetumor,adjuvanttreatment,chemotherapyand
immunotherapy,orradiationtherapy.
Dermatopathology:hyperplasiaandproliferationofmelanocytesinasinglefilepatterninthe
basalcelllayer
ABCDE
Asymmetricalskinlesion

Borderoflesionisirregular
Color:melanomasusuallyhavemultiplecolors
Diameter:molesgreaterthan6mmaremorelikelytobemelanomas
Enlarging:enlargingorevolving
Notes

31.A43yearoldmanpresentswithawelldemarcated,rough,slightlyraisedlesiononhis
finger.Thelesionhasbeenpresentforabout3months.Whatisthecauseofthislesion?
A.Virus
B.Fungus
C.Bacteria
D.Cellularovergrowth
E.Cellularnecrosis
32.A24yearoldmancomestoyourofficecomplainingofanuncomfortablebumponhis
indexfinger(shownabove).Thislesionhasbeenpresentforthepast2yearsdespitemany
attemptsathomeremoval.Thelesionisrough,slightlyraised,andhasacauliflowerlike
appearanceuponcloserinspection.Whichofthefollowingisthemostappropriatetreatment
forthispatient'scondition?
A.Cryotherapy
B.Surgicalexcision
C.Soaklesioninhotwaterfor1030minutes

D.Carbondioxidelasertherapy
E.Photodynamictherapy
31.TheanswerisA:Virus.Thismanhasalesionconsistentwithaverrucae,orwart.Warts
arecausedbythehumanpapillomavirus(HPV)(A).Theyareclassifiedbasedontheirshape
andtheareaofthebodyaffected.ThereareseveralstrainsofHPV.Verrucaearenotafungal(
B)orbacterial(C)infection.Theyarenotcausedbycellularovergrowth(D)ornecrosis(E
).Commonwartsoftendisappearontheirowninafewmonthsbutmaylastyearsandmayalso
recur.Treatmentoptionsincludecryotherapyorsalicylicacid.
32.TheanswerisA:Cryotherapy.Thelesiononthispatient'sfingerismostlikelya
verrucaeorwartduetoaviralinfectionwithhumanpapillomavirus.Themostappropriate
treatmentlistediscryotherapy(A).Surgicalexcisionisrarelynecessary(B).Soakingthe
lesioninhotwaterfor1030minutesdailyfor6weeksmayaidinresolution,butaonetime
soak(C)isunlikelytobebeneficial.Carbondioxidelasertherapy(D)andphotodynamic
therapy(E)shouldbereservedforlesionsthatfailinitialcryotherapyorsalicylicacid
treatment.

Verrucae

Verrucae,alsocalledwarts,arewelldemarcated,rough,hardnodulesorplaqueswithanirregular
surfacecausedbytheHPV.Theytypicallydisappearafterafewmonthsbutcanlastyearsandcan
recur.Wartsareclassifiedbasedontheirshapeandtheareaofthebodyaffected.Commonwartsare
raisedandroughandaremostcommononthehands.Othertypesofwartsincludeflatwarts,
filiformwarts,plantarwarts,genitalwarts,orperiungualwarts.Cryotherapyorsalicylicacidcanalso
beusedtoremovethelesion.
CausedbyHPV
Treatedwithcryotherapyorsalicylicacid
Notes

33.A27yearoldmanpresentstothecliniccomplainingofaseverelypruriticrashthat
recentlyeruptedonhisskin.Hehasneverexperiencedasimilarrashandappearsanxiousfor
symptomaticrelief.HedeniesanyGIcomplaintsandhasnosignificantpastmedicalhistory.
Physicalexaminationrevealserythematouspatchesandplaqueswithcrustsanderosionsina
symmetricpatternonhiselbows,knees,dorsaofthehands,upperback,andglutealcrease.A
biopsyofoneofthelesionsshowssubepidermalvesiclesatthetipsofdermalpapillaewith
intravesicularneutrophilcollections.Immunofluorescentstainingofabiopsyfromnormal
appearingskinrevealsgranularIgAdeposits.Whichofthefollowingstatementsregardingthis
patient'sdiagnosisis/aretrue?
A.Therashwillrespondrapidlytodapsone.
B.ThispatientisatincreasedriskfordevelopingGIlymphoma.
C.Strictadherencetoaglutenfreediethelpstodecreaseexacerbationsandresolvethe
rash.
D.Alloftheabove
34.A32yearoldwomanpresentstoyourofficewitha1yearhistoryofapruriticrashonher
backandextensorsurfacesofbotharmsandlegs.Herpastmedicalhistoryissignificantfor
irondeficiencyanemiaandchronicsteatorrheacausinga30poundweightlossoverthepast
year.Physicalexamissignificantforpaleconjunctivaeandsmallgroupsofvesicleswith
crustingonerythematousplaquespresentsymmetricallyonherback,buttocks,andextensor
surfaces.AbiopsyfromhealthyskinaroundthelesionsshowsIgAdepositsatthedermal
epidermaljunction,confirmingadiagnosisofdermatitisherpetiformis(DH).Bloodtestsare
positiveforantiendomysialandantitissuetransglutaminaseantibodies,andbiopsyofthesmall
bowelshowsatrophicvilliandintraepitheliallymphocytosis,confirmingassociatedceliac
disease.Inthetreatmentofthispatient,whichofthefollowinghasbeenshowntodecreasethe
symptomsofbothautoimmunedisorderslongterm?
A.Dapsone

B.Prednisone
C.Glutenfreediet
D.Sulphapyridine
E.Tetracycline
33.TheanswerisD:Alloftheabove.ThispatientisexperiencingDH,whichpresentsas
anintenselypruritic,chronicpapulovesiculareruptionsymmetricallyontheextensorsurfaces.
Whilethesepatientshaveaglutensensitiveenteropathy,fewerthan10%aresymptomatic.
Thisenteropathy,however,placesthemathigherriskfordevelopingGIlymphoma.Therash
respondsrapidlytosulfadrugs,especiallydapsone,andtoaglutenfreediet,whichmustbe
maintainedtopreventcutaneousdisease.Thusallansweroptionsaretrue(D).Abiopsyfrom
normalappearingskinthatshowsgranularIgAdepositsisdiagnostic.Dapsoneusemaybe
limitedbydoserelatedhemolyticanemiaandidiopathicneuropathy.
34.TheanswerisC:Glutenfreediet.Thispatientexhibitssymptomsofmalabsorption
(irondeficiencyanemia,steatorrhea,andweightloss).BecauseDHisuniquetogluten
sensitiveenteropathy,celiacdiseaseisthemostlikelyandprovencauseofthismalabsorption.
Intreatingthispatient,sheshouldbetestedandtreatedfordeficienciesoffatsolublevitamins
(A,D,E,K),calcium,folate,iron,andvitaminB12.Sheshouldalsobeevaluatedfor
osteoporosis.Dapsoneandothersulfadrugs(sulphapyridine)areusedforimmediatereliefand
controlofDH.AdditionaltherapiesforDHincludetetracyclineandprednisone.However,
initiationandmaintenanceofaglutenfreedietisthelongtermtherapyforbothDHandceliac
disease(C).Inceliacdisease,symptomreliefmaytakedaystoweeks,andhistologicrecovery
maytakemonthstoyearsonaglutenfreediet.After6monthsofstrictglutenandgliadinfree
diet,sulfadrugsmaybedosedecreasedordiscontinuedintreatmentofDH.

DermatitisHerpetiformis

Dermatitisherpetiformis(DH)isachronic,severelypruritic,papulovesiculareruptionthatusually

followsasymmetricdistributionovertheextensorsurfaces.Histologically,itconsistsofdermal
papillarycollectionsofneutrophils,termedmicroabscesses.AlmostallpatientswithDHhavean
associatedglutensensitiveenteropathy.Itmaypresentatanyageandpersistsindefinitely,with
varyingseverityiftheunderlyingglutensensitivityisnottreated.
Chronic,recurrent,intenselyitchyeruptionsymmetricallyonextensorsurfacesofthe
extremitiesandtrunklesionsmaybevesicles(mostcommon),erythematouspapules,
urticarialikeplaques,orbullae(infrequently)
Histologicallycharacterizedbypapillaryneutrophilcollections
GranularIgAdepositsinparalesionalornormalappearingskinarediagnostic.
Rashrespondsrapidlytodapsoneandadherencetoaglutenfreediet.
Notes

35.An82yearoldmanisbroughtfromanursinghometoyourofficebyhisdaughter,whois
visitingfromoutoftownandbecameconcernedaboutbleedingfromalumponherfather's
nose.Thepatient'smemoryispoor,andhecannotgiveacompletehistory,sayingonly,Iguess
Ijustnevernoticedit.Hehasnocomplaints,andhispastmedicalhistoryissignificantonly
forAlzheimer'sdementia.Physicalexaminationrevealsapearlypapuleonthenosethathasa
waxyappearance,centralcraterwithscab,andvisibletelangiectasia.Multipleactinickeratoses
areseenonthefaceandhands.Whatisthenextappropriatestepinthispatient's
management?
A.Shaveorpunchbiopsy
B.Referralforimmediateirradiation

C.WholebodyPETscanforstagingpurposes
D.Threecyclesofcurettageandelectrodesiccation
E.Topicalchemotherapytoshrinkthelesionbeforesurgicalremoval
36.A75yearoldretiredfishermancomesinforaroutinephysical.Hepointsoutaspotonhis
chestthathasbeenpresentformorethanayear.Hesaysitoccasionallybleedsandisaboutthe
samesizeaswhenhefirstnoticedit.Thelesionhasanulceratedbasecoveredwithacrust.
Biopsyrevealsatypicalbasalcellswithpalisadingnuclei.Whichofthefollowingisthemost
likelydiagnosis?
A.Squamouscellcarcinoma
B.Melanoma
C.Basalcellcarcinoma
D.Seborrheickeratosis
E.Actinickeratosis
35.TheanswerisA:Shaveorpunchbiopsy.Thispatienthasbasalcellcarcinoma.The
nodulartype,hasacharacteristictranslucent,waxy,orpearlyappearance.Ashaveorpunch
biopsyisrequiredtoconfirmthediagnosis(A).Thelesioncanthenbetreatedwithexcision,
curettage,electrodesiccation,orMohsmicrographicsurgery.Curettageandelectrodesiccation
maybecurativebutleaveabroadscartherefore,thesemethodsshouldnotbeusedforbasal
cellcarcinomaontheheadandneck(D).Mohsmicrographicsurgeryistissuesparingandhas
thehighestcurerateamongtreatmentoptions.Irradiationmaybeconsideredforolder
patients(>65years),butrecurrentlesionsareaggressiveanddifficulttotreat.Becauseupto
halfofpatientswilldevelopasecondlesion,theyshouldbemonitoredcloselyfornewor
recurrentlesions.Basalcellcarcinomararelymetastatasizessotreatmentislocalexcision.
ThereforePETscanforstagingpurposes,irradiation,and/orchemotherapy,arenotpartofthe
workupandtreatment(B,C,E).
36.TheanswerisC:Basalcellcarcinoma.Thepatienthasanulceratingbasalcell
carcinoma(C).Itiscommonwiththistypeofbasalcellcarcinomafortheulcertobecovered
withacrust.Aswiththenodulartype,ulceratingbasalcellcarcinomalesionsaretypically
translucentandpearly,withperipheraltelangiectasia.Basalcellcarcinomaisslowgrowing,
withvirtuallynometastaticpotential.Atypicalbasalcellswithpalisadingnucleiarefoundon
biopsy.Squamouscellcarcinoma(A)canalsoulcerateandcrust,butbiopsyshowsatypical
keratinocytesandmalignantepidermalcellspenetratingthebasementmembrane.Melanoma(
B)canalsoulcerate,butthelesioninthiscaseisslowgrowingandshowscharacteristicsof
basalcellcarcinomaonbiopsy.Seborrheickeratosis(D)presentsaswaxybrownpapulesand
plaques,whichhaveastuckonappearance.Actinickeratoses(E)arelight,scaly,

erythematouslesions.Theyareaprecursortosquamouscellcarcinoma.

BasalCellCarcinoma

Basalcellcarcinomaisthemostcommonskincancer.Ithasacharacteristictranslucent,waxy,or
pearlyappearance.Asthelesionenlarges,itoftendevelopsanumbilicatedorulceratedcenterand
peripheraltelangiectasia.
Majorriskfactor:chronicsunlightexposure
Tumorisslowgrowingandlocallyinvasive,aggressive,anddestructive
Metastasisisextremelyrare.
Diagnoseclinicallyconfirmwithbiopsy
Treatwithexcision,curettage,electrodesiccation,orMohsmicrographicsurgery
Notes

37.A17yearoldmanpresentswithlesionsoverhisthighs,buttocks,andarms.Thelesions
appeartohaveasmallraisedcentralwhiteareawithanerythematousbaseandcontain
necroticinflammatorycells.Howwouldyoudescribethispatient'slesions?
A.Pustule
B.Vesicle
C.Bullae
D.Nodule
E.Cyst
38.A4yearoldgirlpresentstoherprimarycarephysicianwithseveralitchylesionsoverher
torso,face,andbuttocks.Theselesionsbegan2daysago.Hermothernotesthatsheisenrolled
indaycarewhereseveralotherchildrenalsoexperiencedsimilarlesions.Onphysicalexamthe
lesionsare35mminsizeandappearascircumscribed,raisedlesionswithacentralwhitish,
pusfilledarea.Howwouldyouclassifythispatient'slesiontype?
A.Papule
B.Pustule
C.Nodule
D.Macule
E.Bullae
37.TheanswerisA:Pustule.Thispatienthaspustules(A)duetofolliculitis.Pustulesare
smallraisedlesionsfilledwithcloudy,purulentmaterial(necroticinflammatorycells).Vesicles
(B)arecircumscribedfluidcontainingepidermalelevationsmeasuringlessthan1cm,
commonlyreferredtoasblisters.Bullae(C)arelargevesiclesandaregreaterthan1cm.
Nodules(D)aregreaterthan1cmandarecenteredinthedermisorsubcutaneousfat.Cysts(
E)arecavitieswithaclosedsacthatcontainaliquid,semisolid,orsolidmaterial.
38.TheanswerisB:Pustule.Thispatienthaspustules(B)likelysecondarytoavaricella
infection.Pustulesaresmallraisedlesionsfilledwithcloudy,purulentmaterial(necrotic
inflammatorycells).Papules(A)aresolid,elevatedlesionsmeasuringlessthan0.5cmin
whichasignificantportionprojectsabovetheplaneofthesurroundingskin.Nodules(C)are
greaterthan1cmandarecenteredinthedermisorsubcutaneousfat.Macules(D)areflat
lesionsoflessthan10mmindiameterthatareevenwiththesurfaceofsurroundingskinand
differincolorfromthesurroundingskinormucousmembrane.Bullae(E)arelargevesicles
greaterthan1cm.

Pustule

Apustuleisasmallelevationoftheskincontainingcloudyorpurulentmaterialusuallyconsistingof
necroticinflammatorycellsorexudate.
Elevated
Containspus(necroticinflammatorycells)
Mayvaryinsize
Mayormaynotberelatedtohairfollicles
Notes

39.A29yearoldAfricanAmericanwomancomplainsaboutagrowthontheposterioraspect
ofherearlobeduringanofficevisit.Itappearedwhereshereceivedasecondearpiercing
duringherpregnancylastyearandcontinuedtogrowevenaftersheremovedtheearring.She
isrequestingthatitberemoved.Whatarehertreatmentoptions?
A.Cryosurgery

B.Excisionatthebase
C.Topicaltriamcinolone
D.Widemarginexcisionwithaskinflap
E.Biopsytoconfirmdiagnosis
40.A25yearoldAfricanAmericanpatientpresentstotheclinicwithmultiplegrowthsthat
appearedonherskinafterlocalinjures:pimples,scratches,cuts,andtattoos.Sheadmitsthat
hermotherandsisterhavesimilargrowths.Sheisanxioustohavethemremoved.Onexam
therearemultipleraisednoduleswithrounded,welldefinedbordersonherrightshoulderand
chest.Uponpalpationtheyareslightlytenderandhavearubberliketexture.Whichofthe
followingis/aretrueregardingherdiagnosis?
A.Excisionhasahighriskofkeloidrecurrence.
B.HispanicandAfricanAmericanpopulationshaveahigherrateofkeloidoccurrence.
C.Injectionsofcorticosteroidsandcryosurgerycandiminishkeloids.
D.Thekeytopreventionisavoidanceofanydisruptionoftheskin.
E.Alloftheabove
39.TheanswerisA:Cryosurgery.Thispatienthaskeloidformationatthesiteofherear
piercing.Itisnotuncommonforkeloidstodevelopduringpregnancy.Keloidsaremore
commonindarkskinnedracesandrepresentexcessiveanddysregulatedcollagendepositionat
thesiteofinjury.Simplesurgicaltreatmentisoftenverydifficultbecausekeloidstendtorecur
andmaygetworse.Injectionsofglucocorticoidshaveshownsomepromiseinshrinkinglesions
andrelievingsymptoms.Cryosurgerywithrepeatedfreezingoverthecourseofamonthhas
producedflatteningofmanylesions,butitisnotalwayssuccessful(A).Biopsyshouldnotbe
performed,becauseitcanexacerbatethekeloidformation.
40.TheanswerisE:Alloftheabove.Keloidsarebenignovergrowthsofconnectivetissue
arisingfromsitesofdermalinjury.Theygrowwellbeyondthedermalbordersofinjury,as
opposedtoahypertrophicscarwhichremainswithinthebordersofnormalskin,andcanbe
expectedtogrowovertime.Becausetheyaregeneratedfromtraumatotheskin,keloid
excisioncanleadtorecurrence.Althoughperiodicinjectionsoftriamcinoloneandcryotherapy
havebothbeenusedtodiminishkeloidsize,riddingapatientofkeloidsisinherentlydifficult.
Preventionisthekeyandeducatingpatientsaboutavoidingcosmeticorelectiveprocedures
suchaspiercings,tattoos,andplasticsurgeryisimportant.Darklypigmentedpopulationshave
ahigherincidenceofkeloids.Keloidpronenesscanalsohavearecessiveordominantpatternof
familialinheritance(E).

Keloid

Akeloidresultsfromtheuncontrolledsynthesisandexcessivedepositionofcollagenatsitesof
previousdermalinjuryandwoundrepair.Theycanoccurafterlocalskintraumaorinflammatory
skinreactions.Akeloidextendsbeyondtheborderoftheoriginalwoundandresemblesawell
circumscribedpinktopurplenoduleorpseudotumor.Itdoesnotregressspontaneously.
Dysregulatedcollagendepositionatsiteofdermalinjury
Tendtorecurafterexcisiondifficulttotreat
Predilectionforareasofincreasedtension
MorecommoninAsiansanddarkskinnedpeoples
Notes

41.A12weekoldinfantisbroughttothepediatricianwithmultipleyellow,greasy,flaky
lesionsoverhisscalp,especiallyalonghishairline.Hismotherhastriedseveralhome
remedies,butnothinghasseemedtohelp.Whattherapywouldyourecommend?

A.Seleniumsulfideshampoo
B.Oralfluconazole
C.Topicalnystatin
D.Topicalketoconazole
E.Moisturizinglotion
42.A43yearoldwomanwithahistoryofAIDSpresentswithdandruffanderythemaand
scalingofhereyebrowsandnasolabialfolds.Yoususpectseborrheicdermatitis.Whichofthe
followingtreatmentsshouldbeavoidedintreatingthefacialscaling?
A.Antidandruffshampoo
B.Salicylicacid
C.Ketoconazoleshampoo
D.Hydrocortisonecream
E.Highpotencytopicalcorticosteroids
41.TheanswerisA:Seleniumsulfideshampoo.Thisinfanthasclassicfindingsof
seborrheicdermatitis,thoughttobecausedbyovergrowthoftheyeast,Malasseziafurfur.
Seborrheicdermatitisonthescalprespondstoshampoos,suchasseleniumsulfide(A).
Topicalantifungals,suchasketoconazole(D),canbetriedwhenthelesionsareinother
sebumrichareas,butshampoosareusedwhenlesionsarepresentonthescalp.Topical
nystatin(C)isoftenusedtotreatcandidainfections.Oralfluconazole(B)andotheroral
medicationscanbetriedforrefractivedisease,butwouldnotbethefirstlinechoice,especially
ina12weekoldinfant.Moisturizinglotion(E)willnotprovideanyrelief,becausethe
affectedareasarenotcausedbybreaksintheskinduetodryness.
42.TheanswerisE:Highpotencytopicalcorticosteroids.Antidandruffshampoo(A
),salicylicacid(B),ketoconazole(C)andhydrocortisonecream(D)areallappropriate
optionsfortreatingseborrheicdermatitis.Theuseofhigherpotencytopicalcorticosteroids(E
)shouldbeavoidedontheface,astheycanleadtosteroidrosaceaorperioraldermatitis.

SeborrheicDermatitis

Seborrheicdermatitis,sometimesreferredtoascradlecapwhenitoccursduringinfancy,isaskin
disorderthatcanaffectthescalp,face,ortrunk.Skinappearsgreasy,yellowred,flaky,andscaly.
Seborrheicdermatitisparticularlyaffectsthesebumglandrichareasofskin.Theyeast,Malassezia
furfur,isinvolved.Temporaryhairlossmaybeasideeffecttotheinflammatoryprocess.Topical
treatmentssuchasshampoosandcreamsareusedintreatment.
Greasy,yellowred,flaky,scaly
Usuallynonpruritic
Affectssebumrichareasofskin
Antifungals,suchasketoconazole,maybehelpfulintreatment.
Outbreaksareusuallyworseinthewinterandcanbeworsenedbystressorfatigue.
Notes

43.A34yearoldmancomestotheclinictofollowuponhisantiretroviraltherapy(ART).Six
monthsago,hisCD4+countwas>500/Landviralloadwas<10,000copiesofhuman
immunodeficiencyvirus(HIV)RNA/mL.Hehasnocomplaints,butanewwhite,corrugated
plaqueisnotedonthelateralaspectofhistongueonphysicalexamination.Itdoesnotscrape
offwithatongueblade.Whatisthenextbeststepinthispatient'streatment?
A.Punchbiopsyandreferraltootolaryngologyororalsurgeryforpossibleresection

B.RepeatCD4+count,viralload,andperformdrugresistancetesting
C.Oralfluconazolefor1014days
D.CessationofART
E.Noactionrequired
44.A35yearoldAfricanAmericanmancomestoclinicforhisyearlyphysical.Thepatienthas
nocomplaints.Hereportshavinghadaflulikeillnessafewmonthsago.Hewascoughingwith
highfeversandchillsinbedformorethan2weeks.Hissocialhistoryissignificantformultiple
maleandfemalesexualpartnerswithintermittentcondomuse.Onphysicalexamthereis
anteriorandposteriorcervicallymphadenopathy,aswellasa1cmwhite,adherent,
corrugated,nontenderpatchontheleftanterolaterallingualmargin.Aclinicaldiagnosisoforal
hairyleukoplakia(OHL)ismade.ThepatientispositiveforHIVoninitialELISAtestand
confirmedwithaWesternblot.Whichofthefollowingis/aretrueregardingOHL?
A.ItrepresentsalocalEpsteinBarrvirus(EBV)infection
B.Seenonlyinimmunocompromisedstates
C.Treatmentisdirectedatunderlyingcondition
D.Itisabenignlesion
E.Alloftheabove
43.TheanswerisB:RepeatCD4+count,viralload,andperformdrugresistance
testing.ThispatienthasOHL,whichisabenignlesionoftheoralmucosarepresentingEBV
infection.Itmostoftenappearsasanasymptomatic,corrugatedwhiteplaqueonthe
inferolateralaspectsofthetongueanddoesnotscrapeoff.Whilethelesionitselfdoesnotneed
treatment,itmayheraldthefailureofARTorthedevelopmentofdrugresistance.Candida
coinfectioncanbepresent,butitwillscrapeoff,asopposedtoOHL.TheappearanceofOHL
carriespoorprognosisinHIVdiseaseanddemandsretestingofCD4+counts,viralload,and
investigationintoresistance(B).
44.TheanswerisE:Alloftheabove.OHLremainsanimportantclinicalsignof
underlyingimmunosuppression(HIV,organtransplant,extendedsteroiduse,chemotherapy).
Asinthiscase,OHLcanbetheheraldofHIVinfection.ThisclinicalpicturepromptedHIV
testing.EventhoughpatientswithOHLandHIVareathigherriskofdevelopingmalignancies
(lymphoma),OHLisitselfabenignlesionthatisrarelysymptomatic.Itappearsonthelateral
bordersofthetongueasafixed,whiteorgray,corrugatedpatchthatmaynotberemovedby
scraping.EBVreplicatesfreelywithinthelesion,wherecandidainfectionmaycoexist.The
lesionwillresolvewithhighlyactiveantiretroviraltherapy(HAART)andsubsequent
restorationofimmunocompetence(E).

HairyLeukoplakia

HairyLeukoplakia

Oralhairyleukoplakia(OHL)isabenignhyperplasiaoftheoralmucosathatmostfrequently
appearsonthelateralorinferioraspectofthetongue.ItrepresentsEBVinfectionandpresentsas
asymptomatic,corrugatedwhiteplaqueswithaccentuationoftheverticalfolds.Itisseenprimarilyin
theHIVpopulation.
EBVinfectionintheHIVpopulationcandidacoinfectionmaybepresent
Notapremalignantcondition,butassociatedwithpoorerprognosisinHIVdisease
LesionregressesorclearswithHAARTdevelopmentmayheraldfailureofHAARTorthe
developmentofdrugresistance
Notes

45.A67yearoldCaucasianmancomestotheofficeforthefirsttime.Herecentlymovedtothe
areaandworksparttimeonafishingboat.Hepreviouslyworkedasalandscaperandtrash

collector.Hehasnocomplaints,althoughhispastmedicalhistoryissignificantforaskin
biopsyonhisshoulderforadarkmole10yearsago.Ashespeaks,hishandsarevisible
(shownabove).Similarlesionsarepresentonhischeeksandtemples.Thelesionsarerough
andscalyandrangefrom15mmindiameter.Whatisthenextbeststepforthelesionsonhis
hand?
A.Reassurancewithnofurtherworkup
B.Biopsy
C.Cryosurgery
D.RecommendUVA/UVBsunscreenwithnofurtherworkup
E.Widelocalexcision
46.A62yearoldfarmerisconcernedaboutaroughpatchonhislip.Ithasbeenthereformore
thanayearandhasnotchangedduringthistime.Itisnotpainful.Hehashadsimilarlesions
onhishandsandthetopofhishead,someofwhichhavebeenfrozenbyhisdermatologist.
Onexamination,thelesionappearswelldemarcatedandhasanulceratedbase.Whatisthe
nextbeststepinmanagement?
A.Reassurancewithnofurtherworkup
B.Biopsy
C.Cryosurgery
D.RecommendUVA/UVBsunscreenwithnofurtherworkup
E.Widelocalexcision
45.TheanswerisC:Cryosurgery.Thepatient'shistoryofsunexposure,combinedwith
theappearanceanddescription(roughandscaly)ofthelesions,makesactinickeratosisthe
likelydiagnosis.Sinceactinickeratosiscanprogresstosquamouscellcarcinoma,thelesions
shouldbetreated.Cryosurgeryworksinmostcases(C).Actinickeratosiscanusuallybe
diagnosedclinicallyhowever,highlyhyperkeratoticlesions,aswellasthosethatarepainful,
bleeding,ulcerated,orrapidlygrowing,shouldbebiopsiedtoruleoutmalignancy(B).
Sunscreenisanimportantpreventivemeasure(D),buttreatmentiswarrantedatthistime.
Widelocalexcisionisthetreatmentifsquamouscellcarcinomaisdiagnosed(E).
46.TheanswerisB:Biopsy.Thelesionmostlikelyrepresentsactinickeratosis.Itisawell
demarcated,stablelesioninanexposedareaonalightskinnedmanwhohasworked
extensivelyoutdoors.However,actinickeratosiscanprogresstosquamouscellcarcinoma,and
ulcerationisaconcerningfeature.Thusbiopsyiswarranted(B).Hishistoryissuggestiveof
cryosurgery(C)forotherlesions,andthismaybeanappropriatetreatmentinthiscaseifthe
biopsyisnegative.Sunscreen(D)shouldberecommendedasapreventivemeasurebutdoes

notresolvethequestionofhowtomanagethecurrentlesion.Becausethisislikelyactinic
keratosis,widelocalexcision(E)isnotappropriateatthistime.

ActinicKeratosis

Actinickeratosisconsistsofrough,adherent,scalylesionsthatareoftenerythematousandpainful
whenscratched.Theyresultfromproliferationofabnormalepidermalkeratinocytesinresponseto
prolongedUVradiationexposure.
Precursorofsquamouscellcarcinoma
Themostimportantriskfactorisexposuretosunlight.Lookforanolder,lightskinned
individualwhohasspentalotoftimeoutdoors.
Diagnoseclinically.Highlyhyperkeratoticlesionsmayneedbiopsytoruleoutsquamouscell
carcinoma.
Itshouldbetreatedtopreventprogressiontosquamouscellcarcinoma.Optionsinclude
cryosurgery,topicalagents,curettage,andphototherapy.
Notes

47.A63yearoldAfricanAmericanwomanpresentswithalargeviolaceousplaqueoverthe
bridgeofhernose,hercheeks,andherupperlip.Shealsocomplainsofcoughanddyspnea.A
chestxrayrevealsbilateralhilaradenopathy.Whichofthefollowingfindingswouldyou
expect?
A.Biopsyoftheskinlesionshowingnoncaseatinggranuloma
B.Biopsyofthelungshowingcaseatinggranuloma
C.Lowserumangiotensinconvertingenzyme(ACE)level
D.Pulmonaryfunctiontest(PFT)showingobstructivepattern
E.Positiverapidplasmareagin(RPR)
48.A34yearoldAfricanAmericanmanpresentswithworseningshortnessofbreath,chest
pain,andcoughof3months'duration.Healsocomplainsoffatigueandgeneralmalaise,
describinga10poundweightlossoverthelast3months.Thisisthefirsttimehehas
experiencedsymptomslikethis,andhedeniesanyhistoryoftravelorincarceration.Heworks
asalibrariananddeniesalcohol,drug,ortobaccouse.Physicalexamrevealspatchycrackles.
Chestxrayshowsmediastinaladenopathy,andbiopsyofoneofthelesionsreveals
noncaseatinggranulomas.HisHIVtestisnegative.Whichofthefollowingisthemost
appropriateinitialtreatmentforthispatient?
A.INHandrifampin
B.Chemotherapy
C.Cyclophosphamide
D.Glucocorticoids
E.Infliximab
47.TheanswerisA:Biopsyoftheskinlesionshowingnoncaseatinggranuloma.
Theviolaceousplaqueoverthiswoman'sfaceislupuspernio,whichisacutaneous
manifestationofsarcoidosis.Thiswomanalsohasbilateralhilaradenopathy,aclassicfinding
insarcoidosis,whichindicateslunginvolvement.Abiopsyofthelungorskinlesionwould
shownoncaseatinggranulomas(A).Caseatinggranulomasonlungbiopsy(B)aresuggestive
oftuberculosis(TB).Upto75%ofuntreatedsarcoidpatientswillhaveanelevatedACElevel,
thusalowserumACElevel(C)wouldnotaidinmakingadiagnosis.Whenlunginvolvement
ispresentinsarcoidosis,arestrictivepatternisseenonPFTsratherthananobstructivepattern
(D).RPR(E)isascreeningtestforsyphilis.

48.TheanswerisD:Glucocorticoids.Thispatientissufferingfromsarcoidosis,a
multisystemdiseasethatmostfrequentlyinvolvesthelungs.Othermanifestationsinclude
erythemanodosum,acutepolyarthritis,andanterioruveitis.Mediastinalandparatracheal
adenopathyarehighlysuggestiveofsarcoidosis.AdditionalclassicfindingsonchestXray
includereticulonodularinfiltratesandbiopsywilldemonstratenoncaseatinggranulomas.
Asymptomaticsarcoidosisisoftenfollowedcloselywithouttreatmentduetothehighrateof
spontaneousremission,butpatientswithsymptomaticdiseasereceivesystemicglucocorticoids
(D)asinitialtreatmentforsarcoidosis.INHandrifampin(A)arepartofthetypicalfour
drugregimenusedtotreatactiveTB.WhileTBcouldexplainmanyofthispatient'ssymptoms,
biopsywouldshowcaseatinggranulomas.Cyclophosphamide(C)isachemotherapeuticagent
thatisoftenusedtotreatsystemiclupuserythematosuswithrenalinvolvementandisnotan
appropriateinitialtreatmentforsarcoidosis.Infliximabisatumornecrosisalphainhibitor
(TNFalphainhibitor)approvedfortheuseinankylosingspondylitis,inflammatorybowel
disease,andrheumatoidarthritis.Neitherinfliximabnorchemotherapy(B)wouldbean
appropriatetreatmentforthispatient.

Sarcoidosis(NoncaseatingGranulomas)

Sarcoidosisisasystemicgranulomatousinflammatorydiseaseofunknownetiology.Thediseaseis
characterizedbynoncaseatinggranulomas.Granulomasaremostoftennotedinthelungsorlymph
nodes,butalmostanyorgancanbeaffected.ThehighestincidenceisinNorthernAmericanblacks
andNorthernEuropeanwhites.Amongblacks,womenaremorefrequentlyaffectedthanmen.
Diseaseonsetisusuallyinthe30sor40s.Lupuspernioisaseverecutaneousmanifestationof
sarcoidosis.Violaceousplaquescanoccuronthenose,cheeks,ears,lips,andfingers.Diagnosisof
sarcoidosisrequiresthreecomponents:clinicalandradiographicmanifestations,exclusionofother
diseaseswithsimilarpresentations,andthepresenceofnoncaseatinggranulomas.Chestxray,
pulmonaryfunctiontests,ECG,slitlampeyeexam,liverfunctiontests,andserumcalciumtest
shouldbeincludedintheinitialevaluationofapatientwithsuspectedsarcoidosis.Glucocorticoids
aretheinitialtreatment.
Bilateralhilaradenopathymaybeseenonimaging.
Treatwithcorticosteroids.Otherimmunosuppressivedrugs,cyclosporine,andantiTNF

therapywithinfliximabhavealsobeenused.
Pulmonaryfunctiontestscharacteristicallyrevealarestrictivepatternifthereislung
involvement.
ElevatedserumACElevelin75%ofuntreatedpatients
Histopathologicdetectionofnoncaseatinggranulomaishelpfultomakediagnosis.
Commonpresentingsymptomsincludecough,dyspnea,chestpain,eyelesions,and/orskin
lesions.
Notes

49.A23yearoldmanpresentswithpruriticlesionsonhisrightarmwhichappearedabout24
hoursagooverhisupperarm.Thelesionsappearascentralroundedlocalizedareasofedema
withperipheralerythema.Howwouldyoudefinethecentralareaofedema?
A.Flare
B.Wheal
C.Vesicle
D.Bulla
E.Cyst

50.A45yearoldwomancomestoyourofficecomplainingofaraised,red,itchylesionwitha
centralareaofedemaandperipheralerythemathatappearedafewhoursafterbeingbittenby
amosquito.Shehasaformaleventtomorrowandisconcernedabouthowlongthisunsightly
markwillremainonherarm.Whatisthebestadviceforthispatient?
A.Thislesionwilllikelyfadein7days.
B.Thislesionwilllikelyprogresstoadisseminatedrashin37days.
C.Thislesionwilllikelygoawayin2448hours.
D.Corticosteroidsarenecessaryforthetreatmentofthislesion.
E.Biopsyiswarrantedfordefinitivediagnosis.
49.TheanswerisB:Wheal.Thispatienthasurticaria(hives),whichtypicallylast2448
hours.Thelesionsarepruriticandclassicallydemonstrateacentralwheal(B).Thewhealis
theareaoflocalizededema.Surroundingthewheal,aflatareaoferythemaisoftenpresent,
referredtoastheflare(A).Althoughthelesionissimilartoavesicle(C),asmall,fluidfilled,
elevatedlesion,theevanescentnatureofthelesionandtheassociatedpruriticdescription
indicateurticaria.Abulla(D)isalargevesicle.Acyst(E)isanepitheliallinedsaccontaining
fluidorsemisolidmaterial.
50.TheanswerisC:Thislesionwilllikelygoawayin2448hours.Thispatientis
experiencinganallergicreactionfromtheinsectbiteknownasawhealandflarereactionthat
willlikelyresolvein2448hourswithoutanytreatment(C).Corticosteroidsarenotnecessary
forthetreatmentofthislesion(D).Clinicaldiagnosissufficesinthiscase,andbiopsyisnot
warrantedunlessthelesionpersists(E).

Wheal

Awhealisaprimaryskinlesionandisdefinedasaroundedorflattopped,paleredpapuleorplaque
thatistypicallyevanescent,disappearingwithin2448hours.Awhealappearsasanareaoflocalized
edemathatfollowsvascularleakage.Awhealandflareskinreactionisoftenassociatedwithan

allergicreaction.Thewhealisthecentralraisedarea,whichissurroundedbyaflaterythematous
flare.
Typicallylast2448hours
Primaryskinlesion
Localizededema
Notes

51.A56yearoldwomancomestotheofficeforapostoperativevisit1weekafteropenfixation
ofananklefractureandcomplainsofpainandswellinginherfoot(picturedabove).Shestates
thatshefirstnoticedapatchofrednessandsomepainnearthesurgicalincisionlastnightbut
wokeuptofindithadspreadtoencompassherentirefoot.Shereportsthatshejustdoesn't
feelwell,andreportschillsovernight.Onphysicalexam,hertemperatureis101F(38.3C),
bloodpressure118/78,andheartrate86beatsperminute(bpm).Therightfootisedematous,
erythematous,warm,andtendertothetouchwithnocrepitusorfluctuancepresent.The
incisionsiteisintact,andnopurulentmaterialisnoted.Whichofthefollowinginfectious
organismsshouldbesuspectedinthispatient?
A.GroupAhemolyticstreptococci
B.Pseudomonasaeruginosa
C.Staphylococcusaureus
D.Escherichiacoliandothergramnegativespecies

E.AllexceptB
52.A77yearoldCaucasianwomanwasrecentlybittenonthehandbyherhousecat.Thecat
hasbeenvaccinatedagainstrabiesandneverleavesthehouse.Thewomanhasahistoryof
hypertensionandhyperlipidemiabutisotherwisehealthy.Herhandiserythematousand
edematous.Strengthinherfingersandwristis5/5.Anxrayshowednotoothfragmentsor
bonyinjury.Culturesarepending.SheisgiventetanustoxoidintheED.Whichofthefollowing
isthebesttreatment?
A.Amoxicillinclavulanate
B.Cefepime
C.Ciprofloxacin
D.Clindamycin
E.Notreatmentnecessary
51.TheanswerisE:AllexceptB.Thispatienthascellulitisthatshedevelopedaftera
surgicalprocedure.Thedefectintheskinbarrierandanymetalhardwareinstalledduringthe
procedurearerisksfordevelopingthistypeofinfection.Shehastheclassicfindingsofedema,
erythema,warmth,andpain.Thelackofcrepitusandfluctuanceisreassuringthatthis
infectionisnotnecrotizingfasciitisoranabscess.Commonpathogensforcellulitisinclude
groupAhemolyticstreptococciandS.aureusspecieshowever,thispatientisalsoatriskfor
E.coliandgramnegativeinfectionbecausethewoundisbelowthewaist.Thusinitialantibiotic
coverageshouldhaveactivityagainstthesepathogensandcanbeadjustedlaterifthe
pathogen(s)canbeidentified(E).
52.TheanswerisA:Amoxicillinclavulanate.Thepatienthascellulitisfromacatbite
andislikelyinfectedwithPasteurellamultocida.Thebestantibioticchoiceisoral
amoxicillin/clavulanicacidfor710days(A).Alternativesincludeoraldoxycyclineor
clindamycinplusafluoroquinolone.Clindamycinaloneandcephalosporinsdonotprovide
coverageagainstP.multicida(B,C,D).Catbitesoftenpenetratetotendonsandbones.The
woundshouldbeexaminedfordepthofinjuryandpossiblejointpenetration.Nerveand
tendonfunctionandintegrityofnearbyvesselsshouldbedetermined,andanxrayshouldbe
obtainedtoexcluderetainedtoothfragments,fracture,and/orcrushinjury,whichmayoccurin
thesettingofananimalbite.

Cellulitis

Cellulitisisanacuteinflammatoryreactionrepresentingaspreadinginfectionofthedermisand
subcutaneoustissue.ThemostcommonpathogensincludegroupAhemolyticstreptococciandS.
aureus.Ittypicallypresentsasasmallpatchofswelling,erythema,andpainbeforespreading
diffusely.Astheinfectionspreads,thepatientmayexperiencefever,chills,andmalaise,andisatrisk
fordevelopingsepticemia.
Edematous,expanding,erythematous,warmplaquewithilldefinedborders
Pathogensgainentrytodermisandsubcutaneoustissuethroughdefectsintheskinbarrier,
includingcuts,cracks,abrasions,burns,insectbites,surgicalincisions,orforeignbodiessuch
asIVcatheters.
Biopsyandaspirationoftheleadingedgehavelowyieldinidentifyingthepathogen.Blood
culturesorculturefromanulcer,pustule,orabscess,ifpresent,maybeuseful.
Notes

53.A35yearoldmannoticesaslowgrowingmassonhisarm.Themassismildlytender,and
heisconcernedbecausethemassislargeandnoticeabletoothers.Physicalexamination
revealsarubbery,soft,fluctuantmasswithlobulationandfreemobilityoftheoverlyingskin.
Whatisthemostlikelydiagnosis?
A.Lymphoma

B.Liposarcoma
C.Lipoma
D.Glomustumor
E.Cutaneousleiomyoma
54.A50yearoldmanpresentstohisprimarycarephysicianwithalargemassonhisupper
back.Themassispainlessand,asidefrombeinguncomfortableduetoitssize,hasnot
otherwiseaffectedthepatient.Heis,however,concernedthathemighthavecancerbecause
themasshasbeenslowlygrowingoverthepast5years.Onphysicalexamthemassis
superficial,nontender,homogeneous,andrubbery.Althoughthephysicianfeelsthemassis
highlylikelytobebenign,abiopsyisconductedtobettercharacterizethetissueandassurethe
patient.Whatisthemostlikelyhistologicalfinding?
A.Amixtureofmatureadipocytesandachondroidmatrixcontaininglipoblasts.
B.Unencapsulatedbundlesofpleomorphicspindlecellsandmultinucleategiantcells
mixedwithlipoblasts.
C.Largenumbersofbrownfatcellsmixedwithmaturewhitefat.
D.Lobulesofmaturewhiteadiposetissuedividedbyfibrousseptacontainingthin
walled,capillarysizedvessels.
E.Aproliferationofadiposeandfibroustissuewithintheepineuriuminfiltratingalong
andbetweennervebundles.
53.TheanswerisC:Lipoma.Superficialsubcutaneouslipomasarecommon,benign
neoplasmsoftheskin.Theyrarelycausesymptoms,butpatientsoftenbringthemtothe
attentionoftheirproviderseitherbecausetheyareeitherworriedaboutthepossibilityof
cancerorbecauseofthecosmeticdeformitycausedbylargerlesions.Clinicalhistoryisusedto
distinguishlipomafrommoreseriousconditionssuchaslymphoma(A)basedonthefactthat
lipomasaresoft,asymptomatic,andusuallydonotenlargequickly.Malignanttransformation
ofalipomaintoaliposarcoma(B)israre.Liposarcomastypicallyappearinthedeepfattissue
ofthethighorabdomeninpeoplebetweentheagesof50and70.Ifasuspectedlipomacauses
symptoms,israpidlyenlarging,orisfirmratherthansoft,abiopsyisindicated.Glomustumors
(D)aresmallbenigntumorsthatarisefromtheglomusbody.Cutaneousleiomyomas(E)are
benignsofttissueneoplasmsthatarisefromthesmoothmuscleandmayinvolvevascular
smoothmuscle,thedartosmusclesofthegenitalia,thenipples,ortheareolas.
54.TheanswerisD:Lobulesofmaturewhiteadiposetissuedividedbyfibrous
septacontainingthinwalled,capillarysizedvessels.Thispatientispresentingwitha
lipoma(D),themostcommonsofttissueneoplasm.Thesetumorsoccurpredominantlyin
individualsbetween40and60yearsold,butmayaffectpatientsinawideagerange.Lipomas

areentirelybenignmesenchymalneoplasms.Themostcommonpresentationisasapainless
subcutaneousmasstypicallyoccurringonthetrunk,neck,orextremities.Lesscommonly,
lipomasmaybefounddeepinthesubcutaneoustissueorintramuscularlyandmaybepainful
topatients.Histologicallythesetumorsarecircumscribed,encapsulatedbyathin,fibrous
capsule,andcomposedofmaturewhiteadipocyteslackingevidenceofnuclearatypia.Because
lipomasareentirelybenign,theymaybeleftalonewithoutconcernformalignantpotential
unlesstheycausediscomforttoapatient.Intheinstanceofverylargetumorsorintramuscular
lipomas,surgicalexcisionisthetreatmentofchoice.(A)Chondroidlipomasarerarebenign
tumorsoccurringmostcommonlyinadultwomenandtendtobesmallerthan5cm.Theyare
characterizedhistologicallybyanadmixtureofamatureadipocyticcomponentanda
chondroidmatrixcontaininglipoblasts.(B)Pleomorphicliposarcomasareaggressive
sarcomasfoundontheextremitiesofadultsandareassociatedwithhighratesofmetastasis
andmortality.Thesedeepseatedtumorstendtogrowrapidlyandarelessfrequentlyfoundin
subcutaneoustissue.Histologicallytheyarecharacterizedbyfeaturesofahighgrade
pleomorphicsarcomacontainingspindlecellsandmultinucleatedgiantcellswithadmixed
lipoblasts.(C)Hibernomasarerarebenignneoplasmscontainingbrownfat.Theymost
commonlyaffectyoungadultsandaremostfrequentlyfoundinthethighregion.(E)
Lipomatosisofnerveisararetumorthataffectsinfantsandchildrenandpresentsasagrowing
mass,mostcommonlyonthehand.Histologically,lesionsarecharacterizedbyaproliferation
ofadiposeandfibroustissuewithintheepineurium,infiltratingalongandinbetweennerve
bundles.

Lipoma

Alipomaisabenigntumorofmaturefatcellsenclosedbyathin,fibrouscapsule.Theyaremost
oftenfoundsuperficiallyinthesubcutaneoustissuebutcanmorerarelyinvolvefasciaordeeper
muscularplanes.
Familialmultiplelipomatosisisageneticconditionresultinginmultiplelipomas.

Diagnosticworkup:CTorultrasound(notMRI),excision,biopsy
Surgicalremovalofthefatcellsandfibrouscapsuleisindicatedforcosmeticreasonsifthe
lipomacausespain,orfortissuetoconfirmdiagnosis.
Notes

55.Apreviouslyhealthy64yearoldmanpresentswithlocalizedpain,erythema,andswelling
surroundingasmallwoundonhisrighthand.Thesesymptomsrapidlyprogressedtoinvolve
hisentirerightarm,andhispainbecamesevere.Healsodevelopedfeveranddiarrhea.Initial
laboratoryevaluationrevealsanelevatedWBCandanelevatedcreatinephosphokinase.In
additiontoempiricantibiotics,whatwouldbethenextstepintreatment?
A.Reexaminepatientin24hours
B.Ultrasoundofrightarm
C.SendpatientforimmediateMRI
D.Culturewoundonpatient'srighthand
E.Surgicalexplorationanddebridement
56.A61yearoldmanwithapastmedicalhistoryoftype2diabetesmellitus,IVdruguse,and
chronicskininfectionspresentstotheEDwitha3dayhistoryofsubjectivefeverandconfusion
alongwithswelling,redness,andblistersonhisrightarm.Thepatientadmitsthathehadbeen
skinpoppingIVheroininhisrightarmintheweekpriortotheonsetofhissymptoms.
Physicalexamrevealserythema,brawnyedema,andseverepainwithveryminimalpassive
movementoftherightextremity.Thereareseverallargebullaeseenonhisrightforearm,along
withcrepitusintheoverlyingskin.Whatismostimportantfirststepinthemanagementofthis
patient'scondition?
A.MRIoftherightupperextremity
B.Surgicalconsult

C.Transfusionoftwounitsofpackedredbloodcells
D.Lanceofthebullaeandculturefluid
E.Hyperbaricoxygentherapy
55.TheanswerisE:Surgicalexplorationanddebridement.Thispatientispresenting
withfindingsthatareconcerningfornecrotizingfasciitis.Thisisasurgicalemergencyand
requiresexplorationanddebridement(E).Surgicalexplorationistheonlywaytoconfirmthe
diagnosis,andpromptsurgicalexplorationfacilitatesearlydebridement.BroadspectrumIV
antibioticsshouldbeadministered.Reexaminingthepatientin24hours(A)isnot
appropriate,giventhelikelihoodofrapidprogression.Althoughnecrotizingfasciitismustbe
distinguishedfromotherprocesses,suchasgasgangrene,pyomyositis,andmyositis,the
historyandclinicalfeaturesusuallyallowthisdistinction.Laboratoryfindingsarenotspecific.
Imagingmaybeusefulinsomeinstances.CTismostcommonlyusedtovisualizeairalongthe
fascialplanes.MRIandultrasound(B,C)maynotdelineateairalongthefascialplanes.MRI
oftencannotdistinguishnecrotizingfasciitisfromcellulitisorinflammation.Gramstainand
cultureofthelesion(D)maybepositivefororganisms,butthisisnotasreliableasobtaining
deepsamplesduringsurgicalexploration.Ifthediagnosisofnecrotizingfasciitisissuspected,
awaitingtheresultsofbloodorskinculturesisnotrecommended.
56.TheanswerisB:Surgicalconsult.Thispatientpresentswiththeclassicsymptomsofa
necrotizingsofttissueinfection,whichincludetissuepainoutofproportiontophysical
examinationfindings,erythema,brawnyedema,andcrepitus.Bullaedeveloplaterinthe
courseoftheinfection.Thispatienthasseveralriskfactorsforanecrotizingsofttissue
infection,includingdiabetesmellitus,advancedage,chronicskininfections,andIVdrugabuse.
Severalotherriskfactorsarealcoholism,peripheralvasculardisease,heartdisease,renal
failure,HIV,cancer,NSAIDuse,decubitusulcers,andimmunesystemimpairment.
Necrotizingfasciitisisaclinicaldiagnosisbasedonphysicalexamfindingsandpatienthistory.
Thispatient'spastmedicalhistory,especiallyhisIVdruguse,makethediagnosishighlylikely.
Necrotizingsofttissueinfectionsareasurgicalemergency,andearlysurgicalconsultation(B)
isthemostimportantfirststepinthemanagementofallsuspectedcasesofnecrotizing
fasciitis,andthegoldstandardfordiagnosisandtherapyremainsoperativeexplorationand
surgicaldebridement.MRI(A)ishighlysensitiveinidentifyingfascialthickeninganddeep
tissuecollectionsbutdelaysdefinitivesurgicaltreatment.Bloodtransfusion(C)maybe
indicatedlatertocorrectanemiafromhemolysisbutisnotthefirststepinmanagement.
Althoughculturesofbloodandbullaefluid(D)maybehelpfullaterinthetreatmentwhen
antibioticcoverageisnarrowed,itismoreimportanttogetthepatientintotheoperatingroom
forexplorationanddebridement.Hyperbaricoxygentherapy(E)isacontroversialtherapy
thatwouldbethedecisionofthesurgicalconsultandwouldnotbepartoftheinitialtreatment
ofapatientintheED.

NecrotizingFasciitis

Necrotizingfasciitisisaninfectionofthedeeplayersofskinandsubcutaneoustissuethateasily
spreadsacrossthefascialplanes.Necrotizingfasciitismaypresentinitiallywithlocalizedpain,
erythema,andswelling.However,rapidprogressionmayfollow,resultinginbullaeformationand
systemicsymptoms.Ifuntreated,thebacteriacantravelrapidlyalongthefasciaofthemuscleand
spreadthroughoutthebody,leadingtosepsisanddeath.Therearetwotypesofnecrotizingfasciitis.
Type1isamixedorganismalinfectionthatoccurspostsurgically,ofteninpatientswithdiabetesor
peripheralvasculardisease.Type2isamonomicrobialinfectioncausedprincipallybygroupA
streptococcus(Streptococcuspyogenes).
Ifthereisclinicalsuspicionofnecrotizingfasciitis,surgicalexplorationshouldbedonefor
definitivediagnosis.Nolabtestscanrapidlyandaccuratelydifferentiatebetweennecrotizing
fasciitisandothersofttissueinfections.
Treatwithmultipleepisodesofsurgicaldebridement,hemodynamicsupport,andempiricIV
antibiotics.
StreptococcuspyogenesreleasesanexotoxinknownassuperantigenwhichactivatesTcells
andleadstotheoverproductionofcytokines.
Notes

57.A36yearoldmancomestothecliniccomplainingofweakness.Hestateshehasrecently
noticedalackofenergyandfrequentlyexperiencesbackachesandheadaches.Heisalso
concernedaboutrecentchangesinhisappearance.Examinationrevealsplethoricmoonfacies,
supraclavicularfatpads,andaprotuberantabdomen.Hedoesnotsmokeordrinkalcoholand
takesnomedications.Pastmedicalhistoryissignificantonlyforcellulitis2monthsprior.
Whichsequenceoftestswouldconfirmthediagnosis?
A.Serumadrenocorticotropichormone(ACTH)levelandMRIofthehead
B.Dexamethasonesuppressiontest,urinefreecortisol,andserumACTH
C.CTscanofthechestandabdomen
D.Midnightserumcortisol,morningserumcortisol,andrandomserumcortisoltests
E.Nofurthertestingnecessary
58.A22yearoldwomancomestotheclinicbecausesheisconcernedaboutchangesinher
appearance,aswellasdecreasedlibidoandirregularmenses,whichshereportshavebeen
worseningoverthepastseveralyears.Shefeelsthatherfacehasgottenlarger,andsheno
longerfeelsattractive.Shealsoreportsthatherskinhasbecomedarkspotted,andshehas
noticedpurplelinesonherabdomen.Inthelastfewmonths,shehasgivenuponphysical
activitybecauseshefeelsweak.Examinationrevealsplethoricmoonfacies,supraclavicularfat
pads,andcentralobesity.Herbloodpressureis150/90.Shedoesnotsmokeordrinkalcohol.
Shetakesnomedications.Pastmedicalhistoryissignificantonlyforrecurrentinfectionsin
recentyears.Whichofthefollowingdescribechangesseeninthemostlikelydiagnosisofthis
patient?
A.IncreaseinserumACTHlevelproducedbytheanteriorpituitary
B.IncreaseincortisollevelanddecreaseofACTHlevel
C.IncreaseinACTHlevelproducedectopically
D.Decreaseincortisollevelduetoadrenalinsufficiency
E.Electrolytedisturbances,normalACTHlevels
57.TheanswerisB:Dexamethasonesuppressiontest,urinefreecortisol,and
serumACTH.Thispatientdisplaysthephysicalandsymptomaticmanifestationsof
hypercortisolisminCushingsyndrome,andhisconditionwarrantsaworkup(E).Thecorrect
sequenceofconfirmatorylaboratorytestingisadexamethasonesuppressiontest,urinefree
cortisol,andserumACTH(B).Dexamethasonesuppressiontestingisperformedbygiving1

mgofdexamethasoneorallyat11pmandassessingserumcortisolat8amthefollowing
morning.Anabnormaltestresultwarrantsconfirmationofexcesscortisolby24hoururine
freecortisoltesting,midnightcortisollevels,oralatenightsalivarycortisolassay(D).Once
hypercortisolismisestablished,serumACTHlevelsshouldbeassessed.AlevelofACTHbelow
20pg/mLindicatesaprobableadrenaltumor,whereashigherlevelsaretheresultofapituitary
tumor(termedCushingdisease)orectopicACTHproduction.Imagingtechniquesfor
localizationofthelesionareindicatedoncetheinitialworkuphasbeencompleted(A,C).
58.TheanswerisA:IncreaseinserumACTHlevelproducedbytheanterior
pituitary.ThepatientsuffersfromCushingsyndrome(A),duetoincreasedproductionof
ACTH.Shehastypicalsignsofthisdisorder,includingroundplethoricfacies,purpleskin
striae,andcentralobesity.Shealsohassymptomsofweakness,menstrualirregularities,and
sexualdysfunction.CortisolincreasessecondarytoanincreaseinACTH,ratherthanadecrease
(B).Thepatientisnotasmoker,sotheincreasedinACTHisnotlikelyduetoectopic
productionfromamalignanttumor,suchassmallcellcarcinomaofthelung(C).Addison
diseasecausesanincreaseinACTH(becauseoflackofnegativefeedback).Typicallyitpresents
asanincreaseinskinpigmentationbutisnotassociatedwiththeothersymptomsseeninthis
case(D).Connsyndromeresultsinelectrolytedisturbancesandmuscleweakness,butthe
othersymptomsinthiscasearenotexplainedbythisdisease(E).

CushingSyndrome

Cushingsyndromereferstothestateofexcesscorticosteroidsandtheresultantphysical
manifestations.Themostfrequentcauseisiatrogenic.Cushingsyndromeislessfrequentlydueto
endogenousexcesscorticosteroidproductionbytheadrenalcortex.Cushingdiseaseaccountsfor
nearly40%ofcasesanddescribesthescenariowherethemanifestationsofexcesscorticosteroids
resultfromthehypersecretionofACTHbyabenignpituitaryadenoma,mostfrequentlylocatedin
theanteriorpituitary.Occasionally,hypercortisolismmayrepresentaparaneoplasticcondition
wherenonpituitaryneoplasmslikesmallcelllungcancerproduceandsecreteACTH.Excess
corticosteroidproductionindependentofACTHisduetoautonomoussecretionofcortisolbythe
adrenals,usuallyfromaunilateraladrenaltumor.
Centralobesity,moonfacies,protuberantabdomen,musclewasting,thinskin,hirsutism,

purplestriae,buffalohump,supraclavicularfatpads
Osteoporosis,hypertension,poorwoundhealing,easybruising,superficialskininfections,
andsusceptibilitytoopportunisticinfections
Women:oligomenorrheaoramenorrheamales:impotence
Psychologicchanges:decreasedconcentration,moodliability,frankpsychosis
Hyperglycemia,glycosuria,leukocytosis,lymphocytopenia,hypokalemia
Elevatedserumcortisolandurinefreecortisollackofnormalsuppressionbydexamethasone
Notes

59.A41yearoldmalepatienthasbeenonlongtermhighdosesteroidsforCrohndisease.He
isconcernedbecausehehasnoticedanincreasingnumberofwelldefined,symmetric,
elevated,reddishpurplelesionsinhisaxilla.Healsohasseveralolderlesionswhichhavefaded
toapalecolor.Whichofthefollowingoptionsmayhelptoimprovetheappearanceofthese
lesions?
A.Topicaltretinoin
B.Glycolicacidcream
C.Stophighdosesteroidtreatment
D.Lasersurgery
E.Alloftheabove

Notes

60.A28yearoldwomanwithahistoryofsevereasthmapresentstoyourofficecomplainingof
recentweightgain.Sheisfrustratedaftertryingmanydietsandcomestoyouforadvice.Upon
furtherquestioning,thepatientalsodescribesstretchmarksonherabdomenandfattygrowth
onherneckandshoulders.Younoticethatherarmsandlegsarestillquitethinandherface
appearsmoonlike.Sheseemstobeconcealingherarmpitswhichyounoticetobequitesweaty.
Whichofthefollowingisthebeststepindiagnosinghercondition?
A.Glucosetolerancetest
B.MRIofthepituitarygland
C.Fastingplasmaglucoselevel
D.CToftheadrenalglands
E.Dexamethasonesuppressiontest
59.TheanswerisE:Alloftheabove.Alloftheabovetreatments(E)areknowntohelp
improvetheprogressionofstriae(stretchmarks).Althoughthelesionsarenotreversible,there
aretreatmentsthatcanhelptofadethelesions.Topicaltretinoin(A),glycolicacidcreams(B
),andstoppingsteroidtreatment(C)willhelptofadestriae.Inaddition,lasersurgery(D)
hasbeenshowninsomecasestoimprovetheappearanceofstretchmarks.
60.TheanswerisE.ThemostcommoncauseofCushingsyndromeistheexogenous
administrationofcorticosteroidsbyahealthcareprovider,inthiscasetotreatthispatient's
severeasthma.Thepatient'ssymptoms,centralobesity,striae,andhyperhidrosis,inthesetting
oflikelysteroiduse,suggestexogenousCushingsyndrome.WhenCushingsyndromeis

suspected,eitheradexamethasonesuppressiontestora24hoururinecortisolmeasurementis
indicatedasthenextstepindiagnosis.Ifthistestispositive,anMRIofthepituitaryglandor
CToftheadrenalglandmaybeperformedtodetectthepresenceofanyadrenalorpituitary
adenomasorincidentalomas(B,D).Whilethispatientlikelyhasimpairedglucosetolerance
duetoCushingsyndrome,aglucosetolerancetest(A)orfastingplasmaglucoselevel(C)is
notnecessaryatthistime.

Striae

Striae,orstretchmarks,arewelldefined,symmetric,trophiclesions.Theyarecausedbyatearingof
thedermisandareaformofscarringthatproducesanoffcolorhue.Theyfirstappearaselevated
reddishorpurplelinesinareasofrapidgrowthhowever,theyeventuallyflattenandfadetoapale
color.Striaeareoftentheresultofrapidskinstretching.Theymaybeassociatedwithtimesofrapid
growthsuchaspuberty,weightgain,andpregnancy.Inaddition,striaeformationisinfluencedby
hormonalchanges.Glucocorticoidhormonescausestriaeformationbypreventingfibroblastsfrom
formingcollagenandelastinfibersnecessarytokeeprapidlygrowingskintaut.Thusstriaemay
occurinpatientswithCushingsyndromeorinpatientschronicallytreatedwithhighdosesteroids.
Theycanoccuranywhereonthebodybutaremostlikelytoappearinareasoflargerfatstores
(abdomen,breasts,upperarms,back,thighs,hips,buttocks).Striaearemorecommoninwomen.
Earlystagestriaehaveshownimprovementwiththeuseoftopicaltretinoinandglycolicacidcreams.
Lasertherapyhasbeenshowninsomecasestoimprovetheappearanceofstriaeaswell.
Tearingofthedermis
Appearinareaswithlargerfatstores
Associatedwithtimesofrapidgrowth
Influencedbyhormonalchanges
Notes

61.A27yearoldwomanpresentstothecliniccomplainingofpaininherlowerlegsforthe
past2days.Overthistimeshehasbeenfeelingillandalsocomplainsofbilateralanklepain.
Onphysicalexam,hertemperatureis100.4F(38C),andherbloodpressureis142/92.
Erythematouslesionsthatarenodularandtendertopalpationareseenonherlowerlegs
(shownabove).ESRandcreactiveproteinareelevated.Achestradiographrevealsbilateral
adenopathy,andsarcoidosisissuspected.Thepatientisscheduledfortissuebiopsies.Whatis
thebestmanagementofherlowerlimbfindingsatthistime?
A.Topicalcorticosteroids
B.Oralprednisonewithataper
C.NSAIDsandsymptomatictreatment
D.Bilateralankleaspirationandradiographs
E.Biopsyoftheedgeofanodule
62.A23yearoldwomenpresentstotheclinicwithpleuriticchestpain,bronchialcough,
fatigue,andafeverfor2weeks.ShesaysthatshewenthikingandcampinginsouthernArizona
1monthago.Shethoughtshehadpickedupabadcold.Herfeverresolved3daysago
however,herfatigue,cough,andchestpaincontinue.Shewaspromptedtocomeinyesterday
whenshenoticedpainfulrednodulesonherlowerlegsandachingkneepain.Physicalexamis
significantforrightsidedcracklesand2to3cmhard,red,tendernodulesontheextensor
surfacesofthelowerlegs.Achestxrayshowsarightmiddlelobeinfiltrateandrighthilar
adenopathy.Sputumculturesaresent.Tissuesamplesobtainedviabronchoscopyreveal
spherulesfilledwithendospores.Whatisthebestmanagementatthistime?
A.Oralcorticosteroids
B.Ampicillinclavulanate

C.NSAIDsandsymptomatictreatment
D.Fluconazole
E.CandDonly
61.TheanswerisC:NSAIDsandsymptomatictreatment.Thispatienthaserythema
nodosumandsuspectedsarcoidosis.Erythemanodosumisanimmunologicskinreaction
showinggranulomatousinflammationthathasavarietyofcauses,includingdrugs,infection,
inflammatoryorgranulomatousdisease,andmalignancy.Treatmentissymptomatic,
antiinflammatory(C),anddirectedattheunderlyingcauseifknown.Patientsoftenhave
associatedfever,malaise,andarthralgia,particularlyoftheanklejoints.Glucocorticoidsshould
beusedonlyiftheetiologyisknown.Womenareaffected10timesmorefrequentlythanmen.
Lesionswilltypicallydisappearafter6weeksbutmayrecur.
62.TheanswerisE:CandDonly.Thispatienthasconfirmedcoccidioidomycosis
pneumoniawitherythemanodosum.Closeto70%ofallcasesoferythemanodosumhavean
identifiedcause(infection,drugs,systemicillness,pregnancy).Commoncausesinclude
streptococcalinfection,sarcoidosis,lymphoma,andinflammatoryboweldisease.NSAIDsare
helpfulintreatingtheinflammationoferythemanodosum.However,onceanunderlyingcause
forerythemanodosumhasbeenidentified,treatmentofthatcause,inthiscasefluconazolefor
fungalpneumonia,isindicated(E).

ErythemaNodosum

Erythemanodosumisanacuteinflammatory,immunologicreactiontothesubcutaneousfat
characterizedbytheappearanceofpainfulnodulesinthelowerlegs.Itrepresentsareactionpattern
toavarietyofetiologicagents,includingdrugs,infections,andotherinflammatoryorimmunologic
diseases.Lesionstendtobebrightred,nodular,andtendertopalpation,andareoftenaccompanied
byfeverandmalaise.
Multipleanddiverseetiologies:drugs,infections,inflammatoryorgranulomatousdisease
Painful,tenderlesionsonlowerlegsforafewdays'durationaccompaniedbyfever,malaise,

andarthralgia(commonlyankles)
Managementissymptomatic(bedrest,compressionbandages)ordirectedattheetiology,
whenidentified(antiinflammatorydrugs,glucocorticoids).
Womenareaffectedmorethanmen(10:1ratio).
Notes

63.A25yearoldmanwithahistoryofmitralvalveprolapse(MVP)presentswith1dayof
sharp,tearingchestpainatthecenterofhischestthatnowradiatestowardhisneck.Physical
examrevealslowbloodpressure,moderatesternaldepression,andatall,thinmanwithlong
arms.Whatislikelytobeabnormalinthispatient?
A.Collagen
B.Fibrillin1
C.LDLcholesterol
D.HDLcholesterol
E.Urinehomocystinelevel
Notes

64.A16yearoldmanpresentstohisprimarycarephysicianwithacomplaintofnear
sightedness.Hereportsthathehasrecentlyhaddifficultyreadingsignsanddiscerningfaces
atadistance.Thepatientisaverytall,thinmanwithlongarmsandfingers.Physicalexam
revealsmoderatesternaldepressionandstriaedistensaeontheshoulderandaxillaryarea.On
visualacuityexamthepatientisfoundtohave20/100ODand20/80OSvision.What
additionalscreeningtestwillnotbenecessaryinthispatient?
A.Annualophthalmicexam
B.Urinehomocystinelevel
C.Annualechocardiograms
D.Annualchestxray
E.Annualmusculoskeletalevaluationforkyphoscoliosis
63.TheanswerisB:Fibrillin1.Thispatientishavinganaorticdissectionandhasclassic
findingsofMarfansyndrome,includingMVP,longarms,andpectusexcavatum.Marfan
syndromeiscausedbyadefectinfibrillin1(B).Collagendefects(A)areassociatedwith
EhlersDanlossyndrome,inwhichpatientsexhibitskinhyperextensibilityandjoint
hypermobility.LowLDLandhighHDLcholesterol(CandD)wouldputapatientatriskfor
anacutecoronarysyndrome.Urinehomocystineabnormalities(E)canbeseenin
homocystinuria,arecessivelyinheritedmetabolicdisorderthatcanleadtoectopialentis,
mentalretardation,boneovergrowth,andahighpredispositiontothromboembolismand
coronaryarterydiseaseintheabsenceofaorticrootdilation.
64.TheanswerisD:Annualchestxray.Thispatientexhibitsmanyoftheclassicfeatures
ofMarfansyndrome,includingmyopia,longlimbs,arachnodactyly,striaedistensae,and
pectusexcavatum.Marfansyndromeisageneralizeddisorderofconnectivetissuecausedbya
defectinfibrillin1thathasprimarymanifestationsintheskeletal,ocular,andcardiovascular
systems.Asaresult,Marfanpatientscommonlysufferfromdisordersinthesesystemsand
mustbecloselyscreened.Annualophthalmicexams(A),includingslitlampexamination,are
necessarytoscreenforectopialentis,anupwarddisplacementofthelensaffectingupto70%of
patientswithMarfansyndrome,whichmayleadtodislocationandacuteglaucoma.Urine
homocystinelevels(B)shouldbecheckedinallpatientssuspectedofhavingMarfan

syndrometoruleouthomocystinuria,arecessivelyinheritedmetabolicdisorderthatcanlead
tosimilarclinicalmanifestationsasMarfansyndrome,includingectopialentis.Annual
echocardiograms(C)areindicatedinallpatientswithMarfansyndrometoscreenforMVP
anddilationoftheproximalaorta,whichcanleadtoaorticdissection.Annualmusculoskeletal
evaluationforkyphoscoliosis(E)isparticularlyimportantinyoungerpatientswhoarestill
growing,asseverekyphoscoliosisisfrequentlyseeninMarfansyndrome.Anannualchestxray
(D)isnotarecommendedscreeningtestforpatientswithMarfansyndrome.

MarfanSyndrome

Marfansyndromeisanautosomaldominantdisorderofconnectivetissueprotein,fibrillin1.
Abnormalconnectivetissuedevelopmentmayresultincardiovascular,musculoskeletal,skin,lung,
andCNSabnormalities.Musculoskeletalfeaturesinclude:longlimbs,arachnodactyly,pectus
excavatumorcarinatum,scoliosis,higharchedpalate,highpedalarches,andjointhypermobility.
CardiovascularfeaturesincludeMVP,aorticrootdilation,aorticdissection,andrupture.Ocular
featuresincludesuperiorlensdislocationandretinaldetachment.AdiagnosisofMarfansyndrome
shouldalsobeconsideredinpatientswithrecurrentspontaneouspneumothoracessecondaryto
ruptureofblebs.
FBN1gene
Fibrillin1protein

Affectsabout1in5000peopleintheUnitedStates
Treatwithblockerstodecreasemyocardialcontractilityandslowrateofaorticdissection
Routineechocardiogrammonitoringtoevaluateaorticrootdiameter
Notes

65.A25yearoldCaucasianmotherbringsher4weekoldgirltotheoffice.Sheexpresses
concernthatherdaughterappearsdifferentthansheandthefather.Thebaby'sskinissnow
white,asarehereyebrowsandscalphair.Sheappearswellandexhibitsnormaldevelopment.
Herpaleblueiridesaretranslucent,andthecorneallightreflexisasymmetric.Nystagmusis
present.Whichadviceshouldbegiventothemotherregardingherdaughter'scondition?
A.Sheislikelytohavelowerthanaverageintelligenceandbehavioralproblems.
B.Obtainaperipheralbloodsmear.
C.Preventivemeasurestominimizetheriskofsundamageshouldbeinitiatedwhenshe
reachespuberty.
D.Sheshouldbeunderthecareofanophthalmologistanddermatologistinadditionto
herprimarycarephysician.
E.Sheshouldhaveemergentgenetictesting.
66.A10montholdCaucasianboyhasrecurrentskinandlunginfections.Hisskinandhairare

white,andhisfacehasapinkhue.Hismothersayshe'sthefirstalbinointhefamily.Arecent
workuprevealedneutropeniaandneutrophilswithlargegrayishbluegranules.Whichofthe
followingisthemostappropriatetreatment?
A.Regulartransfusions
B.Splenectomy
C.Bonemarrowtransplant
D.IVimmunoglobulin
E.Reassurance
65.TheanswerisD:Sheshouldbeunderthecareofanophthalmologistand
dermatologistinadditiontoherprimarycarephysician.Theinfanthas
oculocutaneousalbinism(OCA),evidentbyherwhiteskinandhair,aswellasiristranslucency,
nystagmus,andotherocularabnormalities.PeoplewithOCAhaveanincreasedriskofskin
cancer,mostcommonlysquamouscellcarcinoma,andvisionproblems.Thereforetheyshould
befollowedbyadermatologistandophthalmologist(D).OCAresultsfromadefectinmelanin
synthesis,causingobviousskinandhairmanifestations,aswellaslessobviousbut
pathognomonictranslucentiridesandnystagmus.Iristranslucencyisbestseeninadarkroom
withaflashlightshinedatthesclera.Preventivemeasurestodecreasesunexposureshouldbe
initiatedatbirth(C).OCAisnotassociatedwithcognitiveorbehavioralproblems(A).
Albinismmayresultfromanumberofgeneticmutations,whichcanbeidentifiedwithgenetic
testinghowever,thisisunlikelytoaffectmanagement(E).Inaddition,therearespecialtypes
ofalbinismassociatedwithhematologicabnormalities,suchasHermanskyPudlaksyndrome
andChdiakHigashisyndrome,butthepatientinthiscasedoesnotappeartohaveoneof
thesesyndromes(B).
66.TheanswerisC:Bonemarrowtransplant.ChdiakHigashisyndromeisarare
autosomalrecessivedisorderthatpresentswithoculocutaneousalbinismandneutropenia.
Giantgrayishbluegranulesareseeninneutrophils.Thepreferredtreatmentisbonemarrow
transplant(C).PatientswithChdiakHigashihaverecurrentpyogenicinfections.Common
organismsincludeS.aureus,S.pyogenes,andPseudomonas.PatientswithChdiakHigashi
syndromemayhavemildbleeding,butregulartransfusionsarenotindicated(A).IV
immunoglobulinisappropriateforsomeantibodyrelatedimmunodeficiencies(D).Thereis
noroleforsplenectomyinthemanagementofChdiakHigashisyndrome(B).Chdiak
Higashisyndromeisaseriousconditionthatwarrantsaggressivemanagementasitoften
resultsindeathbeforetheageof7ifuntreated(E).

Albinism

Albinismismostcommonlyduetoautosomalrecessivemutationsinmelaninsynthesis.Itispresent
atbirthandtypicallyaffectstheeyesandskin(OCA).InOCA,thereisanormalnumberof
melanocytes,butakeyenzymeinmelaninsynthesis,tyrosinase,isinactive.
Typicalfeatures:whiteskinandhair,includingeyelashesandeyebrowstranslucentirides
andnystagmusarepathognomonic
Nystagmusisduetofovealhypoplasiawithdecreasedvisualacuityandalteredopticnerve
formation.
Somefairskinnedpeoplemayappeartohavealbinism,butabsenceofiristranslucencyand
nystagmusdifferentiatesthemfromindividualswiththedisorder.
Earlydiagnosisofalbinismisessentialtominimizingtheincidenceofskincancersothat
patientscantakepreventivemeasurestolimitsunexposure.
Notes

67.A16yearoldmanpresentswithcomplaintsofarash.Duringthephysicalexamination,you
notemultipletiny,symmetricpapulesandplaques,mostprominentoverthetrunk.Asingle
largerlesionispresentoverthepatient'sside.Thislesionissalmoncoloredwithacentralscale
locatedwithintheovalplaque.Whatisthebesttreatmentforthisrash?
A.Notreatmentisneeded.Itshouldresolvein68weeks.
B.Topicalantifungaltreatment
C.Oralantifungaltreatment
D.Topicalnystatin
E.Topicalhydrocortisonecream
68.A23yearoldwomanwithnopastmedicalhistorypresentscomplainingofa1week
historyofanitchyrashonherbackandabdomen.Shenotesthattherashinitiallybeganasa
singlelargerpatchonherabdomenoneweekago.Shehasnotrecentlystartedanynew
medicationsorchangedherdetergentsorsoaps.Onphysicalexamthepatienthasseveral
erythematous,fine,scalingplaquesandpatchesrangingfrom13cmwithfinecollarettescales
attheperipheryofeachlesion.Whatisthemostlikelydiagnosisofthispatient'srash?
A.Secondarysyphilis
B.Guttatepsoriasis
C.Erythemamultiforme(EM)
D.Pityriasisrosea
E.Tineacorporis
67.TheanswerisA:Notreatmentisneeded.Itshouldresolvein68weeks.This
patienthasclassicfindingsofpityriasisrosea,includingaheraldpatchandarashmost
prominentoverthetrunk.Becausethisisthoughttobeduetoareactiontohumanherpesvirus
(HHV)infection,theviralexanthemshouldresolvespontaneouslyin68weeks(A).Herald
patchesmaybeconfusedwithringwormhowever,thescaleofaheraldpatchisclassically
locatedinsidethepatch.Inringworm,whichiscausedbytinea,thescaleislocatedoutsidethe
patch.Ringwormisduetoafungalinfectionandismostoftentreatedwithtopicalantifungals(
B).Insomecases,suchastineacapitis,anoralantifungaltreatment(C)maybeneeded.
Topicalnystatin(D)isoftenusedtotreatcandidainfections.Topicalhydrocortisonecream(
E)isnotneededtotreatthisviralexanthem.
68.TheanswerisD:Pityriasisrosea.Thispatientpresentswiththecharacteristic

symptomsofpityriasisrosea(D),includinganinitialheraldpatch(a2to5cm,salmonred
patchthatisthepresentingsymptomin80%ofpatients)withascalelocatedinsidethepatch.
Thisisincontrasttotineacorporis(E)thatpresentswithscalinglesionswithscaleslocated
outsidethepatch.Secondarysyphilispresentsasapapulosquamoustruncaleruptionthat
involvesthepalmsandsoles,whichisnotseeninthispatient.Guttatepsoriasis(B)is
characterizedbyeruptionofsmall(0.51.5cm)papulesovertheuppertrunkandproximal
extremities.Theselesionsaremuchsmallerthanthoseinpityriasisanddonotpresentwitha
heraldpatch.EM(C)presentsastargetlesionsthatmaycoalesceanddeveloperosionsover
thebodyinresponsetoherpessimplexvirus(HSV)andcertaindrugs.

PityriasisRosea

Pityriasisroseaisanacuteviralexanthemandmanifestswithacharacteristicskinrash.Therash
beginswithaheraldpatch,asinglescalysalmoncolored2to5cmovallesion.After12weeks,a
secondaryeruptionofsmallerfinescalingpapulesandplaqueswithcollarettesevolves.The
secondaryexanthemisdistributedsymmetricallyinaChristmastreelikepatternmostprominent
overthetrunk,neck,andadjacentextremities.Thelongaxisoftheseovallesionstendstobeoriented
withthecleavagelinesoftheskin.Ofnote,theinitialheraldpatchmaybeconfusedwithtinea.
Pityriasisrosea'sscaleislocatedwithintheovalplaque,whereastinea'sscaleislocatedattheborder
oftheovalplaque.Pityriasisroseaistypicallyselflimitingandresolveswithin68weeks.Treatment
isusuallynotneeded,buttopicalzincoxide,calaminelotion,ormoderatepotencycorticosteroids
canbeusedforpruritis.
Heraldpatch
Christmastreelikerash
Scalelocatedwithinovalplaque
Selflimiting
ThoughttobeareactiontoHHV7
Notes

69.A19yearoldmanpresentswithedematous,pruriticpapulesonhisabdomenandchest.
Thelesionsappearedshortlyaftereatingashellfishmealatarestaurant.Healsocomplainsof
difficultybreathingandswallowingandhaswheezingonexamination.Whatisthemost
appropriatetreatmentforthispatient?
A.Benadryl
B.Topicalglucocorticoids
C.Systemicglucocorticoids
D.Intramuscular(IM)epinephrine
E.Ranitidine
70.A23yearoldwomancomestotheurgentcarecentercomplainingofarasharoundher
neck.ForValentine'sDay,herfiancgaveherabeautifulgoldnecklace.Sincewearingthis
necklace,shehasdevelopedaskinrashnotableforpalered,raised,itchypapulesandvesicles
aroundherneck.Yoususpectthatshehasdevelopedcontactdermatitisfromhernecklace.
Whichofthefollowingisthemostlikelymechanismofherrashdevelopment?
A.Type4delayedcellmediatedhypersensitivity
B.Immunecomplexdisease
C.Cytotoxic,antibodydependentresponse
D.Type1hypersensitivityreaction
E.Releaseofhistaminefromeosinophils

69.TheanswerisD:IMepinephrine.Thispatientishavingasevereallergicreactionto
shellfish.Hehasurticariaaswellassignsofangioedemaandairwayconstriction.Thusthis
patientshouldbegivenIMepinephrine(D).Benadryl(A)isanH1blockerandranitidine(E
)isanH2blocker.Althoughtheyarebothantihistamines,epinephrineshouldbeusedasfirst
linetreatmentiftherearesignsofairwaycompromise.Systemicglucocorticoids(C)canbe
usedtoalleviateurticaria,butwouldnotbefirstlineinthissituation.Topicalglucocorticoids(
B)mayhelptoalleviatetheitchingassociatedwithurticariaandmayhelpthelesionsresolve,
butthispatientisshowingconcerningsignsofsystemicinflammation.Thispatientshouldbe
instructedtoavoidshellfishandshouldcarryanEpiPeninthefuture.
70.TheanswerisA:Type4delayedcellmediatedhypersensitivity.Thiswomanis
experiencinganallergiccontactdermatitis,atype4hypersensitivityreaction(A).Hercollar
likerashisconfinedtotheareaofallergenexposure,inthiscasenickelsulfate.Immune
complexdisease(B)isatype3hypersensitivityreaction,characteristicofanarthusreaction
orserumsickness.Cytotoxic,antibodydependentreactions(C)aretype2hypersensitivity
reactions.Examplesincludeautoimmunehemolyticanemiaanderythroblastosisfetalis.Type1
hypersensitivityreaction(D)ischaracteristicofanaphylaxisandasthma.Histamineisnot
releasedfromeosinophils(E).

Urticaria

Urticaria,commonlycalledhives,areduetoeitheranallergicandnonallergicstimulithatcausesthe
releaseofinflammatorymediators,includinghistamineandmastcells.Urticariacanresultfrom
contactdermatitis,food,drugs,recentupperrespiratoryinfections,andenvironmentalfactors.Key
featuresincludeevanescentwheals(raisedareassurroundedbyaredbase)orhives.Itchingis
usuallyintense.Mostincidencesareacuteandselflimited.Lesionstypicallylastlessthan24hours.
Chronicurticariaisdefinedasrecurrentepisodeslastinglongerthan6weeks.Urticariamaybe
associatedwithangioedema,flushing,burning,wheezing,andabdominalpain.Antihistaminesand
prednisonearethemainstaysoftreatment.Ifangioedemaandwheezingareprominent,IM
epinephrinemayalsobeused.
Histamineandmastcells

Mostcommoncauses:food,drugs,contactdermatitis
Chronicurticaria>6weeks
Notes

71.A41yearoldwomanpresentswithpainfululcerationsofherdistalfingers.Onphysical
examinationshealsohasnailfoldcapillarychangesandtightthickenedskinoverherhands,
causinglossofmobilityofherfingers.Whichofthefollowingwouldhelptoconfirmthe
diagnosis?
A.Anticentromereantibodies
B.AntidoublestrandedDNA(antidsDNA)antibodies
C.AntiSmith(antiSm)antibodies
D.Antineutrophilcytoplasmicantibody(ANCA)levels
E.Biopsyoftheulcerations
72.A52yearoldmanpresentswithachangeinhisfacialappearance.Hehasnoticed
tighteningoftheskinaroundhismouthandeyebrows.Inaddition,hisfingershavedeveloped
asausagelikeappearance.TwoyearsagohehadbeendiagnosedwithRaynaudphenomenon
(RP).Herecallshismothersufferingfromaconditionseverelyaffectingthemobilityofher
fingers.Whichofthefollowingisthemostaccurateregardingthispatient'slikelydiagnosis?
A.Womenarefourtimesaslikelyasmentodevelopthedisease.
B.PositiveantidsDNAandantiSmantibodiesconfirmthecondition.

C.Theconditioncanbecuredwithagentsusedtoincreasebloodflowtotheaffected
areas.
D.RPisaresultofchronicdiseaseprogression.
E.Thisfamilialconditioncanbemappedtotheshortarmofchromosome3.
71.TheanswerisD:ANCAlevels.Thispatienthasdigitalulcerationduetovasculitis
causedbyRP.Inthiscase,theRPislikelysecondarytosclerodermagiventhephysicalexam
findingsofnailfoldcapillarychangesandsclerodactyly.ANA(antinuclearantibodies),antiScl,
andanticentromereantibodies(A)arehelpfulindiagnosingscleroderma.Anticentromere
antibodyisspecificforCREST,alimitedformofscleroderma.AntidsDNAantibodies(B)and
antiSm(C)antibodiesarepositiveinlupus.LupuspatientsmayalsoexperienceRP,butthe
physicalexamfindingsinthispatientareconsistentwithscleroderma,notlupus.ANCAlevels(
D)canbepositiveincertainconditionscausingvasculitis.Cytoplasmicantineutrophil
cytoplasmicantibody(CANCA)ispositiveinconditionssuchasWegenersyndrome.
Perinuclearantineutrophilcytoplasmicantibodies(PANCA)maybepositiveinChurgStrauss
syndrome,primarysclerosingcholangitis,andpolyarteritisnodosa.Abiopsy(E)ofthe
ulcerationwillshownecrosisbutwouldnotbehelpfulindeterminingacause.
72.TheanswerisA:Womenarefourtimesaslikelyasmentodevelopthedisease.
Thispatienthasscleroderma.Sclerodermaisfourtimesaslikelyinwomencomparedtomen(
A).RPisanearlymanifestationofdisease,andmanypatientswillpresentwithRaynaud(D).
PositiveantidsDNAandantiSmantibodiessuggestlupus(B).Sclerodermaisachronic
autoimmuneconditionwithoutacure.Treatmentisaimedatalleviatingsymptoms(C).The
causeofsclerodermaisunknown,andwhilesymptomsseemtoruninfamilies,aspecificgene
hasnotyetbeenidentified.

Scleroderma(Hands)

Sclerodermaisachronicautoimmunediseasecharacterizedbyfibrosis,vascularalterations,and
autoantibodies.Limitedsclerodermamainlyaffectsthehands,arms,andface.Facialinvolvement
mayincludeasmooth,unwrinkledbrow,tightareaaroundthenose,andashrunken,expressionless

mouth.Involvedskinfeelssmoothandinduratedandasifitisboundtounderlyingstructures.Areas
ofhyperpigmentationorhypopigmentationmaybepresent.CRESTsyndromeisanexampleof
limitedscleroderma,whichresultsinCalcinosis,Raynaudphenomenon,Esophagealdysfunction,S
clerodactyly(fibrosedskincausesimmobiledigits),andTelangiectasias.Pulmonaryarterial
hypertensionisthemostseriouscomplicationofthisformofscleroderma.Limitedsclerodermacan
alsocausevascularchangesintheextremities,suchasthehands,whichmayleadtolocalulcerations
anddigitischemia.Diffusesclerodermaisrapidlyprogressingandaffectslargeareasofskinandat
leastoneinternalorganfrequentlythekidneys,esophagus,heart,andlungs.Fibrosisand
tighteningoftheskinmayresultinflexioncontractures.
NinetyfivepercentofcaseshavepositiveANA.AntiSclhaslowersensitivity,buthigher
specificity.
Anticentromereis80%sensitiveforCRESTsyndrome.
Treatment:immunosuppressiveagents,suchasmethotrexate
Handfindingsinscleroderma:Raynaud,sclerodactyly,nailfoldcapillarychanges
RPisseeninapproximately90%ofpatients.
Womenareaffectedthreetofourtimesmorefrequentlythanmen.
Notes

73.Ahealthy58yearoldmancomestotheclinicbecauseofarashoverhisneckandshoulder.
Hestatesthathestartedhavingaburningpainsensationintheareaonedaypriortotherash
appearing.Hehasbeenexperiencingmanylifechangesrecently,ashiscompanyclosedandhe
hasbeenoutofworkfor1month.Onphysicalexamination,therashislocatedontheright

aspectoftheneck,overtherightclavicleandrightdeltoidareaintheC3C4dermatomes.What
measure(s)canbetakentodecreasehisriskofpostherpeticneuralgia?
A.Earlyinstitutionofantiinflammatorymedications
B.Earlyantiviraltherapyincludingvalacyclovir1gbid
C.Administerzostervaccinenow
D.Alloftheabove
74.A23yearoldfemalegraduatestudentpresentstotheofficethedayafterdefendingher
thesis.Shecomplainsthat2daysagoshefeltatingling,pulsing,rightsidedheadache.It
graduallyturnedintoanintenseburningpainoverherrightforehead,andthenshenoticedred
bumpsoverthatarea.Onexam,vesiclesofvaryingsizeareseeneruptingfroman
erythematousbaseinthesuperiordermatomeoftherightcranialnerve(CN)V1withminimal
periorbitalinvolvement.Therashextendsovertherightforeheadandscalptothevertex,
overlappingthemidlinebyonly1cm.Thispatientisdiagnosedclinicallywithherpeszoster
ophthalmicus.Whichofthefollowingis/aretrueregardingthiscondition?
A.Earlyantiviraltherapycanreduceextentoferuptionandsequelae.
B.Highriskforcornealinvolvement,lidinvolvement,anduveitis
C.Associatedwithaprodromeofheadaches,nausea,andvomiting
D.Alloftheabove
73.TheanswerisB:Earlyantiviraltherapyincludingvalacyclovir1gbid.This
patientissufferingfromherpeszosterintheC3andC4dermatomes.Thisrashrepresents
reactivationoflatentvaricellazostervirusfromthespinalganglia.Itreachestheskinthrough
sensorynervesandthereforepresentsinadermatomalpatternoferythematousmaculopapular
clustersofvesicles.Animportantpotentialconsequenceispostherpeticneuralgia,which
representschronicpainintheaffectednervesduetoinflammatorydamageduringreactivation.
Limitingthedurationandspreadofthereactionareimportantintheacutephase,and
administrationofthezostervaccinetoappropriatepatientscanreducetheincidenceof
postherpeticneuralgiabytwothirds(E).
74.TheanswerisD:Alloftheabove.Thispatienthasherpeszosterophthalmicusalong
theCNV1dermatome(rightophthalmicbranchofthetrigeminalnerve)mostlikelybroughton
bystress.Shehadaprodromalheadache.Otheraspectsoftheprodromeincludenausea,
vomiting,andpreauricular/submaxillarylymphadenopathy.Althoughsheisatriskforocular
involvement,thisherpeszostereruptionisprimarilyinthefrontalbranchofCNV1.The
highestriskforseriousocularcomplicationsiswitheruptionalongthenasociliarybranchof
CNV1,whichinnervatesthetipandsideofthenoseandtheeye(Hutchinsonsign).Early
antiviraltherapy(valacyclovir,acyclovir,famciclovir)candecreaseeruption,ocular

involvement,andpostherpeticneuralgia(D).

HerpesZoster

Herpeszosteriscausedbythevaricellazostervirusthatcauseschickenpox.Herpeszosteris
classicallycharacterizedbyunilateral,dermatomalpainandrashasaresultofreactivationofthe
latentvaricellazostervirusinthespinalganglia.Erythematous,vesicular,maculopapularlesions
appearinclustersinadermatomaldistribution.Painisthemostimportantclinicalmanifestation,
andthepatientmayexperiencechronicpainfrompostherpeticneuralgiaafterresolutionoftherash.
Avaricellazostervaccinereducestheincidenceofherpeszosterbyonehalfandtheincidenceof
postherpeticneuralgiabytwothirds.
Latentvaricellazostervirusreactivatingandtravelingtoskinindermatomalpatternvia
sensorynerves
Elderlyandimmunosuppressedindividualsaremostcommonlyaffected.
Antiviraltherapyandanalgesiaarehelpfulintheacutesetting.
Goalsoftherapyarelimitationofpainandrashdurationacutely,limitingdiseasespread,and
preventingpostherpeticneuralgia.
Postherpeticneuralgiatreatmentcanincludelidocainepatches,oralgabapentin,andtricyclic
antidepressants.
Notes

75.A54yearoldwomanwithafamilyhistoryofosteoporosisanda40packyearsmoking
historyuntil2yearsagowasrecentlystartedonacombinedestrogenandprogesterone
regimentopreventosteoporosis.Shenowreturnstothecliniccomplainingofnewonsetdark
patchesofskinonherface.Shenotesthatshehasrecentlybeenspendingseveralhoursinthe
suneverydaytendinghergarden.Onphysicalexam,thepatientexhibitssymmetricmacular
hyperpigmentationonhercheeksandforehead.Whatisthemostlikelydiagnosisofthis
patient'shyperpigmentation?
A.Postinflammatoryhypermelanoticmacules
B.Systemiclupuserythematosus(SLE)
C.Melasma
D.Vitiligo
E.Hypersensitivityreactiontocombinedhormonereplacementtherapy(HRT)
76.A31yearoldG1P0womanpresentswithcomplaintsofdarkpatchesofskinonherface.
Thisisthefirsttimeshehasexperiencedtheselesions,andshenotesthatherskinhasalways
beenhealthy.Thelesionshaveappearedgradually,buttheyhavebecomemorevisibleinthe
lastmonth.Younotehyperpigmentedpatchesoverhercheeks,upperlip,andforehead.What
advicewouldyougivethispatient?
A.Begintopicaltherapywithhydrocortisonecream
B.Begintopicaltherapywithprogesteronecream
C.Referhertodermatologistforabiopsy
D.Recommendthatsheavoidsexposuretosunlight
E.Reassureherthatthelesionsarebenignandarecommonlyassociatedwithaging
75.TheanswerisC:Melasma.Thispatientismostlikelyexperiencingmelasma(C),an
acquiredlightordarkbrownhyperpigmentationresultingfromexposuretosunlight
combinedwiththeinitiationofHRT.Melasmaoccursmostcommonlyinwomen,andonly
about10%ofpatientsaremales.Thedisorderisverycommon,particularlyamongpregnant
womenandwomentakingcombinationestrogenandprogesteronetherapywhoareexposedto

sunlight.Melasmadoesnotoccurinwomentakingunopposedestrogentherapy.
Postinflammatoryhypermelanoticmacules(A)mayresemblethispatient'sdisorderbutoccur
inpatientsfollowinganinflammatoryresponsesuchasadrugeruption,psoriasis,orlichen
planus.SLE(B)mayalsopresentwiththecharacteristicmalarrashonwomeninthis
patient'sagegroup.However,thisdiagnosisismuchlesslikelygiventheabsenceofanyfamily
orpersonalhistoryofSLEandtherecentinitiationofHRT.Vitiligo(D)presentsina
distributionsimilartomelasma.However,thepresentationisthatofacompleteabsenceof
melanocytesandresultinghypopigmentationasopposedtothehyperpigmentationseenin
melasma.Thispatient'spresentationcouldbeseenfollowingadrugeruption(E)as
postinflammatoryhypermelanoticmaculesbutwouldnotbetheinitialpresentationofa
hypersensitivityreaction.
76.TheanswerisD:Recommendthatsheavoidsexposuretosunlight.Thispatient
hasmelasma,alsocalledthemaskofpregnancywhenitoccursinpregnantpatients.Sunlight
accentuatesthehyperpigmentation,andpatientsshouldbeadvisedtoavoidsunexposureas
muchaspossible(D).Topicaltherapywithhydroquinonecreamcanbeusedtolightenareas
ofhyperpigmentation,buttopicaltherapywithhydrocortisonecream(A)wouldnotbe
beneficial.Hormonesduringpregnancy,suchasestrogenandprogesterone,arethoughtto
stimulatemelanocytes,thusatopicalprogesteronecream(B)wouldnotbehelpful.Thereis
noneedforbiopsy(C)asthelesionsarebenignandtypicallyfadeoncepregnancyhasended
orhormonaltherapyisstopped.Theselesionsarenotcommonlyassociatedwithaging(E).
Theyareassociatedwithpregnancy,combinedHRT,andhormonalcontraceptionthatcontains
bothestrogenandprogesterone.

Melasma

Melasma,alsoknownasthemaskofpregnancywhenpresentinpregnantwomen,isatanordark
skindiscoloration.Thedarkpatchesarecommonlyseenonthecheeks,nose,upperlip,andforehead
areas.Melasmaismorecommoninwomen,especiallypregnantwomenandthosetakingcombined
oralorpatchcontraceptivesorHRT.Thecombinationofestrogenandprogesteronearethoughtto

stimulatemelanocytes.Sunlightaccentuatesthishyperpigmentation.Thelesionstypicallyfadeafter
stoppinghormonetherapyoratthecessationofpregnancy.
Masklikehyperpigmentationonface
Moreoftenseeninwomenduringpregnancyorinwomentakingcombinedestrogenand
progesterone
Sunlightaccentuatespigmentation
Treatment:minimizesunexposureorapplyhydroquinonecream(worksforany
hyperpigmentation)
Notes

77.A24yearoldmancomestotheclinicforaneruptiononhisface,back,neck,andthedorsa
ofhishands.Inthepasthewastoldtherashonhisfacewasseborrheaandwastreatedwith
topicalglucocorticoids.Healsocomplainsof2monthsoffatigueandweakness.Heisa
graduatestudentandhasdifficultyclimbingthethreeflightsofstairstohislecturehalland
office.Physicalexaminationrevealsaviolaceousperiorbitalrashmostprominentontheupper
eyelids,erythematousplaquesontheposteriorshouldersandneck,violaceouspapulesonthe
knuckles,anddystrophicnailswithtelangiectasiaspresent.Musclestrengthis4/5inall
extremities.Whichofthefollowingstatementsregardingthispatientis/aretrue?
A.Diagnosismustbeconfirmedwithtestingforantinuclearandmyositisspecific
antibodies.
B.Shortnessofbreathisalwaysrelatedtointerstitiallungdisease.
C.HallmarkskinlesionsofheliotropeperiorbitalrashandGottrenpapulesare
pathognomonicforthisdisease.

D.Systemictreatmentshouldonlybeinstitutedforadvancedcases.
E.AandConly
78.A37yearoldmanpresentstotheclinicwith3monthsofdifficultystandingonceseated
andweaknessinreachingabovehishead.Hereportssomemuscleachesandmorningjoint
painandstiffness.Healsocomplainsofdifficultyswallowingandshortnessofbreathwith
exercise.Forthelastfewweeksthepatienthasnoticedaredrasharoundhiseyesandonhis
chestandshouldersafteraweek'svacationatthebeach.Physicalexamshowsaperiorbital
heliotroperash,patchesoferythemaontheposteriorshouldersandchest,violaceouspapules
ontheknuckles,andperiungualerythemawithtelangiectasias.Musclestrengthis4/5inall
extremities.Whichofthefollowingis/aretreatmentoption(s)forthispatient?
A.Decreasesunexposure
B.Hydroxychloroquine
C.Oralprednisone
D.Methotrexate
E.Physicaltherapy
F.Alloftheabove
77.TheanswerisC:HallmarkskinlesionsofheliotropeperiorbitalrashandGottrenpapules
arepathognomonicforthisdisease.Thispatienthasthehallmarksignsofdermatomyositis
(DM),whichincludetheheliotropeperiorbitalrash,Gottrenpapules(violaceouspapulesonthe
knuckles),andproximalmuscleweakness(C).Macularerythemaontheposteriorshoulders
andneck(shawlsign)isalsopresent.Autoantibodytestingisnotrequiredforthediagnosisbut
maybeofprognosticvalue.ShortnessofbreathinapatientwithDMcanhaveavarietyof
causes,includingweaknessofthemusclesofrespiration,pulmonarydisease(restrictivelung
disease,interstitialpneumonitis,aspirationpneumonia,opportunisticinfections),and
myocardialdisease(conductiondefects,cardiomyopathy).Earlysystemictreatmentslowsthe
progressionofdiseaseandisassociatedwithimprovedoutcomes.
78.TheanswerisF:Alloftheabove.Thispatientexhibitscutaneousdiseaseandsystemic
effectsofDMwithproximalmuscleweaknessandesophageal,pulmonary,andjoint
involvement.Oralprednisoneiscentraltotreatment.However,for25%ofpatientswhowill
notrespondtosteroids,adjuvanttherapywithmethotrexateandotherchemotherapyagents
(mycophenolatemofetil[MMF],azathioprine,cyclophosphamide)canbeused.Patientsneed
physicaltherapytopreventmuscleatrophyfromDM.Cutaneousdiseaseisphotosensitive,so
bothdecreasingsunexposureandusingsunscreenarekey.Additionally,antimalarial
medications(hydroxychloroquine)areeffectiveintreatingcutaneousDM(F).

Dermatomyositis

Dermatomyositis(DM)isanautoimmunemyopathythatresultsinskeletalmuscleweaknessand
cutaneousinflammatorydisease.Itischaracterizedbyviolaceous(heliotrope)inflammatorychanges
oftheeyelidsandperiorbitalarea,macularerythemaoftheposteriorshouldersandneck(shawl
sign),andviolaceousorlacypinkGottrenpapulesovertheknuckles.Itmayhaveassociated
polymyositis,interstitialpneumonitis,myocardialdisease,andjointdisease.Juvenileonsetof
diseasemaybeassociatedwithvasculitisandcalcinosis,whileadultonset(age>55)diseasemaybe
relatedtomalignancy.Laboratorytestswillshowincreasedserumcreatinekinasewithmuscle
disease,andANAandmyositisspecificautoantibodiesmaybedetected.Treatmentisaimedat
decreasingexposuretosunlight,localtherapywithtopicalcorticosteroidstodecreasecutaneous
inflammationandpruritus,andsystemictherapyconsistingofsystemicglucocorticoids,
hydroxychloroquine,andantihistamines.
Classicviolaceous(heliotrope)rashofperiorbitalareaandeyelidsphotosensitivity
Shawlsign:macularerythemaonshoulders,neck,andchest
GottrenpapulesaredistinctviolaceouspapulesovertheDIPjoints.
Progressiveproximalmuscleweaknessmanifestedbydifficultyrisingfromseatedposition,
raisingthearmsabovethehead,andclimbingstairs
Notes

79.A74yearoldwomanpresentstoyourofficecomplainingofblistersonherarms,legs,and
lowerabdomen.Sheinitiallyhaditchingandhivesintheareapriortodevelopingtheblisters.
Herpastmedicalhistoryisremarkableforaurinarytractinfectiontreatedwith
trimethoprim/sulfamethoxazole1yearagoandprehypertensiontreatedwithlifestylechanges.
Physicalexamrevealstenseblistersontheupperarms,forearms,thighs,lowerlegs,andlower
abdomen.Noorallesionsarepresent.Someexcoriationsarepresent,andseveralblistershave
ruptured,leavingbehinderosions.Abiopsyofthelesionrevealsasubepidermalblisterand
superficialdermalinfiltrate.Immunostainingofthebiopsyshowsalinearpatternalongthe
basementmembrane.Whatarethemostlikelydiagnosisandfirstlinetreatmentforthis
patient?
A.Idiopathicbullouspemphigoidtopicalcorticosteroidsandanesthetics
B.Idiopathicbullouspemphigoidoralglucocorticoids
C.Druginducedpemphigoidoralglucocorticoidsanddiscontinuationofoffendingagent
D.Druginducedpemphigoidimmunosuppressanttherapyonly
E.Pemphigusvulgaris(PV)oralglucocorticoidsandanimmunosuppressant
80.A63yearoldwomanwastreatedintheoffice1weekagoforaurinarytractinfectionwith
ciprofloxacin.Shenowpresentswith1to2cmblistersonherabdomen,inhergroin,andon
theflexorsurfacesofherlegs.Physicalexamshowsthat,afterdigitalcompression,theblisters
donotchangesize,andtheydonotbreak.Digitalcompressionofthebonysurfacesofthelegs
doesnotproduceanylesions.Whatisthemostlikelydiagnosisinthispatient?
A.PV
B.Bullouspemphigoid
C.Dermatitisherpetiformis
D.StevenJohnsonsyndrome
E.Scaldedskinsyndrome
79.TheanswerisB:Idiopathicbullouspemphigoidoralglucocorticoids.This
patientissufferingfromidiopathicbullouspemphigoid,whichcanbeselflimitingbutmay
persistformonthstoyears,withexacerbationsandremissions.Thebiopsyshowstheclassic
findingsofasubepidermalblisterwithasuperficialdermalinfiltrate,andimmunofluorescence
revealsthecharacteristiclinearpatternthatresultsfromautoantibodiesagainst

hemidesmosomesthatconnectbasalcellstothebasementmembrane.Generalizedlesions,as
inthispatient,requireoralglucocorticoids(B)and/orimmunosuppressiveagents
(azathioprine,MMF,orcyclophosphamide).Localizeddiseasecanbetreatedwithtopical
corticosteroidsalone(A).Pemphigoidcanalsobeareactiontoamedication,suchassulfa
drugs,certainantibiotics,orblockers.However,herhistorysulfadruguseisremoteand
thereforeunlikelytobethecauseofhercondition(C,D).
80.TheanswerisB:Bullouspemphigoid.Thispatientissufferingfromdruginduced
bullouspemphigoid(B),whichcaneasilybedifferentiatedfromPV(A)sincethelatteris
characterizedbyflaccid,ratherthantense,lesions.TheflaccidlesionsofPVgetbiggerafter
digitalcompressionandcanbeproducedbydigitalcompressiononboneysurfaces(Nikolsky
sign).Bullouspemphigoidischaracterizedbytenselesions,mostlylocatedonflexorsurfaces.It
iscausedbyautoantibodiesagainstbasementmembranehemidesmosomalglycoproteinsand
maybeeitherspontaneousordruginduced.Dermatitisherpetiformis(C),whichisrelatedto
IgAproduction,isassociatedwithceliacsprueandproducesvesicles/blistersthatareclosely
clustered(asintheherpeticlesions,hencethename).StevenJohnsonsyndrome(D),also
referredtoasmucocutaneoussyndrome,isadruginducedsyndromethatproducesboth
cutaneousandmucosalblisters.Blistersarealsopresentinscaldedskinsyndrome(E),which
isrelatedtothesuperantigentoxinproducedbyS.aureus.However,thisisunlikelytoexplain
thepatient'ssymptomsinthiscase,becauseS.aureusisunlikelytobethecauseofthepatient's
urinarytractinfection.

BullousPemphigoid

Bullouspemphigoidisanautoimmunediseasethatcausessuperficialblistering,usuallyinpatients
overage60.Thelesions,whichmaturefrompruriticurticariatotensebullae,aretypicallyfoundon
thelowerabdomen,groin,andflexorsurfacesoftheextremities,butmaybelocatedontheoral
mucosainaminorityofpatients.Asthelesionsevolve,theblistersoftenruptureandarereplacedby
erosions.
Histologyshowingsubepidermalblisterwithoutepidermalnecrosisandasuperficialdermal
infiltratecontaininglymphocytes,histiocytes,andeosinophils

Immunopathologyrevealsimmunoreactantsdepositedinalinearpatternalongthebasement
membraneC3detectedinalmostallpatientsIgGinmostpatientsaswell
Autoantibodiesdirectedagainstbullouspemphigoidantigeninthehemidesmosome,which
anchorsthebasalcelltothebasementmembrane
Notes

81.A65yearoldmanpresentswitha2weekhistoryofwhitepatchesofskinonhisbilateral
hands.Theyareasymptomatic,andhedeniesanyhistoryoftopicalmedicationuse.Whichof
thefollowingismostlikelyassociatedwiththispatient'scondition?
A.Guttatehypomelanosis
B.Tineacorporis
C.Perniciousanemia
D.Pityriasisrosea
E.Tineaversicolor
82.A24yearoldmanpresentstoyourofficeforaroutinecheckup.Heisconcernedaboutthe
appearanceofhisskinandrequestsanexplanationforthedepigmentedareasonhishandsand
arms.Hehashadtheselesionssincebirthbutnoticesthattheychangeinshape.Whatisthe
mostlikelyexplanationfortheseareasofdepigmentation?
A.OverproductionofmelaninduetoadefectintheTYRgene
B.Destructionorlossofmelanocytesofunknownetiology

C.InfectionwithMalasseziaglobosa
D.Corticosteroiduseininfancy
E.Drugreactiontobenzoylperoxidetreatment.
81.TheanswerisC:Perniciousanemia.Thispatienthasvitiligo,anacquiredlossof
pigmentationdescribedhistologicallybythelossofmelanocytesintheepidermis.Vitiligohas
beenassociatedwithanumberofdiseases,includingperniciousanemia(C),stemmingfromB
12deficiency.Guttatehypomelanosisisabenignacquiredleukodermaofunknownetiology

mostcommonlyseeninmiddleaged,lightskinnedwomen(A).Fungalinfections(B,D,E)
mayhavesimilarappearancetovitiligo,butthepatient'sassociatedsymptomsmakethis
diagnosislesslikely.
82.TheanswerisB:Destructionorlossofmelanocytesofunknownetiology.The
patientinthevignettemostlikelyhasvitiligo,whichisduetolossofmelanocytes.Whilethe
originofvitiligoisunknown,itmayarisefromautoimmune,genetic,oxidativestress,neural,
orviralcauses(B).Overproductionofmelanocyteswouldnotleadtoareasofdepigmentation
(A).InfectionwithMalasseziaglobosa(C)isresponsibleforthemajorityofcasesoftinea
versicolor.Corticosteroiduseorbenzoylperoxidetreatment(D,E)areunlikelytoleadtothe
picturedpatternofhypopigmentation.

Vitiligo

Vitiligoisanacquiredlossofpigmentationdescribedhistologicallybythelossofepidermal
melanocytes.
AssociatedwithautoimmunediseasessuchasHashimotothyroiditis,Addisondisease,type1
diabetes,andperniciousanemia
Thediseaseisthoughttohaveahereditarycomponent.
Commonlyinvolvedsitesincludetheface,bodyfolds,andthebackofthehands.
Notes

83.A45yearoldCambodianwomanpresentstotherefugeeclinicwithnocomplaints.On
exam,afewdiscretelargeandsmallhypopigmentedmaculesarenotedontheleftsideofher
back.Biopsyrevealslargenumbersoflymphocytes,lownumbersofAFB,andnoncaseating
granulomasinvolvingnerves.Whatisthenextbeststepinmanagementofthispatient?
A.Oralprednisone
B.Dapsone,clofazimine,andrifampin
C.Sunscreen
D.Gabapentin
E.NSAID
84.A50yearoldmanfromIndiapresentstotheclinicforroutinevaccinations.Hispast
medicalhistoryissignificantforlepromatousleprosythatwastreated3yearsago.Onphysical
exam,younoticethefollowing:leoninefaciesathickened,palpableleftulnarnervea
paralyzed,leftclawhandwithshorteneddigitsandhypopigmentedtoerythematous,annular,
raised,hypoestheticplaquesthroughouthisbody.Whichofthefollowingis/aretrueregarding
lepromatousleprosy?
A.Commonsequelaeareduetoskininfiltrationandnervedamage.
B.Causalorganism,Mycobacteriumleprae,isunabletobeculturedinvitro.
C.Diagnosedwithskinslitsmearsorskinbiopsies
D.Treatmentisdapsone,rifampin,andclofaziminefor2years.
E.Alloftheabove

83.TheanswerisB:Dapsone,clofazimine,andrifampin.Thispatienthas
multibacillary,tuberculoidleprosy(TL)withsixormorediscrete,hypopigmentedmaculeson
theleftsideofherleftback.SensorylossintheselesionsiscommoninTL.M.lepraedamages
peripheralnervesbyinvadingSchwanncellsandcausingdemyelination.Highernumbersof
inflammatorylymphocytes(Th1,CD4+,Tcells)inTLlimitandcontrolinfectionbutaddto
nervedamage.Thetreatmentregimenformultibacillaryleprosyisdapsone,clofazimine,and
rifampinfor2years(B).Sideeffectsincludehemolysiswithdapsone,orangediscolorationof
bodilyfluidswithrifampin,andredtoblueskindiscolorationwithclofazimine.
84.TheanswerisE:Alloftheabove.Thispatienthasburnedout,treatedlepromatous
leprosy(LL),withmultiplehypopigmentedanderythematousmacules,papulesand/ornodules
withlossofsensation.ThediseaseiscausedbyM.leprae,whichisnonamenabletocultureto
dateandendemicprimarilyinBrazil,India,andpartsofSoutheastAsia.Aclassicsymptomof
LLisleoninefacies,thickenedskin,andmarkedskinfoldsonthefaceduetodermal
infiltration.NervedamageinitiatedbySchwanncellinvasioncausesbothcutaneouslesional
hypoesthesiaandasymmetricperipheralneuropathy(inthiscaseulnarnervedamageandclaw
hand).Affectedperipheralnerves(ulnar,median,posteriortibial)becomeinflamedand
palpable.Boneinfectionhascausedreabsorptionofthispatient'slefthanddigitalbones.Inthe
diagnosisofLL,skinslitsmearsandbiopsywouldidentifymanyAFB.Thetreatmentregimen
forLLisa2yearcourseofcombineddapsone,rifampin,andclofazimine(E).In
paucibacillaryleprosy,dapsoneandrifampinareusedfor1year.

Leprosy

Leprosy(Hansendisease)isararelyfatal(deathsfromamyloidosisandchronicrenalfailurehave
beenreported),chronicinfectiousdiseasecausedbyM.leprae.Clinicalmanifestationsarelargely
confinedtotheskin,peripheralnervoussystem,upperrespiratorytract,eyes,andtestes.Morbidity
isrelatedtothisuniquetropismandcausescharacteristicdeformitiesifleftuntreated.Thespectrum
ofdiseasefrompolartuberculoidtofullblownpolarlepromatousdiseasecorrelateswithsymmetric
generalizedskinmanifestations,increasingbacterialload,andlossofTcellmediatedimmunity
againstM.leprae.Treatmentiswithanestablishedregimenofdapsone,clofazimine,andrifampin.
Leprosyisadiseaseofthedevelopingworld,affectingareasofAsia,Africa,LatinAmerica,
andthePacific.
TL:leastsevereformofdiseasecharacterizedbyafewdiscretehypopigmentedmaculesor
plaqueswithabsentorlownumbersofAFBasymmetricenlargementofoneorafew
peripheralnervesmaybepresent
LL:symmetricallydistributedskinnodules,raisedplaques,anddiffusedermalinfiltration
leoninefaciesresultsifthefaceisinvolvedAFBarenumerousintheskinaswellasthe
peripheralnerves,wheretheydamagebothSchwanncellsandaxons
Notes

85.A43yearoldwomanpresentstoyourofficecomplainingofapainlessnoduleonher
elbow.Thisnoduleappeared4daysago,andthepatientcannotrecallanytraumaorabnormal
activity.Shedeniesothersymptoms,butasyoucontinuetotakeahistory,thepatientrecalls
painandcrepitusinherwristsforthepast3yearsthatisrelievedwithNSAIDs.Sheusually
experiencespainandstiffnessinthemorningbutfindsthatitisrelievedasshegoesabouther
dailyactivities.Thepatientisveryconcernedformalignancy,andyoudecidetoperforma
biopsyofthenodule.Whichofthefollowinghistologicalpatternsismostconsistentwith

rheumatoidarthritis(RA)?
A.Septalpanniculitiswithacuteandchronicinflammationinthefatandaroundblood
vessels
B.Ashelloffibroustissuesurroundingacenteroffibrinousnecrosis
C.Homogeneouseosinophilicstainingwithhematoxylinandeosin
D.Nodularlesionsintheupperdermiscomposedofcalciumphosphate
E.Tumorcellswithanelongatedshape,intracellularhyalineglobs,andhighvascularity
86.A60yearoldwomanpresentswithincreasingmorningstiffnessandpaininherfingers
andhands.Shehasseveretendernessoverhermetacarpophalangeal(MCP)andproximal
interphalangeal(PIP)jointsandhashaddifficultyusingherhandsduetothepain.Shehas
severeswanneckdeformitiesandboutonnieredeformitiesofherhands.Thewomanalsonotes
thatshehashaddifficultybreathingandhaspainwhenshetakesdeepbreaths.Whichofthe
followingisariskfactorforthisdisease?
A.HLAB27positive
B.HLADR4positive
C.AntidsDNAantibodies
D.Anticentromereantibodies
E.AntiSmantibodies
85.ThecorrectanswerisB:Ashelloffibroustissuesurroundingacenterof
fibrinousnecrosis.FivepercentofpatientswithRAexperiencerheumatoidnoduleswithin
thefirst5yearsofdiagnosis,andtheabsoluteprevalenceofrheumatoidnodulesinpatients
withRAis25%.Nodulesrangeinsize(frompeasizedtothesizeofaclementine)andare
usuallysubcutaneous,primarilyoccurringoverbonyprominencesliketheknucklesorelbows.
Mostarepainlessandcauselittletonodisability.Histologicalexaminationshowsafibrous
shellwithacenteroffibrousnecrosis.Smallernodulestendtobeunilocular,whilelarger
nodulesareoftenmultilocular,separatedbyafibrinousbandandcontainingsynovialfluid.
Septalpanniculitiswithacuteandchronicinflammationinthefatandaroundbloodvessels(A
)isconsistentwitherythemanodosum.Nodulesusuallyoccurontheshinsandareassociated
withHomogeneouseosinophilicstainingwithhematoxylinandeosin(C)isconsistentwith
amyloidosis.Nodularlesionsintheupperdermiscomposedofcalciumphosphate(D)occur
incalcinosiscutis,aprocessofdystrophiccalcificationthatisusuallyfoundatsitesofprevious
inflammationordamagetotheskin.Tumorcellswithanelongatedshape,highvascularity,and
intracellularhyalineglobsareconsistentwithKaposisarcoma.
86.ThecorrectanswerisB:HLADR4positive.ThiswomanhasRA.Shehasclassic

jointfindingsanddeformitiesduetothedestructiveinflammatoryarthritis.Sheisalso
experiencingpainwithdeepbreathing,whichmayindicatepleuritis.Pleuritiscanbeassociated
withautoimmunediseasessuchasRAandSLE.HLADR4positive(B)patientsareat
increasedriskofRA.PatientswhoareHLAB27positive(A)areatincreasedriskof
ankylosingspondylitis.AntidsDNAantibodies(C)andantiSmantibodies(E)arehighly
specificforlupus.AnticentromereantibodiesarespecificforCRESTsyndrome.

RheumatoidArthritis(RA)

Rheumatoidarthritis(RA)isachronicsystemicinflammatorydisorderinvolvingmanytissuesand
organs,particularlythesynovialjoints.Theinflammatoryresponsecausesthesynoviumofthejoint
tobecomeinflamedandeventuallyerodescartilageandbone.Inthehands,rheumatoidarthritis
maycausejointproblemsandhanddeformitiesincludingrheumatoidnodules,swelling,joint
stiffness,contractures,ulnardrift,orwristsubluxation.Thispatientisdemonstratingmanyofthe
moreclassicsignsofRAinthehand.BoutonnieredeformityishyperextensionoftheDIPjointand
flexionofthePIPjoint.SwanneckdeformityisflexionoftheDIPjointandextensionofthePIPjoint.
RAcanalsoaffectlargerjoints,suchasthekneesandshoulders.Otherfindingsincluderheumatoid
nodules,crepitusduringmovement,andjointswelling.RAcanalsoproducediffuseinflammationin
thelungs,pericardium,pleura,andsclera.Onsetismostfrequentbetweentheagesof40and50,and
thediseaseaffectswomenmorethanmen.Xrayscanshownarrowingofthejointspace,swelling
anddiminishedbonedensitynearthejoints,anderosionsofthebone.Jointaspirationandblood
testsforrheumatoidfactorareusefultoolstohelpmakethediagnosis.Serologicelevationof
rheumatoidfactorisoftenfoundinRA,butisnonspecificasitisalsofoundin5%ofhealthyadults
andisassociatedwithanumberofotherconditions.TreatmentforRAismainlysupportive,withthe
maingoalsoftreatmenttorelievepainandrestorefunctiontothejoints.NSAIDsandoralsteroids
canhelpdecreaseinflammation,relievepain,andslowprogressionofthedisease.Severaldisease
modifyingtreatmentsarenowavailable,includingantimalarialdrugs,methotrexate,cyclosporine,
gold,remicade,andenbrel.Physicaltherapy,splints,andwarmwax(paraffin)bathsmayprovide
somerelief.
Rheumatoidnodulesconsistofashelloffibroustissuesurroundingacenteroffibrinoid
necrosis.

Swanneckdeformitiesandboutonnieredeformitiesareclassicdeformitiesofthehandsin
patientswithRA.
Findingsincludechronicsynovitiswithpannusformationandhyperplasiaofsynovialcells.
Morningstiffness,jointpainthatisworseafteraperiodofrestandisoftenrelievedwith
activity
Usuallysymmetricandmostcommonlyaffectshandsandfeet
HigherriskofRAifHLADR4positive
Notes

87.A7yearoldgirlisbroughttotheclinicbyhermother,whostatesshehasafeverandhas
refusedtoeatoverthepast2days.Thepatientadmitsshedoesnotfeelwell,andhermouth
hurtswhensheeatsfood.Physicalexaminationrevealsatemperatureof100.4F(38C)oral
vesiclesandulcerationswithsurroundingerythemaonthetongue,gingiva,andbuccalmucosa
andgrayvesiclesonherpalmsandsoles(shownabove).Whichofthefollowingisthemost
likelycauseofthepatient'ssymptoms?
A.CoxsackievirusgroupA
B.Poliovirus
C.HSV
D.Rickettsialorganism
E.Echovirus

88.Atoddlerisbroughttotheclinicbyhisfatherwhoisconcernedaboutspotsthatsuddenly
appearedonhisson'shandsandfeet.Thetoddlerrecentlystartedgoingtoadaycarecenter.
Thefathernotesthathissonhasn'tbeeneatingasmuchasusualthepast2days.Thetoddler
hasatemperatureof100.7F(38.2C).Therearevesiclesonhishandsandfeet,aswellasin
hismouth.Whichofthefollowingistrueofthetoddler'scondition?
A.Heislikelytohavefebrileseizures.
B.Heislikelytodevelopsepsis.
C.Heislikelytodevelopencephalopathy.
D.Heislikelytodevelopthrombocytopenia.
E.Heisunlikelytodevelopcomplications,asthediseaseistypicallyselflimited.
87.TheanswerisA:CoxsackievirusgroupA.Thispatienthashandfootandmouthdisease
causedbycoxsackievirusgroupA,asinglestrandedRNAenterovirus(A).Thisdiseaseis
transmittedbythefecaloralrouteandischaracterizedbyaprodromeoffever,anorexia,and
malaise,followed12dayslaterbythecharacteristiclesionsinthemouthandonthehandsand
feet.Theinitiallesionsappearinthemouthas4to8mmvesicleswithanerythematousbase,
whichlaterulcerate.Theyarelocatedonthetongue,softpalate,gingiva,andbuccalmucosa.
Lesionsonthepalmsandsolesappearshortlyafterwardasredpapulesthatprogresstogray
vesicles.Treatmentissupportive,andthelesionshealin710days.
88.TheanswerisE:Heisunlikelytodevelopcomplications,asthediseaseis
typicallyselflimited.Thetoddlerhashandfootandmouthdisease,whichisaselflimited
conditioncausedbycoxsackievirusgroupA.Suddeneruptionofvesiclesonthepalmsandsoles
andinthemouthispathognomonicforhandfootandmouthdisease.Heisunlikelytodevelop
seriouscomplications(A,B,C,D)andwillrecoveronhisowninapproximately10days(E).

Coxsackievirus

Coxsackievirusisanimportantcauseofillnessinyoungchildren.Itisanenterovirusspreadthrough

theoraloralandfecaloralroutesthatcauseshandfootandmouthdisease.Therash
characteristicallypresentsonthepalmsandsoles,aswellasinthemouth,asrhomboidorsquare
vesicularlesionswitherythematousbases.
Transmissionviafecaloralroute
GroupAviruses:handfootandmouthdisease,herpangina,acutehemorrhagic
conjunctivitis
GroupBviruses:pleurodynia,myocarditis,pericarditis
Allcoxsackievirusescancausenonspecificupperrespiratoryinfection,febrilerashes,aseptic
meningitis,andparalysis,whichisoftenincompleteandreversible.
Notes

89.A42yearoldmanpresentswithtinyverticallinesthatrununderthedistalareaofhis
fingernails.Theyaremostprominentonhissecondandthirddigitsbilaterally.Allofthe
followingshouldbeconsideredinyourdifferentialdiagnosisexcept:
A.Microembolifromcholesterolplaque
B.Subacutebacterialendocarditis
C.Subungualmelanoma
D.Scleroderma
E.Trauma
90.A55yearoldmanwithahistoryofIVdrugusepresentstoyourofficecomplainingofa
subjectivefever,weakness,andfatigueforthepastmonth.Uponphysicalexam,younotice
small,dark,verticallinesalonghisnailbed(shownabove).Twoseparatebloodculturesare

positiveforcommunityacquiredS.aureusinabsenceofaprimaryfocus.Whichofthe
followingcriteriawouldhelptofulfilladiagnosisofinfectiveendocarditis?
A.Evidenceofendocardialinvolvementwithanechocardiogramdemonstratingan
intracardiacmassonthetricuspidvalve
B.Worseningorchangingofapreexistingmurmuronphysicalexam
C.Evidenceofconjunctivalhemorrhages
D.Nontender,erythematouslesionsonthepalmsandsoles
E.Temperatureover37.5C
89.TheanswerisC:Subungualmelanoma.Allofthefollowingarepartofthedifferential
diagnosisforsplinterhemorrhages,whicharetinylinesthatrunverticallyunderthedistal
nails,exceptsubungualmelanoma(C).Althoughsubungualmelanomasaredarkverticallines
alongthenail,itisraretohavemultipledigitsinvolved,andthelesionswouldnotbemore
likelytooccurinthedistalnail.Microemboli(A),subacutebacterialendocarditis(B),and
trauma(E)mayallcausesplinterhemorrhages.Inaddition,systemicvasculitisdueto
scleroderma(D),SLE,orRAcanalsocausesplinterhemorrhages,whichresultfromrupture
ofnailbedcapillaries.
90.TheanswerisA:Evidenceofendocardialinvolvementwithan
echocardiogramdemonstratinganintracardiacmassonthetricuspidvalve.The
Dukecriteriaareacollectionofmajorandminorcriteriausedtoestablishthediagnosisof
endocarditis.Therearethreewaystoreachadiagnosis:twomajorcriteria,onemajorandthree
minorcriteria,orfiveminorcriteria.
Majorcriteriainclude:
PositivebloodculturewithatypicalmicroorganismsuchasS.aureusfromtwoseparate
bloodcultures
Evidenceofendocardialinvolvementviaapositiveechocardiogramdemonstratingan
absence,newpartialdehiscenceofaprostheticvalve,newvalvularregurgitation(increaseor
changeinpreexistingmurmurnotsufficient),orintracardiacmassonavalveorsupporting
structure
Minorcriteriainclude:
Predisposingheartconditionorevidenceofinjectiondruguse
Feverover100.4F(38C)
EvidenceofvascularembolisationsuchasJanewaylesions,majorarterialemboli,septic
pulmonaryinfarcts,ormycoticaneurysm

Immunologicphenomena:glomerulonephritis,Oslernodes,Rothspots,rheumatoidfactor
Evidenceofactiveinfectionwithorganismconsistentwithinfectiveendocarditisorpositive
bloodculturenotmeetingmajorcriteria
Thispatienthasonemajorcriterionandneedsanothermajorcriterion(A)orthreeadditional
minorcriteria.Evidenceofaworseningmurmur(B)isinsufficienttomakethediagnosisof
endocarditis.C,D,andEallrepresentminorcriteriaininsufficientquantitytomakethediagnosis.

SplinterHemorrhage

Splinterhemorrhagesaretinylinesthatrunverticallyundernails.Theyaremorecommonlylocated
inthedistalnailplateandrepresentruptureofnailbedcapillaries.Thisfindingisnonspecific,but
mayduetotrauma,aclotfromheart(mostoftenduetosubacutebacterialendocarditis),cholesterol
microemboli,orsystemicvasculitis(duetoSLE,scleroderma,RA,etc).
Verticallineslocatedatdistalnailplate
Duetoruptureofnailbedcapillaries
Associatedwithsystemicillnesses,autoimmunediseases,trauma,etc.
Notes

91.A45yearoldwomanpresentswithpainfuldenudedareasofskin.Younoteseveralflaccid
bullaethateasilyrupturewhentouched.Thewomanhascutaneousandmucosallesions.There
isapositiveNikolskysign.Whatwouldhelptomakethediagnosisinthiscase?
A.Skinbiopsyshowingimmunofluorescenceasalinearbandalongthebasement
membranewithincreasedeosinophilsinthedermis
B.Skinbiopsyshowingimmunofluorescencesurroundingepidermalcellsshowing
tombstonefluorescentpattern
C.Clinicalhistoryofherpesinfection
D.Woodlampofurineshowingurinefluoresceswithdistinctiveorangepinkcolor
becauseofincreasedlevelofuroporphyrins
E.Recentuseofsulfonamides
92.A48yearoldwomanwithahistoryofdietcontrolledtype2diabetesmellitusand
hypertensionrecentlystartedoncaptoprilpresentstotheEDduetoaneruptionofpainful
blistersthatbeganinhermouthandarenowpresentonhertorsoandback.Onphysicalexam
thepatienthasseveralpainfulflaccidbullaeontheskinofthetorso,andtheepidermisisnoted
toseparatewithmanualpressureappliedtotheseareas.Abiopsyofthepatient'sskinistaken
tobetterdiscernthediagnosis.Whichisthemostlikelyfindingonskinbiopsy?
A.Subepidermalblistersrichwitheosinophils
B.Acantholyticintraepidermalblisters
C.Neutrophilrichinfiltrateswithindermalpapillae
D.Hydropicdegenerationofbasalkeratinocytes
E.Eosinophilicnecrosisoftheepidermis
91.TheanswerisB:Skinbiopsyshowingimmunofluorescencesurrounding
epidermalcellsshowingtombstonefluorescentpattern.Thispatienthasclassic
findingsofPemphigusvulgaris(PV),includingflaccidbullaeandpositiveNikolskysign
(separationoftheepidermisbymanualpressureappliedtotheskin).PVisdiagnosedviaskin
biopsyshowingimmunofluorescencesurroundingepidermalcellsleadingtoatombstone
fluorescentpattern(B).Askinbiopsyshowingimmunofluorescenceasalinearbandalongthe

basementmembranewithincreasedeosinophilsinthedermis(A)isindicativeofbullous
pemphigoid.Bullouspemphigoidisanautoimmuneprocess,butpresentsashard,tensebullae
andanegativeNikolskysign.Aclinicalhistoryofherpesinfection(C)inthesettingof
blisteringmayindicateasevereformofEMleadingtoSJS.EMmaybeseenfollowingaherpes
infectionandusuallypresentswithdiffuseerythematoustargetlesions.SJSisamoresevere
presentationofEMandtypicallyaffectsmucosalsurfaces.Woodlampoftheurine(D)would
behelpfulindiagnosingporphyriacutaneatarda.Thisconditionisduetoanautosomal
dominantdefectinhemesynthesis.Itcancauseblistering,buttheblisteringisgenerally
confinedtosunexposedareas.Recentuseofsulfonamides(E)couldproduceadrugreaction
andresultintoxicepidermalnecrolysis(TEN),whichwouldleadtothewidespreadlossof
sheetsofskin.ThedamageinTENwouldinvolvetheentireepidermis.
92.TheanswerisB:Acantholyticintraepidermalblisters.Thispatientpresentswitha
caseofPV,anautoantibodymediatedintraepidermalblisteringdiseasecharacterizedbylossof
cohesionbetweenepidermalcells,termedacantholysis(B).Thestimulusforthecirculating
autoantibodiesisoftenunknown,butPVhasbeendescribedinassociationwithautoimmune
diseases,malignancy,andmedications,particularlypenicillamineandcaptopril.Patients
developflaccidbullae,whichruptureeasilywithapplicationofpressure(referredtoasa
positiveNikolskysign),onnormalorslightlyerythematousskin.ThediagnosisofPVismade
byidentifyingalossofadhesionbetweenkeratinocytes(acantholysis)abovethebasallayer,
resultinginanintraepidermalsplit.Bullouspemphigoidisalsoaselflimitedautoimmune
blisteringdiseasecharacterizedbytense,subepidermalbullae.Biopsiesshowsubepidermal
blistersrichwitheosinophils(A).Neutrophilrichinfiltrateswithindermalpapillae(C)are
seenindermatitisherpetiformis,anintenselypruritic,papulovesicularskindisease
characterizedbylesionsdistributedoverextensorsurfaces.Hydropicdegenerationofbasal
keratinocytesismorecharacteristicoflesionsseeninDMorSLE.Eosinophilicnecrosisofthe
epidermis(E)isseenwithTENorSJS,andpatientspresentwithpainfulerythematous
maculeswithpurpuriccentersthattendtocoalesceandultimatelyleadtoblistersand
epidermaldetachmentovertheentirebody.

Pemphigusvulgaris

Pemphigusvulgaris(PV)isanautoimmunediseasewhichcauseschronicpainfulblisteringofthe
skin.Itiscausedbyantibodiesagainstdesmosomeproteins(bothdesmoglein1anddesmoglein3),
whichresultsinlossofcohesionbetweenkeratinocytesintheepidermis.Extensiveflaccidblisters
andmucocutaneouserosionsaretypicallyseen.Onhistology,basalkeratinocytesareusuallystill
attachedtothebasementmembrane,leadingtotheappearanceandtermtombstoning.PVisan
intraepidermalprocess,whereasinbullouspemphigoidthedetachmentoccursbetweenthe
epidermisandthedermis.Antidesmogleinautoantibodiescanbevisualizedbydirect
immunofluorescenceonaskinbiopsy.TheantibodiesappearasIgGdepositsalongthedesmosomes
betweenepidermalcells.
Definitivediagnosisviapunchbiopsy
Treatmentwithcorticosteroidsandimmunosuppressivedrugs
Acantholysisisthebreakingapartofepidermalcells.
Bothcutaneousandmucosallesions
Flaccidbullae
PositiveNikolskysign
Notes

93.A31yearoldwomanwithhyperthyroidismcomestotheclinicfordoublevision.This
occursmainlywhenshetriestolookupatobjects.Onphysicalexamination,hereyesappear
protruded,theright(seetongueblade)toagreaterdegreethantheleft.Thereislidedema,lid
retraction,andophthalmoplegiaonattemptedupgaze.ThepatientisdiagnosedwithGraves
ophthalmopathy.Whatsymptomwouldrequireemergenttreatmentofhercondition?
A.Afferentpupillarydefectandslightlossofvisualacuity
B.Diplopiainmultiplegazefields
C.Accompanyingptosisfrommyastheniagravis(MG)
D.Cornealdryness
E.Coexistingbacterialsinusitis
94.A36yearoldwomancomestocliniccomplainingofpalpitations,excessivesweating,heat
intolerance,nervousness,weightlosswithincreasedappetite,difficultysleeping,and
infrequentperiods(every23months).Onexam,thepatient'spulseis115bpm,andblood
pressureis130/85.Sheisdiaphoreticandhasnotableexophthalmoswithlidlagondownward
gaze.Shehaspretibialskinthickeningbilaterally.Anelectrocardiograph(ECG)revealssinus
tachycardia.Labtestsreveallowthyroidstimulatinghormone(TSH),highT3,highT4,and
elevatedthyroidstimulatingimmunoglobulin.Radioactiveiodineuptakeisdiffuselyelevated
throughoutthethyroidgland.Ablockerisprescribed.Whatisthebesttreatmentforher
underlyingconditionatthistime?
A.Levothyroxine
B.Totalthyroidectomy
C.Propylthiouracil
D.Radioactiveiodinethyroidablation
93.TheanswerisA:Afferentpupillarydefectandslightlossofvisualacuity.This
patienthasproptosisasaresultofGravesdisease.Thishyperthyroidstateoccasionallycauses
engorgement(myxedema)oftheextraocularmusclesandforcestheglobetoprotrudeout.
Successfulmanagementrequiresgoodcontrolofthethyroidconditionbyanendocrinologist,
butoccasionallyemergentinterventionfromanophthalmologistisrequired.Thissituation
involvescompressionoftheopticnervefromthemassivelyenlargedextraocularmuscles.Early
signsincludeanafferentpupillarydefect,colorvisionimpairment,andaslightdecreasein
visualacuity(A).Ifdecompressionisnotachievedquickly,blindnessmayresult.
94.TheanswerisC:Propylthiouracil.Thispatientissufferingfromthyrotoxicosisdueto
effectsofhighlevelsofT3andT4.Boththepresenceofthyroidstimulatingimmunoglobulin
anddiffuseradioactiveiodineuptakeinthethyroidhelpconfirmthediagnosisofGraves

disease.Initialsymptomatictherapyiswithblockerstodecreasetachycardia.Firstline
therapytoachieveaeuthyroidstateisoneofthethiocarbamidedrugs(propylthiouracil,
methimazole,carbimazole)toinhibitsynthesisofthyroidhormone(C).Thistherapyisgiven
forupto2years.Almost50%ofpatientswillcontinuetobeeuthyroidafterthiocarbamide
cessation.Otherdefinitivetherapiesincludethyroidectomyandradioactiveiodinethyroid
ablation.Levothyroxineisusedinhypothyroidstates.

GravesProptosis

Gravesdiseasedenoteshyperthyroidismduetoanautoimmunecause,andasmallproportionof
patientswiththisconditiondevelopcharacteristiceyesignsknownasGravesproptosis(aka,Graves
ophthalmopathy,exophthalmos).Theproptosisisoftenasymmetricandmaybeaccompaniedby
thecardinalsignsandsymptomsoflidretraction,cornealexposure,diplopia,gazerestriction,and
visualloss.
Protrusionofthegloberesultsfromorbitalinflammationandengorgementoftheextraocular
muscles,especiallythemedialrectusandinferiorrectus.
Diplopiausuallybeginsintheupperfieldofgazebutmayprogresstoallfieldsofgazewhen
allextraocularmusclesareinvolved.
Theopticnervemaybecompressedandcausevisionloss,whichisamedicalemergency.
Notes

95.A40yearoldwomanpresentswitha1yearhistoryofeasybruising,oralulcers,anda
prominentrashoverhercheeksandnasalbridgethatseemstobemoreprominentafter
exposuretosunlight.Whichofthefollowingisnotpartofthediagnosticcriteriaforher
disease?
A.Rash
B.Oralulcers
C.Photosensitivity
D.Leukopenia
E.Hypercomplementemia
96.A45yearoldwomanpresentswithareasofhypopigmentationonthetragusareaofboth
ears.Shenotesthatoverthepast2yearsshehasnoticedredscalingpatchesinthesameareas,
butthesealwaysresolvedafteraboutamonth.Areviewofsystemsisalsosignificantfora
historyofjointpainandorallesionsoccurringintermittentlyoverthepast3years.Whichof
thefollowingisnotatreatmentthatmaybeindicatedforthispatient'ssymptoms?
A.TopicalsunscreenswithSPF>30
B.Hydroxychloroquine
C.Thalidomide
D.Intralesionaltriamcinoloneacetonide

E.Topicalmetronidazole
95.TheanswerisE:Hypercomplementemiaisnotpartofthediagnosticcriteriafor
lupus.Oftenhypocomplementemiaispresentinactivelupus.Thispatientpresentswithclassic
findingsoflupus,includingamalarrash(A).Lupuspatientsmayalsodevelopadiscoidrash,
whichappearsaserythematous,raisedpatcheswithadherentkeratoticscalingandfollicular
plugging.Oralulcers(B)ornasopharyngealulcerationmaybepresentinlupusandare
usuallypainless.Photosensitivity(C),arthritis,andserositisarealsodiagnosticcriteriafor
lupus.Laboratorymarkersincludeproteinuria,leukopenia(D),lymphopenia,anemia,and
thrombocytopenia.
96.TheanswerisE:Topicalmetronidazole.Thispatientispresentingwithatrophyof
theepidermissecondarytochroniccutaneouslupus,previouslytermeddiscoidlupus.
Inflammationfromthelesiondestroyspigment,resultinginthehypopigmentedareadepicted
intheimagebelow.Thepatientalsonotessymptomsofsystemiclupus,suchasjointpain
(occursinover90%ofpatientsandisoftentheearliestmanifestationoflupus)andoralulcers.
Alloftheabovechoicesarepotentialtreatmentsfordiscoidorsystemiclupusexceptfor(E)
topicalmetronidazole,whichisatreatmentforrosaceaandhasnoroleinthetreatmentof
discoidlupus.(A)SunscreenwithSPF>30isrecommendedtopreventcutaneoussymptoms.
(B)Anantimalarial,hydroxychloroquine,isofteneffectivetotreatthesystemicandcutaneous
manifestationsoflupus.(C)Thalidomideisaneffectivetreatmentfordiscoidlupusinsome
patients.(D)Triamcinoloneacetonidegivenintralesionallymaybeusefulforsmallerlesions,
especiallywhensteroidsproveineffective.

Lupus(MalarRash)

Themalarrashoflupus,alsocalledthebutterflyrash,isoneoftheclassicpresentationsofSLE.Itis
characterizedbyaflatorraisederythematousrashoverthecheeksandbridgeofthenosethatspares
thenasolabialfoldsandtendstooccurafterUVexposure.Amalarrashcanbepresentindiseases
otherthanlupus,soadetailedhistoryisveryimportant.SLEisachronicinflammatorydiseasethat
canaffecttheskin,joints,kidneys,lungs,nervoussystem,serousmembranes,and/orotherorgansof
thebody.Immunologicabnormalitiesarealsoaprominentfeatureofthedisease,especiallythe
productionofanumberofANA.
SLEismostcommoninwomenintheir20sand30s.
AutoantibodiesthatareroutinelyassayedforlupusareANA,antiphospholipidantibodies,
antidsDNA,andSmantibodies.
Over80%oflupuspatientswillhavecutaneousmanifestationsofthediseaseatsomepoint
duringtheirlifetime.
Notes

97.A25yearoldwomanpresentstotheofficecomplainingofarashonherlowerabdomen.
Shestatesitstartedoveramonthagoasasmallareaofdry,itchingskin.Shehadnoreliefwith
moisturizingcreamanddecidedtoseektreatmentwhentherashbecamelarger.Shehasno
othercomplaintsandherpastmedicalhistoryissignificantformild,intermittentasthma.
Whatisthebestnextstepinthediagnosisandtreatmentoftherash?
A.Lowdoseoralprednisoneuntiltherashresolves
B.Allergenskinpatchtestingandtopicaldiphenhydramine

C.Skinbiopsyandreferraltoadermatologist
D.Topicalcorticosteroidsuntiltherashresolves
E.Carefulhistoryandavoidanceoftheoffendingagent

Notes

98.A28yearoldwomancomplainsofarashonherneck.Ithasbeenpresentfor2daysandis
intenselyitchy.Sheattendedacocktailparty4daysago,towhichsheworeanewnecklace.
Whichofthefollowingtestswouldconfirmthecauseofthelesion?
A.Potassiumhydroxidepreparationofscraping
B.Tzancksmear
C.Patchtest
D.Culture
E.Completebloodcountwithdifferential
97.TheanswerisE:Carefulhistoryandavoidanceoftheoffendingagent.This
patientissufferingfromallergiccontactdermatitis,adelayedtypehypersensitivityreactionto
anexogenousagent,whichinthiscase,isthepatient'sbelt.Lesionsoftenappearinthe
distributionofthecontact.Thebestapproachistoidentifytheallergenthroughacareful
historyandadvisethepatienttoavoidcontactwithit(E).Theimmunereactionoccursinan
individualwhoisgeneticallysusceptibleandhasbeenpreviouslysensitizedbycontactwiththe
agent.Theskinispruriticandappearseczematous,withscaly,edematouspatchesandpossibly
weepingvesicles.Lichenificationcanoccurwithchronicexposure.Topicalcorticosteroidstwice
dailyfor23weekscanbeusediftherashdoesnotresolvewithremovaloftheoffendingagent

(D).Oralcorticosteroidsshouldonlybeusedinseverecasesandmustbeslowlytaperedover
morethan3weekstodecreasetheriskofreboundreactions(A).Skinpatchtestingshouldbe
usedincaseswheretreatmentfails(B).Allergiccontactdermatitisisaclinicaldiagnosis,and
skinbiopsyisgenerallynotwarranted(C).
98.TheanswerisC:Patchtest.Theappearanceofthelesionandhistoryareconsistent
withallergiccontactdermatitis.Thispatient'snecklacewasmadeofnickel,acommoncauseof
thisreaction.Contactdermatitisisdiagnosedclinically,butafterthelesionhasbeentreated,
sensitivitytothesuspectedallergencanbeconfirmedwithapatchtest(C).Atopicdermatitis
andherpeszosterarealsodiagnosedclinicallyhowever,eosinophiliamaybeseeninatopic
dermatitis(E),andTzancksmear(B)willshowmultinucleatedgiantcellsinherpessimplex
andvaricellazostervirusinfections.Incasesofcellulitis,culture(D)maybeimportantfor
determiningdrugsensitivity.Potassiumhydroxidepreparationofscrapingscanbeusedto
diagnosefungalinfections,suchastineacorporisandonychomycosis(A).

AllergicContactDermatitis

Allergiccontactdermatitisisadelayedtypehypersensitivityreactiontoanallergentowhicha
patienthasbeenpreviouslysensitized.Hands,feet,eyelids,andlipsarecommonsitesforcontact
dermatitis,sincetheyoftencomeincontactwiththeenvironment.Commonallergensarecontained
inplants(poisonivy,poisonoak),topicalmedications(neomycin,bacitracin,sulfonamides),jewelry
(nickel),antiseptics(thimerosal),chemicals(formaldehyde),treeoilsorperfumes,occupational
exposures(chromium,cobalt),andrubberproducts(latexgloves).
Allergiccontactdermatitismaypresentacutelyafterallergenexposureandinitial
sensitizationorafterexposureinapreviouslysensitizedindividual.
Acutephaseischaracterizedbythedevelopmentoferythematous,indurated,scalyplaques.
Severecasesmaydemonstratevesiculationandbullae.
Lesionsoftenmimicthepatternofexposuretotheallergen.
Treatmentinvolvesremovaloftheoffendingagentandtopicalororalsteroidsand/or

antihistaminesinseverecases.
Notes

99.A24yearoldmancomestothecliniccomplainingofarashonhisarms,hands,andfeet.
Therashdevelopedyesterdayandispruritic.Healsocomplainsofmyalgias,headache,anda
painfulrashinhisgenitalregion.Hehasbeensexuallyactivewithfourpartnersthisyearand
onlyoccasionallywearsacondom.Onphysicalexam,amaculopapularrashwithtargetlesions
ispresentontheforearmsandpalms.Thegenitalrashispresentatthebaseofthepenisand
consistsofmultiplevesiclesonanerythematousbase.ATzancksmearrevealsmultinucleated
giantcells,andthelaboratoryconfirmsdetectionofHSV2DNAbypolymerasechainreaction.
Whatisthebestongoingtherapyforthispatient?
A.Valacyclovir500mgoncedailyindefinitely
B.Hospitalizationandadmissiontotheburnunit
C.IVacyclovir5mg/kgq8hfor5daysandtopicalbenadryl
D.Valacyclovir1gbidfor714days,followedbyvalacyclovir500mgdailyindefinitely
E.Oralprednisone100mgbiduntilresolutionoftherashontheextremitiesand
valacyclovir1gbidfor714days
Notes

100.A19yearoldwomancomesinfora1weekfollowupvisitafterbeingstartedonacourse
ofamoxicillinforacuteotitismedia.Thepatientcomplainsoffeverto102Fandablistering
rasharoundhermouthandhandsthatstarted2daysago.Aftertherashappeared,she
discontinuedantibiotictreatment.Shehasnofevertodayandistoleratingfoodandfluids.On
physicalexam,sheisafebrileandherothervitalsignsarewithinnormallimits.Shehas
conjunctivalhyperemiaandred,coalescingmaculesandbullaeonherfaceandhands.Thereis
evidenceofperioralepidermalnecrosisandslightbuccalmucosadesquamation.Thelesionson
thehandsandfacearepositiveforNikolskysign.Culturesaresentonblood,lesionalfluid,and
stool.Whatisthenextbeststepinthemanagementofthispatient?
A.IVvancomycin40mg/kg/dfor2days
B.Hospitalizationandadmissiontotheburnunit
C.IVacyclovir5mg/kgq8hfor5days
D.Supportivecarewithhydrationandcloseobservation
E.Oralprednisone50mg/dfor7days
99.TheanswerisD:Valacyclovir1gbidfor714days,followedbyvalacyclovir500
mgdailyindefinitely.ThispatientisexperiencingEMminor(nowcalledherpesassociated
erythemamultiforme[HAEM])whichischaracterizedclinicallybyanacuteinflammatoryskin
reactionconsistingofsymmetricmaculopapulareruptionsontheextremities,palms,andsoles
andclassictargetlesions.Themucousmembranesofthemouthmaybeinvolvedaswell.The
mostcommoncauseisHSVinfection.TreatmentofHAEMisgenerallynotindicatedunlessthe
patientisunabletoeatortheeyeisinvolved.ThepatientshouldbetreatedfortheacuteHSV2
genitallesionsandthenmaintainedonantiHSVmedication(suchasvalacyclovir)indefinitely
topreventrecurrencesofbothrashesanddecreasetransmissionofHSV(D).
100.TheanswerisD:Supportivecarewithhydrationandcloseobservation.This

patienthasSJSwithamoxicillinasthetrigger.Aprodromalsyndromeoffeverandocularand
oralirritationusuallyprecedestheskinrashoftargetlesionsorerythematousmacules/papules
uponwhichbullaeformonpalmsandsoles,trunk,neck,proximalupperextremities,andface.
Thentheepidermisnecrotizesanddesquamates.SJSbullaeusuallyformonatleasttwo
mucousmembranes(conjunctivae,nares,mouth,anus,vagina,orurethralopening).This
patientpresentedwithclassiculcerativestomatitisandinflamedconjunctivae.Specialcare
mustbegiventotheconjunctivae,asocularbullaeandscarringcancauseblindness.An
ophthalmologistshouldfollowthesepatients.Themostappropriatecaretobegivenatthis
timeissupportivecare(D).Triggers(allopurinol,antibiotics,anticonvulsants,andNSAIDs)
mustbediscontinued.Sincelessthan6%ofthispatient'sbodysurfacearea(BSA)isaffected,
thereisnoneedtotransferhertoaspecializedburnunitatthistime.Sheshould,however,be
givenadequatehydrationandnutrition,monitoredforelectrolytedisturbancesandanemia,
andreevaluatedforpercentBSAskinloss.Prophylacticantibiotictreatmentisnotused,and
antiviralmedicationisnotindicated(A,C).Althoughbothglucocorticoidsandintravenous
immunoglobulin(IVIG)canbeused,theirefficacyhasnotbeensubstantiallyproven(E).Both
arereservedformoresevereSJSandTEN.

ErythemaMultiforme

Erythemamultiforme(EM)isanacuteinflammatoryskindiseasethatisdividedclinicallyinto
majorandminortypesbasedonclinicalfindings.EMmajorfavorsthetrunk,whileEMminoraffects
theextensorsurfaces,palms,soles,andmucousmembranes.Thelesionsmaybemacular,papular,
urticarial,bullous,orpurpuric.EMminorischaracterizedbytargetlesions,whichhaveaclearcenter
andconcentric,erythematousrings.Itscourseisoftenbenignbutfrequentlyrecurrent.The
diagnosisisclinical.TreatmentisreservedforseverecaseswithhighBSAinvolvement,ocular
manifestations,ororalmucosalinvolvement.Moderatetohighdoseprednisonecanbeusedaswell
asIVIG.PatientswithsevereEMmajor(>2530%BSA)shouldbetreatedinaburnunit.
ThemostcommoncauseofEMminorisHSVinfection(HAEM),andrecurrencescanbe
preventedbycontinueduseofantiHSVmedication,suchasvalacycloviroracyclovir.

ThemostcommoncauseofEMmajorinadultsisdrugreactions(subtypes:SJS<10%BSA
skinlossTEN>30%BSAskinlossoverlapbetween10%and30%BSAskinlossinSJSand
TEN)
Theclinicalcoursehasfrequentrecurrences,withfrequencyandseveritytendingtodecrease
spontaneouslyover2yearsorlonger.
Notes

101.A24yearoldmancomesintotheclinicduringwinterwithapruriticrashonhischestand
legfor3weeks.Henoticeddry,scaly,itchy,redareasonhischestthatbecameinflamedand
largerwithscratching.Hecomplainsofitchingandburning,whichheadmitshavelessenedin
intensitysincetherashfirstappeared.Onphysicalexam,severalerythematous,coinshaped
lesions,24cmindiameter,arenotedonthechestandrightpoplitealfossa.Thelesionsare
erythematouswithmildthickeningandundefinedborders.Whichofthefollowingis/arethe
firstlinetreatmentoption(s)?
A.Avoidingirritants
B.Topicalemollients
C.Antihistamine
D.Topicaltriamcinolonecream
E.Alloftheabove
Notes

102.Amotherbringsher1montholdinfantintotheclinicwithanew,red,flakyrashonher
forehead,cheeks,ears,andscalp.Onphysicalexam,greasyscalinganderythematouspatches
arenotedonthepatient'suppercheeks,forehead,ears,andscalpwithparticularlythickwhite
scalingonthevertexofthescalp.Whichofthefollowingstatementsis/aretrueregardingthis
patient'sdiagnosis?
A.Treatedwithtopicalketoconazole
B.Treatedwithseleniumsulfideshampoo
C.FungiofMalassezia(Pityrosporum)genusisafactorintheinflammatoryprocess.
D.Infantileseborrheicdermatitisdisappearsinthefirstyearoflife.
E.Alloftheabove
101.TheanswerisE:Alloftheabove.Thispatienthasnummular(discoid)eczemawith
atopicdermatitisinasubacutestage(dry,red,scaly,itchy).Theselesionsoftenappearonthe
dorsalsurfaceofhandsorextensorsurfacesofforearmsandlegs.Thekeytotreatmentis
removingknownirritants(hotshowers,excessivewashing,drying,scratching)andallergens.
Keepingthelesionsmoistwithtopicalemollientsdecreasesirritationandinflammation.
Antihistaminesaresometimesneededtodecreaseintenseitchingthatcanelicitscratching,
particularlyduringsleep.Althoughoralsteroidsareindicatedinsomeacutestagesinorderto
penetrateeczematousvesicles,topicalsteroids(triamcinolone)efficientlyspeedresolutionof
thesedry,subacutelesions.Althoughnotasrapidactingassteroids,theimmunosuppressives
tacrolimusandpimecrolimuspreventproductionofcytokines,whichcanbeusefulin
controllinginflammation.Soallofthesetreatments(E)canbeusedtotreateczema,which
resolvesorbecomeschronicinnature.
102.TheanswerisE:Alloftheabove.Thepatientisdiagnosedwithseborrheic

dermatitis,commonlyreferredtoininfantsascradlecap.Seborrheicdermatitiscanbetreated
withtheantifungalketoconazole,topicallowpotencysteroidsforinflammation,and/or
antiseborrheicshampoos(seleniumsulfide)forthescaling.Malasseziarequiresfattosurvive
andispresentindermalregionsofhighlyconcentratedsebaceousglandswhereseborrheic
dermatitisappears.Infantileseborrheicdermatitisismostprevalentinthefirst3monthsoflife
andusuallydissipatesby1yearofage.Applicationoftopicalbabyoilsfollowedbycombingthe
hairwithafinetoothedcombcanbeusedtoremovelargescalesfromthescalp.Soallofthese
treatments(E)canbeusedtotreatseborrheicdermatitis.

Dermatitis

Thetermdermatitisencompassesavarietyofskinconditionsthatmaypresentindifferentways.
Somecasespresentwithacuteedema,erythema,andoozingwithcrustingsomewithmilderythema
aloneandotherswithlichenification,whichistheresultofchronicirritation.Histologically,the
manyformsofdermatitisaredifficulttodistinguish,butknowncausesandthenaturalhistoryof
lesionshelpstocategorizetheconditions.Thesetypesincludebutarenotlimitedtoatopic
dermatitis,nummulareczema,primaryirritantcontactdermatitis,allergiccontactdermatitis,
seborrheicdermatitis,anddryskindermatitis(xerosis).
Atopicdermatitisischronicsuperficialinflammationofskin.Itmayhavethreeclinical
phases(infantileeczema,childhoodorflexuraleczema,adolescenteczema).Treatmentconsists
ofavoidingirritants,keepingtheskinmoist,andtopicalcorticosteroidsorimmunosuppressive
agents.
Nummulareczemapresentswithnumeroussymmetric,coinshapedpatchesofdermatitison
theextremities.Treatmentissimilartoatopicdermatitis.
Primaryirritantcontactdermatitisappearswithinhoursofcontact,peaksat24hours,and
thendisappears.Themostcommonformisdiaperdermatitis,causedbyprolongedcontactof
theskinwithurineandfeces.Treatmentistoincreasethefrequencyofdiaperchangesand
washwithwaterafterabowelmovement.
Allergiccontactdermatitisisadelayedtypehypersensitivityreactiontooffendingagent.

Thereisdelayedonsetaround18hours,whichthenpeaksat4872hoursandmaylastaslong
as23weeksevenaftercessationofcontactwiththeallergen.Nickel,latex,andpoisonivyare
verycommoncausesofallergiccontactdermatitis.Allergiccontactdermatitiscommonly
causesblisterformationwithoozingandcrusting,ofteninthepatternofallergenexposure.
Treatmentmainlyinvolvesavoidanceoftriggers.
Seborrheicdermatitisiserythematous,scalydermatitisandhighsebumproductioninareas
containingahighdensityofsebaceousglands(face,scalp,andperineum).Firstlinetreatment
involvestopicalantifungalagents(suchasketoconazole)resistantcasescanbetreatedwith
topicalcorticosteroids.
Dryskindermatitis(xerosis)presentswithlargecrackedscaleswitherythematousborders
andoccursinaridenvironments.Treatmentisaimedatincreasingwatercontentofskinby
minimizingbathing,avoidingsoaps,andfrequentlyusingemollients.
Notes

103.A32yearoldmanpresentswitharedbrownmaculopapularrashoverhistrunkand
extremities.Therashispresentoverhispalmsandsoles.Hehashadafeverandheadacheover
thepastweek.Hecurrentlytakesnomedicationsandhasneverhadanadversereactionto
medications.Aboutamonthago,heremembershavingafirm,painlessskinulcerationonthe
shaftofhispenis.Hedeniesotherskinlesions.Whatisthefirstlinetreatmentforthispatient?
A.OnedoseofIMbenzathinepenicillinG(2.4millionunits)
B.IMbenzathinepenicillinG(2.4millionunits)givenonceaweekfor3weeks

C.Doxycycline100mgbidfor2weeks
D.Onedoseofazithromycin2gtakenorally
E.Tetracycline500mgtakenorallyfourtimesdailyfor2weeks
104.A28yearoldmanattherefugeeclinicisconcernedaboutasingle,firm,nonitchy,
painlessskinulcerationwithacleanbaseandsharpbordersthathasbeenonhispenisforthe
past3weeks.Headmitstohavingunprotectedsex2monthsago.Ifleftuntreated,whatisthe
mostlikelylongtermtertiarymanifestationofthisdisease?
A.InfectioninvolvingtheCNS,suchasgeneralparesisortabesdorsalis
B.Syphiliticaortitisandpossibleaneurysmformation
C.SaddlenosedeformationandCluttonjoints
D.Soft,tumorlikeballsofinflammationwhichmayaffecttheskin,liver,andbone
E.Reddishpink,nonitchyrashthatoccursonthetrunk,arms,palms,andsoles
103.TheanswerisA:OnedoseofIMbenzathinepenicillinG(2.4millionunits).
Thispatienthasarashcharacteristicofsecondarysyphilis.Becauseherecallshavingaprimary
lesion1monthago,thedurationofillnessisknown.Thispatientshouldbetreatedwithone
doseofIMbenzathinepenicillinG2.4millionunits(A).OnedoseofIMpenicillinGisgiven
onceweeklyfor3weeks(B)totreattertiarysyphilisorsecondarysyphilisthathasbeen
presentformorethan1year.Ifthepatientispenicillinallergic,doxycycline100mgbidfor2
weeks(C)ortetracycline500mgfourtimesdailyfor2weeks(E)canbeusedasalternatives.
Thereislimiteddatatosupportaonetimedoseofazithromycin2g(D),andthisisnotfirst
linetreatment.
104.TheanswerisD:Soft,tumorlikeballsofinflammationwhichmayaffectthe
skin,liver,andbone.Thispatientpresentswithclassicsymptomsofprimarysyphilis.Ifleft
untreated,thiswillprogresstosecondarysyphilis410weekslater,manifestingasanonitchy
redrashonthetrunkandextremities,includingthepalmsandsoles(D).Thisrashwilllikely
resolvein36weeks.Tertiarysyphilisisthelongtermcomplicationthatmayoccurdueto
syphilisandcanbedividedintothreeforms:lateneurosyphilis(A)affects6.5%,gummatous
syphilisaffectsthemajority(15%),andcardiovascularsyphilis(10%).Saddlenosedeformation
andCluttonjoints(C)aremanifestationsofcongenitalsyphilis.

Syphilis(ChancreandSecondary)

SyphilisisasexuallytransmitteddiseasecausedbythespirocheteTreponemapallidum.Syphilis
cangenerallybetreatedwithantibiotics,includingpenicillin.Ifuntreated,syphiliscandamagethe
heart,aorta,bones,eyes,andbrain.Primarysyphilisistypicallyacquiredbydirectsexualcontact.
Approximately26weeksaftertheinitialexposure,askinlesionappearsatthepointofcontact,
whichisusuallythegenitalia.Thislesion,calledachancre,isafirm,painlessskinulceration.The
lesionmaypersistfor46weeksandusuallyhealsspontaneously.Secondarysyphilisoccurs16
monthsaftertheprimaryinfection(average68weeks).Therearemanymanifestationsofsecondary
syphilis.Aredorbrownmaculopapularrashmaydevelopoverthetrunkandextremitiesandmay
involvedthepalmsandsoles.Othersymptomsmayincludefever,malaise,weightloss,headache,or
enlargedlymphnodes.Secondarysyphilisalsocommonlyinvolvesthemucousmembranes.Latent
syphilisispresentwhenthereisserologicevidenceofsyphilis,butthepatientdoesnothavephysical
signsofthedisease.Lateortertiarysyphilisinvolvesthecardiovascularsystem,CNS,andocular
system.Gummatouslesionsmayinvolvetheskin,bones,orviscera.
SerologicscreeningtestsincludeRPRandvenerealdiseaseresearchlaboratory(VDRL).All
positivetestsshouldbeconfirmedwithtreponemalantibodytest(FTAABS).
Treatmentforearlydisease(primaryandsecondaryforlessthan1year)isonedose
benzathinepenicillinG.Forlatedisease,benzathinepenicillinGisgivenonceaweekfor3
weeks.
Darkfieldmicroscopicexaminationcanbeusedtovisualizespirochetes.
Notes

105.A30yearoldwomanpresentswithfeverandpainfulmucocutaneouslesions.Shenotes
thatshehasnomedicalconditions.Herprimarycaredoctorstartedheronasulfadrugfora
bacterialinfection4daysago.Younotesloughingoftheepidermisonphysicalexam.Whatis
themostappropriatetreatment?
A.Stopsulfonamideantibioticimmediately
B.Administeranalgesics
C.StartIVfluids
D.InitiateNGorparenteralfeeding
E.Alloftheabove
106.A28yearoldwomanwithahistoryofepilepsypresentstoyourofficecomplainingof
fever,sorethroat,andfatiguefor5days'duration.Shecomestoyoutodaybecauseshebeganto
noticeulcersinhermouthandlips(shownabove).Theseulcersareextremelypainful,andshe
isbeginningtohavedifficultyeatinganddrinking.Youstopthemediationandstartsupportive
therapy.WhichofthefollowingfindingswouldsupportadiagnosisofSJS?
A.Morethan30%BSAepidermaldetachment
B.Involvementofthehairbearingscalp
C.Welldemarcatedplaquesofhypopigmentationthatarecooltothetouch
D.PresenceofNikolskysign
105.TheanswerisE:Alloftheabove.Thiswomanissufferingaseveredrug
hypersensitivityreaction.Theinvolvementofskinandmucousmembranesisconsistentwith
SJS.Alloftheabove(E)arenecessarytreatmentstepsforSJS.Allmedicationsshouldbe
stoppedimmediately(A),especiallythoseknowntocauseSJSreactions(suchas
sulfonamides,amoxicillin,penicillin,andallopurinol).Supportivecare,suchasanalgesics(B),
IVfluids(C)andNGorparenteralfeeding(D),shouldalsobeinitiated.

106.TheanswerisD:Welldemarcatedplaquesofhypopigmentationthatarecool
tothetouch.LamotrigineistraditionallyknowntoleadtoSJS.Othermedicationsinclude
sulfonamides,penicillins,phenytoin,andbarbiturates.PresenceofNikolskysigniscommonin
SJSandoftenhelpstodifferentiatetheconditionfromfixeddrugeruptions(D).Fixeddrug
eruptionsappear,710daysafterexposure,involveadefinedareaofhypopigmentation,and
arerarelyhottothetouch.Involvementof30%ormoreoftheskinissuggestiveofTEN.
InvolvementofthehairbearingscalpisrareinSJS(B).

StevensJohnsonSyndrome(SJS)

StevensJohnsonsyndrome(SJS)isalifethreateningconditioncharacterizedbyfeverand
mucocutaneouslesionsleadingtonecrosisandsloughingoftheepidermis.Thesyndromeaffectsthe
skinandthemucousmembranesandisthoughttobeahypersensitivitycomplex.SJSislesssevere
thanTENinthatalesserpercentofthebodysurfaceisinvolved(definedaslessthan10%to30%of
thebodysurfaceisinvolved).Thissyndromeismostcommonlytriggeredbymedications,butitcan
alsobecausedbyinfections,cancers,ormaybeidiopathic.SJSisanemergency,andallmedications
shouldbeimmediatelydiscontinued,particularlythoseknowntocauseSJSreactions.Initially
treatmentissimilartothatforpatientswiththermalburns(IVfluids,NGorparenteralfeeding,
analgesics,etc).
Affectsskinandmucousmembranes
Mucousmembranesareaffectedin92%to100%ofpatients.
Stopmedicationsimmediately.

Treatwithsupportivemeasures
Notes

107.A7yearoldgirlisbroughttothepediatricianbyhermother.Shenotesthatthepatient
hasbeenquiteillwithafever,cough,andrunnynoseforthepast5days.Shenoticedtodaythat
arashwasdevelopingonherheadandface.Hermotheralsonoticedsmallwhitespotsonher
oralmucosawhileshewashelpingherbrushherteethacoupleofdaysago.Onphysical
examination,younotethatthegirlhasconjunctivitisandamaculopapularrashoverherupper
body.Whatisthecauseofthisgirl'sillness?
A.Rubellavirus
B.HHV6
C.Varicellazostervirus
D.Paramyxovirus
E.ParvovirusB19
Notes

108.A65yearoldmanborninPolandwithanunknownmedicalhistorypresentsforthefirst
timetohisprimarycarephysicianinArizonacomplainingofa3dayhistoryofcough,feverof
101103F,runnynose,andpinkeye.Onphysicalexam,thepatient'srightconjunctivais
injected,andhiseyelidisencrustedwithyellowmucus.Healsohasrhinorrheaandonoral
examisfoundtohavewhitelesionswithanerythematoushaloonhisbuccalmucosabutno
tonsilarerythemaoredema.Rapidstrep,influenza,andmonospottestsarenegative.On
furtherquestioning,thepatienthadrecentlybeenexposedhisgranddaughterwhohereports
hadarashonherfaceandchest.Whatisthemostlikelydiagnosisofthispatient's
constellationofsymptoms?
A.Influenza
B.Scarletfever
C.Measles
D.Infectiousmononucleosis
E.Toxoplasmosis
107.TheanswerisD:Paramyxovirus.Thisgirlispresentingwithmeasles(rubeola)which
iscausedbyaparamyxovirus(D).Classicsymptomsincludefever,cough,coryza,and
conjunctivitis.Inaddition,thismothernoticedwhitespotsonthegirl'sbuccalmucosa,possibly
representingKoplikspots,whicharepathognomonicforthediseaseandareseenbeforethe
rash.Thetypicalrashofmeaslesiserythematousandmaculopapular,startingontheheadand
spreadingtothebody.Rubellavirus(A)causeslymphadenopathyandanerythematousrash
thatstartsonthefaceandspreadsdistally.Incontrasttomeasles,childrenwithrubellaoften
arenotasillanddonotpresentwithcough,coryza,conjunctivitis,orKoplikspots.HHV6
causesroseolainfection.Thesepatientspresentwithanacuteonsetofahighfever,whichlasts
34days.Asthefeverbreaks,amaculopapularrashstartsonthetrunkbeforespreadingtothe
faceandextremities.Varicella(C)causesgeneralizedpruriticvesicularlesionsthatareoften
visualizedatdifferentstagesofhealing.ParvovirusB19(E)causesfifthdisease(erythema

infectiosum).Fifthdiseasecausesaslappedcheekerythematousrash.Amaculopapular,
erythematousrashcanbeseen,startingonthearmsandspreadingtothelegsandtrunk.
108.TheanswerisC:Measles.Giventhispatient'ssignsandsymptoms,heismostlikely
presentingwithacaseofadultonsetmeasles(C),causedbyaparamyxovirusinfection.
Measlesbeginwitha2to4dayrespiratoryprodromeofmalaise,cough,coryza,conjunctivitis,
rhinorrhea,andhighfevers.OftenpatientswillalsodemonstrateKoplikspots(1to2mm,
bluewhitespotsonaredbackground)onthebuccalmucosajustbeforetheonsetofan
erythematous,nonpruritic,maculopapularrashbeginningontheheadandspreadingdownthe
trunkandarmstoincludethepalmsandsoles.AlthoughthisdiseaseisrareintheUnited
Statesandwasdeclaredeliminatedin2002,recentoutbreaksarebecomingincreasingly
common.In2008,140confirmedcaseswerereportedtotheCDC,and25%ofthosewere
foundinadults.Twentytwoofthe140totalcaseswerereportedinArizona.Theseoutbreaks
primarilyaffectedthosewhohadnotbeenvaccinatedwiththemeasles,mumps,andrubella
(MMR)vaccine,dueeithertocasesimportedfromothercountriesortoarecentresistanceof
someparentstovaccinatetheirchildrenduetoscientificallyunfoundedfearsofanassociation
betweenthevaccineandautism.BecausethispatientwasbornoutsideoftheUnitedStates,it
islikelythatheneverreceivedtheMMRvaccineandthuswassusceptibletobecominginfected
fromhisgrandchild.Influenza(A)maypresentwithacough,fever,andrhinorrheabut
generallydoesnotcauseconjunctivitis.Giventhispatient'snegativerapidflutest,thisisnot
themostlikelydiagnosis.Scarletfever(B)presentsasasandpaperyrashoccurringonthe
neckandchestfollowinginfectionwithgroupAstreptococci.Thepatient'sconstellationof
symptomsarenotseenwithgroupAstrepinfections,andgivenhisnegativerapidstreptest,
thisisnotthediagnosis.Infectiousmononucleosis(D)iscausedbytheEBV.Itmaypresent
withfever,sorethroat,malaise,anorexia,andmyalgia,alongwiththephysicalfindingsof
lymphadenopathy,splenomegaly,and,lesscommonly,amaculopapularrash.Althoughthe
virusmayoccasionallycauseexudativepharyngitisortonsillitis,Koplikspotsarenotseen.The
virusdoeshavethepotentialtocauseconjunctivalhemorrhageintheeyebutdoesnotpresent
withconjunctivitislikethatseeninmeaslespatients.Toxoplasmosis(E)inimmunocompetent
patientspresentswithfew,ifany,symptoms.TheprotozoanToxoplasmagondiimay
infrequentlyleadtomildlymphadenopathy,lowgradefevers,andsorethroat.Chorioretinitis
mayresultfromreactivationofcongenitaltoxoplasmosis,butconjunctivitisisnotseen.

Measles

Measles,alsocalledrubeola,isaninfectionoftherespiratorysystemcausedbyaparamyxovirus.
Theviralinfectionproducesanerythematousmaculopapularrash.Therasherupts5daysafteronset
ofprodromalsymptoms.Typicallytherashbeginsontheheadandthenspreadstothebody,lasting
45days.Koplikspotsarewhitespotsonthebuccalmucosa,whichresolvepriortotheappearanceof
therash.Koplikspotsarepathognomonicforthediseasewhentheyareseen.Typicalassociated
symptomsincludefeverandtheThreeCs(cough,coryza,conjunctivitis).Commoncomplications
includeotitismedia,pneumonia,andlaryngotracheitis.Ararecomplicationofmeaslesissubacute
sclerosingpanencephalitis.
Diagnosis:feverandtheThreeCs(cough,coryza,conjunctivitis)
ChildrenarevaccinatedformeaslesaspartoftheMMRvaccine.
Spreadthroughrespiratoryfluids,eitherviadirectcontactorthroughaerosoltransmission
Notes

109.A26yearoldwomanhashadtheabovelarge,flatlesiononherbacksincebirth.Sheis
concernedabouttheriskofdevelopingmelanoma,fromwhichacolleagueatworkjustdied.

Shewouldliketoknowifthelesionshouldberemoved.Shehasfewerthan20nevomelanocytic
nevionherbody,andthereisnohistoryofskincancerinherfamily.Howshouldshebe
counseledregardingthislesion?
A.Thereisahighriskofprogressiontomelanoma,andthelesionshouldberemoved.
B.Thereisamildriskofprogressiontomelanoma,sothelesionshouldbeexamined
every6months.
C.Thelesionisbenign,andexcisioncancausedisfigurationtherefore,excisionisnot
advisable.
D.Thelesionmustbebiopsiedbeforeatreatmentdecisioncanbemade.
E.Thelesionismalignantandhasapoorprognosis.
110.A20yearoldwomanwithblueeyespresentstotheofficeforanannualvisit.Shehasno
complaintsandnosignificantpastmedicalhistory.Shespendshersummersawayfromcollege
atthebeach,wheresheworksasaswimmingteacher.Herfamilyhistoryissignificantforher
maternalgrandmotherhavingbasalcellcarcinomaofhernose.Withfurtherquestioning,she
admitstooccasionalsunburns.Totalbodyskinexamrevealsmultiplemelanocyticnevionher
trunkandarms,numberingover50,whichshestateshavebeenthereforsometime.Noneof
thenevimeasureover5mmindiameter.Theneviallhavearoundorovalshapewithsharp
bordersandlightbrownpigmentation.Whichofthefollowingstatementsaboutthispatientis
true?
A.ThedevelopmentofbenignneviisstimulatedbyUVradiationexposure.
B.Thenumberofbenignnevionthispatientplacesherathigherriskfordeveloping
melanoma.
C.Growthalonemaynotbesufficientreasontoexciseanevus.
D.Obtainingbaselinephotographicdocumentationofthispatient'snevicanhelpguide
futuretreatmentstrategies.
E.Alloftheabove
109.TheanswerisC:Thelesionisbenign,andexcisioncancausedisfiguration
therefore,excisionisnotadvisable.Thisisacongenitalnevus,whichisabenign
proliferationofnormalskinconstituents.Itispresentfrombirthandarisesfromcellsdeepin
thesubcutaneousfattherefore,excisionisdifficultandcancausesignificantdisfiguration(C
).Thereisalowriskofprogressiontomalignancy(A,B,E).Shehasfewerthan50neviand
nofamilyhistoryofskincancertherefore,sheisnotatincreasedriskofdevelopingmelanoma.
Thisisaclinicaldiagnosis,andbiopsyisnotnecessary(D).
110.TheanswerisE:Alloftheabove.Alloftheabovestatementsarecorrect(E).This

patientisafaireyedindividualwhohasaconsiderableamountofsunexposureduetoher
occupationandhashadsundamageasevidencedbyherhistoryofsunburns.Herskinfindings
areconsistentwiththepresenceofmultiplebenignnevi,alsoknownasacquiredmelanocytic
nevi.Thispatientisatriskforsubsequentdevelopmentofmelanomaandnonmelanomaskin
cancerbecauseofherhighcumulativeUVradiationexposureandfamilyhistoryofcutaneous
malignancy,aswellasthenumberofbenignnevipresentonherskin(>50).Thus,sheshould
beassessedwithtotalbodyskinexaminationsduringherannualofficevisit.Photographic
documentationofherfindingscanhelptracktheskinfindingsovertime.Becausebenignnevi
areoftenstillindevelopmentatherage,somecanbeexpectedtogrowandmaynotnecessarily
needexcisionintheabsenceofotherconcerningfeatures,whichmayincludeirregularandill
definedborders,eccentricpigmentpatterns,andrapidgrowth.Dermatoscopycanaidin
trackinglesionsovertimeandmakingdecisionsontreatmentstrategies.

BenignNevus

Abenignnevusdevelopsslowlyafterbirth,usuallyduringchildhoodoryoungadulthood,andgrows
symmetricallybeforestabilizingatacertainsize.Somemayregressafteraperiodoftime.
EnvironmentalexposuretoUVradiationseemstobetheincitingfactorforthedevelopmentof
benignnevi.Geneticfactorsplayaroleaswell,andthereisanassociationwithpaleskin,blueor
greeneyes,blondorlightbrownhair,andatendencytosunburn.
Developafterbirthandaretypicallysmallerthan5mm
Flatorraisedsymmetric,smooth,welldefinedborderuniformpigmentation
Higherlifetimeriskofmelanomainpatientswithmorethan50benignnevi

Excisionisdifficultanddisfiguringand,therefore,usuallynotperformed.
Notes

111.The45yearoldwomaninthepictureabovepresentstoyourofficecomplainingof
blanchingofherfingers,extendingfromthetipstovariouslevelsofthedigitswithsparingof
thethumbwheneversheiscoldorstressedout.Sheisotherwisehealthyandhasanegative
reviewofsystems.Whichofthefollowingstatementsaretrueinregardtohercondition?
A.Itisusuallyseeninoldermalepatients.
B.Sheisathighriskfordevelopinganautoimmunedisease,evenifshedoesnotdevelop
anyadditionalsymptomsformorethan2years.
C.Drugtherapyusingcalciumchannelblockers(CCBs)canbeusedforseverecases.
D.Smokingcessationdoesnotimprovesymptoms.
E.Itismostcommonlyassociatedwithankylosingspondylitis.
112.A35yearoldwomanpresentstoyourofficecomplainingofpainandtinglinginher
fingersandtoes.SherecentlytookatriptoQuebectoiceclimbanddescribesaparticular
episodewhereshenoticedthispainandtinglingandremovedherglovestofindthatherfingers
haddevelopedastartlingbluehueandthenturnedwhite.Yourecommendthatsheavoid
unnecessarycoldenvironmentsanddresswarmly,takingcaretoprotecthertrunkaswellas
herextremities.Inaddition,yourecommendthatshestopsmoking,astobaccocanexacerbate
constrictionofherbloodvessels.Despitehercompliance,hersymptomspersist.Shereturnsto
youdesperateforamedicationtohelpalleviatehersymptoms.Youprescribeamedicationto
reducethefrequencyandseverityofherattacks.Whichofthefollowingisthemechanismof
actionofthemedication?
A.Blockvoltagegatedcalciumchannelsinbloodvessels,decreasingintracellular
calcium,resultingindecreasedcontractionofthevascularsmoothmuscleandincreasein
arterialdiameter

B.BlocktheconversionofangiotensinItoangiotensinII,therebyloweringarteriolar
resistanceandincreasingvenouscapacity
C.Penetratethevascularendotheliumandcausenitrousoxideproduction,which
activatescyclicguanosinemonophosphate,ultimatelyleadingtovenodilation
D.Increasereabsorptionofcalciuminthedistalconvolutedtubuleofthekidneyand
decreasesodiumtransport,leadingtonatriuresisandwaterloss
E.Reducedopaminergicandadrenergictransmissionintheperipheralnervoussystem,
loweringbloodpressureandsympatheticnervoussystemoutput
111.TheanswerisC:Drugtherapyusingcalciumchannelblockers(CCBs)canbe
usedforseverecases.ThispatienthasRP,anabnormalconstrictionofthebloodvesselsin
responsetocold,mentalstress,orsmoking.Itisseenin10%20%ofyoungwomen(A),and
thesequenceofcolorchangesincludepallor(white)withsharpdemarcationasabove,then
cyanosis(blue)becauseofslowbloodflow,followedbyerythema(red)reflectingthereactive
hyperemicphase.RP(secondaryRaynaud)shouldbedistinguishedfromRaynauddisease
(primaryRaynaud).RPisassociatedwithscleroderma(mostcommon,with90%prevalence)(
E),DM,mixedconnectivetissuedisease,RA,SLE,Sjgren,cryoglobulinemia,andvasculitides
likepolyarteritis.YoungfemalepatientswhohavehadRPaloneformorethan2yearsandhave
notdevelopedanyadditionalmanifestationsareatlowriskfordevelopinganautoimmune
disease(B)andwouldhaveprimaryRDinstead.Smokingcessationismandatoryduetothe
vasoconstrictiveeffects(D),anddrugtherapyusingCCBsshouldbereservedforseverecases
only(C).
112.TheanswerisA:Blockvoltagegatedcalciumchannelsinbloodvessels,
decreasingintracellularcalcium,resultingindecreasedcontractionofthe
vascularsmoothmuscleandincreaseinarterialdiameter.CCBssuchasnifedipine
workbydecreasingintracellularcalcium,resultinginincreasedbloodflowtoischemic
extremities.ThesemedicationsarecommonlyprescribedtotreatsevereRP.ACEinhibitors(B
)andthiazidediuretics(D)arecommonlyusedtotreathypertensionandhavelittleusein
alleviatingthesymptomsassociatedwithRP.Nitrates(C)andmethyldopa(E)canbeusedto
treatRP,butonlyinseverecasesthatarerefractorytoCCBtreatmentorwhereCCBsare
otherwisecontraindicatedorpoorlytolerated.

RaynaudPhenomenon

Raynaudphenomenon(RP)isaconditionresultinginaseriesofdiscolorationsofthefingersand/or
thetoesafterexposuretochangesintemperature.Skindiscolorationoccursbecauseanabnormal
spasmofthebloodvesselscausesadecreasedbloodsupplytothelocaltissues.Initiallythedigit(s)
involvedturnswhitebecauseofthediminishedbloodsupply.Thedigit(s)thenturnsbluebecauseof
prolongedlackofoxygen.Finallythebloodvesselsreopen,causingalocalflushingphenomenon,
whichturnsthedigit(s)red.
Morecommoninwomen
MaybepartofCRESTsyndrome
CCBsforsymptomcontrol
Mostcommonlyfoundinfingers,toes,andotheracralareassuchasthenoseandears
Notes

113.A42yearoldwomanpresentswithrepeatedepisodesofadroopingrighteyelid,which
becomesmorenoticeableintheevenings.Shestatesthatsheisingoodhealthbuthasbeen
experiencingtransientepisodesofeyeliddroopinganddoublevision.Shealsohasexperienced
intermittentdifficultywithprolongedchewingandswallowing.Whatisthemechanismcausing
thispatient'ssymptoms?

A.Antibodiestopresynapticvoltagedependentcalciumchannels
B.Antibodiestopostsynapticacetylcholinereceptors
C.Toxin
D.Degenerationofmotorneurons
E.Viralinfection
114.A32yearoldwomanpresentsduetoepisodesofdroopingofherrighteyelidandblurry
visionthatseemtobeworselaterintheday.Shehasnosignificantmedicalorfamilyhistory
andtakesnomedications.Physicalexamissignificantforrightsidedptosisanddiplopiaas
wellasprogressiveweaknesswithprolongedarmabduction.Alaboratorytestshowsthe
presenceofantiacetylcholinereceptor(antiAChR)antibodiesinthepatient'sserum.Allof
thefollowingadditionalscreeningtestswouldbeindicatedinthispatientexcept?
A.CTscanoftheanteriormediastinum
B.TSHandT3/T4levels
C.ANA
D.Fineneedleaspiration(FNA)ofthethyroid
E.Rheumatoidfactor
113.TheanswerisB:Antibodiestopostsynapticacetylcholinereceptors.This
patientisexperiencingsymptomsconsistentwithmyastheniagravis(MG),includingtransient
ocularandbulbarmusclefatiguewhichworsensasthedayprogresses.MGiscausedby
antibodiestopostsynapticacetylcholinereceptors(B).Antibodiestopresynapticvoltage
dependentcalciumchannels(A)arefoundinLambertEatonsyndrome.Theclassic
presentationofLambertEatonsyndromeincludesproximallegweakness,autonomic
dysfunction,andptosis,butrarelyincludesinvolvementofthebulbarmuscles.Symptomsin
LambertEatonareworseinthemorningandimprovewithexercise.Approximatelyhalfof
LambertEatonpatientshavecancer,usuallyofthesmallcelllungtype.Atoxinmediated
process(C),suchasbotulism,maycausebulbarandeyemuscleinvolvement.However,
pupillaryparalysisiscommon,andthereisoftenrapidprogressionfollowingingestionoffood
contaminatedbyClostridiumbotulinum.MGtypicallysparespupillarymuscles.Degeneration
ofmotorneurons(D),asseeninamyotrophiclateralsclerosis(ALS),typicallydoesnot
produceptosisordiplopia.ALSmaybeassociatedwithbulbarmuscleweakness.Patientshave
findingsofuppermotorneuronandlowermotorneuroninvolvement.Bellpalsy,whichmaybe
causedbyaviralinfection,causesfacialpalsy.Thehistoryofmultipletransientepisodesof
ptosismakesBellpalsylesslikelyinthiscase.
114.TheanswerisA:CTscanoftheanteriormediastinum.Thispatientpresentswith

severalofthesignsandsymptomsofMG,aconditioncausedbyautoantibodiestopostsynaptic
acetylcholinereceptors.ThepresenceofantiAChRantibodiesinapatient'sserumisdiagnostic
ofthisdisease.AntiAchRantibodiesaredetectableintheserumofcloseto85%ofmyasthenic
patientsbutinonlyabout50%ofpatientswithweaknessconfinedtotheocularmuscles,which
isseenearlierinthediseaseprocess.Thesepatientshaveanincreasedincidenceofseveral
associateddisorders.Seventyfivepercentofpatientsexhibitthymicabnormalities,andmany
mayexhibitenlargementofthethymusduetoneoplasticchanges,andthereforethesepatients
shouldbescreenedwithaCTscanoftheanteriormediastinum(A).Hyperthyroidismmaybe
seenin3%8%ofpatientsandmayexacerbatetheirweakness,sothyroidfunctiontests(B)
shouldbeobtainedinallpatientswithMG.Thereisalsoanassociationwithotherautoimmune
disorders,andpatientswithMGshouldhavebloodtestsforrheumatoidfactor(E)andANA(
C).FNA(D)ofthethyroidisnotindicatedforpatientswithMG.

MyastheniaGravis

Myastheniagravis(MG)isanautoimmuneneuromusculardiseasecharacterizedbyfluctuating
weaknessandfatigabilityinocular,bulbar,limb,andrespiratorymusclesthatincreaseswith
repeateduse.WeaknessistheresultofTcelldependent,antibodymediatedattackonpostsynaptic
acetylcholinereceptorsoftheneuromuscularjunction.Patientspresentwithptosis,ocularpalsies,
andproximalmuscleweakness,whichworsensasthedayprogresses.
Physicalexam,repetitivenervestimulationtest,andedrophonium(Tensilon)testcanbe
performedifdiagnosisissuspected.Tensilonisashortactingcholinesteraseinhibitorthat
allowsfornormalsynapticfunctioninaffectedmuscles.
Amorespecifictesttodetectautoantibodiesshouldbedonetoconfirmdiagnosis.
ScreenforthymomaswithachestCT.
Treatmedicallywithcholinesteraseinhibitors(neostigmine)orimmunosuppressants.
Thymectomymaybeindicatedinsomecases.
Duringcrisis,intubationmaybeneededforrespiratorysupport,alongwithplasmapheresis.

ReferencesBoardImages

FauciA.S.,BraunwaldE.,KasperD.L.,HauserS.L.,LongoD.L.,JamesonJ.L.,et.al.:17thed.March
6,2008.McGrawHillProfessional
GrimesP.E.:Melasma.Etiologicandtherapeuticconsiderations.ArchDermatol1995,December
131:pp.14531457.
HallJ.B.,SchmidtG.A.,WoodL.D.H.:3rded.July2005.McGrawHillProfessional
HigginsS.P.,FreemarkM,ProseN.S:Acanthosisnigricans:apracticalapproachtoevaluationand
management.DermatolOnlineJ2008,September1514:pp.2.
KnoopK.J.,StackL.B.,StorrowA.B.,ThurmanR.J.:3rded.July2002.McGrawHillProfessional
McPhee,S.J.,&Papadakis,M.A.(2011).LangeMedicalBooks/McGrawHill,2010.
RutterM:Incidenceofautismspectrumdisorders:changesovertimeandtheirmeaning.Acta
Paediatr200594:pp.215.
TintinalliJ.E.,StapczynskiJ.S.,ClineD.M.,MaO.J.,CydulkaR.K.,MecklerG.D.:(7thed.).October
14,2003.McGrawHillProfessional
WolffK.,GoldsmithL.A.,KatzS.I.,GilchrestB.,PallerA.S.,LeffellD.J.:(7thed.)October17,2007.
McGrawHillProfessional
WolffK.,JohnsonR.A.:Fitzpatrick'sColoratlasandsynopsisofclinicaldermatology.6thed.2009.
McGrawHillNewYork

Copyright2015Elsevier,Inc.Allrightsreserved.

BOOKCHAPTER

Cardiology
AmberS.TullyMDandJamesS.StuddifordMD,FACP
USMLEImagesfortheBoards,SectionII,133133

Notes

1.A30yearoldwomanpresentstoherprimarycarephysicianwithcomplaintsofoccasional
palpitationsoccurringoverthelast10years.Shecurrentlyisasymptomaticandhasno
significantpastmedical,family,orsocialhistory.Theelectrocardiogram(ECG)shows:
A.Atrialtachycardiawithinferiorinfarct,ageindeterminate
B.Atrialtachycardia
C.Atrioventricularreentranttachycardia
D.Sinustachycardiawithinferiorinfarct,ageindeterminate
E.SinustachycardiawithborderlinepoorRwaveprogression

1.TheanswerisE:SinustachycardiawithborderlinepoorRwaveprogression.PoorRwave
progressionisdefinedastheabsenceoftransitionfromanegativetopositiveQRScomplexin
theprecordialleadsbyleadV4.BorderlinepoorRwaveprogressioncanbedefinedas
transitionappropriatelyoccurringbyleadV4,butinappropriatelydelayedpositivevoltage
transitionfromV1toV3,asinthiscase.PoorRwaveprogressioncanbefoundinpatientswith
anteriormyocardialinfarction,leftventricularhypertrophy,andinnormalindividuals.
Sometimesitoccursduetoleadmisplacement,whichismostlikelythecaseinthispatient
giventheclinicalhistory.Infact,thispatient'sechocardiogramrevealednormalwallmotion
andoverallfunction.
A,B.Thisisnotatrialtachycardia,becausethePwaveshaveanaxisconsistentwithasinusnode
origininthehighrightatrium(positivePwavesinleadII).Rarelyanatrialtachycardiacouldbe
comingfromnearthesinusnodeandbeindistinguishablefromsinustachycardia.Inthissituation,
seeingtheonsetoroffsetofthetachycardiawoulddistinguishsinustachycardiafromatrial
tachycardiasinustachycardiabeginsandendsslowly,whereasatrialtachycardiabeginsandends
suddenly.
C.WhentherearenarrowQRScomplexes,atrioventricularreentranttachycardia(AVRT)would
haveretrograde(invertedinleadII)Pwaveswithadifferentaxis.AVRTinvolvesconductionthrough
anaccessorybypasstractaspartofthereentrycircuit.
D.ThisECGisnotconsistentwithaninferiorinfarct.PathologicQwavesaredefinedashavinga
durationgreaterthan30ms(~1smallbox)anddeeperthan0.1mV(>1smallbox)intwocontiguous
leads.ThoughtheQwaveinleadIIImeetspartofthecriteria,thesmallQwaveinleadaVFdoesnot.

ManypatientswillhaveanisolatedQwaveinleadIII,andthisisnotpathologic.
Notes

2.Thesame30yearoldwomanpresentstotheemergencydepartment1monthlaterwiththe
suddenonsetofpalpitationsandmildlightheadedness.Shehadexperiencedbrieferepisodes
overthelast10years,butthisepisodepersistedfor5minutes.Therehasbeennointerval
changeinhernonsignificantpastmedical,family,orsocialhistory.Herphysicalexamis
normalexceptfortachycardia.TheECGshows:
A.Sinustachycardia
B.Ventriculartachycardia(VT)
C.Atrialflutter
D.Atrioventricularnodalreentranttachycardia(AVNRT)
E.WolffParkinsonWhitesyndrome

2.TheanswerisD:Atrioventricularnodalreentranttachycardia(AVNRT).ThisECGshows
AVNRT(alsoknownasatrioventricularnodalreciprocatingtachycardia),andpoorRwave
progression.TheQwavesinleadsIIIandaVFarenotpresentinleadIIandthereforedonot
reflectaninfarctpattern.AVNRTisasupraventriculartachycardiathatcommonlyoccursin
adultswithnostructuralheartdisease,mostcommonlyfemalesinthethirdtofourthdecadeof
life.Itpresentswithpalpitationsanddizziness,sometimeswithdyspneaandchestpain,and
rarelysyncope.Ratesaretypicallybetween140250bpm.AVNRTisareentrantarrhythmia
involvingtheAVnodeandperinodaltissuewithanterogradeconductioninaslowpathwayand
retrogradeupafastpathway,causingnearsimultaneousactivationoftheatriaandventricles.
Asaresult,theremaybeainvertedPwavenoted,mostcommonlyimmediatelyfollowingthe
QRScomplexintheformofapseudoSwave,asseeninthistracingintheinferiorandleft
precordialleads(thoughinsomecasesthisPwavewillbeburiedintheQRScomplexor,less
commonly,immediatelybeforetheQRS).ItishelpfultohaveabaselineECGforcomparison,
atwhichpointtheretrogradePwavesbecomeclearlyevident(seequestion1,whichdepictsthis
patient'sbaselineECGwhichlackstheretrogradePwaves).
A.ThisECGisnotsinustachycardiabecauseofthelackofuprightPwavesinleadIIprecedingthe
QRScomplex.
B.TheQRScomplexesarenarrowandthereforenotVT.
C.Therateofabout150shouldpromptthethoughtofatrialflutterwith2:1conduction,butthereare
noflutterwavespresent.
E.WolffParkinsonWhitesyndromeisthecombinationofpreexcitation(exhibitedbyashortPR
intervalandadeltawaveduetothefasterconductionovertheaccessorypathwayfusingwithnormal
conductionthroughtheAVnode),andarrhythmiassuchasparoxysmalatrioventricularreentrant
tachycardia(alsoknownasatrioventricularreciprocatingtachycardia).Thiscaselacksadelta
waveonthebaselineECGseeninquestion1.
Notes

3.A63yearoldmanpresentstoyourofficewithcomplaintsofintermittentlightheadedness
withoutsyncope.Hehastype2diabetic,ishypertensive,andhasgoutbuthasnoknown
cardiacdisease.HisECGshows:
A.Sinusrhythm,firstdegreeAVblock,intermittentblockedatrialprematurecomplexes,
rightbundlebranchblock,andleftanteriorfascicularblock
B.Sinusrhythm,MobitztypeIseconddegreeAVblock,rightbundlebranchblock,and
leftanteriorfascicularblock
C.Sinusrhythm,MobitztypeIIseconddegreeAVblock,rightbundlebranchblock,and
leftposteriorfascicularblock
D.Sinusrhythm,thirddegreeAVblock,ventricularescaperhythm
E.AcceleratedidioventricularrhythmwithAVdissociation

3.TheanswerisB:Sinusrhythm,MobitztypeIseconddegreeAVblock,right
bundlebranchblock,andleftanteriorfascicularblock.Thisisbradycardiawithan
atrialratejustabove60bpm,withlengtheningofthePRintervalbeforedroppingQRS
complexes.NotetherearePwavesseenattheendoftheTwavesofthe3rd,5thand7thbeats
(seenbestinrhythmleadaVF),withnoconductionafterthe3rdand5thbeats,andevenlonger
PRprolongationafterthe7thbeat,allconsistentwithMobitztypeIseconddegreeAVblock.
TheQRSduration>120ms(>3smallboxes),therSRinleadV1,andthedeep,wideSwavesin
leadV6andleadIareallconsistentwithrightbundlebranchblock.Leftaxisdeviationinthe
absenceofleftventricularhypertrophy,inferiorinfarct,orleftbundlebranchblockis
consistentwithleftanteriorfascicularblock.ThereisalsoleftatrialenlargementdefinedbyaP
wave>120ms(>3smallboxes)and/ortwopositivedeflectionsseparatedby>40ms(>1small
box)inleadII,and/oraPwavewithanegativedeflectioninleadV1thatis>40ms(>1small
box)anddeeperthan0.1mV(>1smallbox).
MobitztypeIseconddegreeAVblockoccurswithintheAVnodeandiscommonlyasymptomatic,
notrequiringtreatment.Thispatientwassymptomaticwithevidenceofotherconductionsystem
disease(rightbundlebranchblockandleftanteriorfascicularblock)andthereforerequiresa
pacemaker.
A.ThisisnotfirstdegreeAVblockwithblockedatrialprematurecontractionsbecausenoneoftheP
wavesarepremature(theyareregular),andthepatternofprogressivePRlengtheningbeforethe
droppedQRScomplexisconsistentwithMobitztypeIseconddegreeAVblock.
C.MobitztypeIIseconddegreeAVblocklacksprogressivePRlengtheningmanifestingasrandom,
unpredictabledropsofQRScomplexes.Thisdegreeofheartblockisconsideredhighergrade(below
theAVnode)andoftenwillnecessitateapacemaker.Leftposteriorfascicularblock,notpresenthere,
isdefinedasrightaxisdeviationgreaterthan100degreesintheabsenceofrightventricular
hypertrophy,pulmonaryembolism,lateralwallinfarct,dextrocardia,orleadreversal.
D.ThirddegreeAVblockrequirescompleteAVdissociation,withtheatrialrategoingfasterthan
theventricularrate.Ventricularescaperatesareusuallylessthan40bpmandarefairlyregular.

E.Acceleratedidioventriculararrhythmiaisclassicallyseenasanasymptomaticventricular
tachyarrhythmiafollowingtheopeningofanacutelyoccludedcoronaryarterywiththrombolyticsor
percutaneouscoronaryintervention.Theyarefasterthanaventricularescaperhythm,butslower
thanVTandordinarilyresolveontheirownwithouttreatment.
Notes

4.A58yearoldmanwithahistoryofmedicallymanagedcoronaryarterydisease,endstage
renaldisease,hypertension,anddiabetespresentedwith2weeksofintermittentshortnessof
breathwithexertion.Onthedayofadmission,hedevelopedsubsternalchestdiscomfort
describedas,Someonesteppingonme.Intheemergencydepartment,hewasfoundtohavea
newleftbundlebranchblockandwenturgentlytothecatheterizationlabwherehewasfound
tohaveacompletelyoccludedproximalleftanteriordescendingarterythatwasopened
successfullywithabaremetalstent.Thirtyminutesafterhisintervention,thepatienthasthis
ECGdoneinthecoronarycareunit.Whatisthemostreasonablenextstep?
A.Continueobservation
B.Startintravenous(IV)amiodarone
C.Preparethepatienturgentlyforcardioversion
D.Calltheinterventionalcardiologisttotransportemergentlybacktothecatheterization
labforrepeatangioplasty
E.Havetranscutaneouspacingpadsplacedonthepatient,andthepacemakeronstand

by

4.TheanswerisA:Continueobservation.ThisECGrevealssinusrhythm,leftatrial
enlargement,firstdegreeAVblock,leftbundlebranchblock,prolongedQT,withtheonsetof
anacceleratedidioventricularrhythm(AIVR)startingwiththefourthbeat,andaventricular
prematurecomplex(thelastbeat).NotethereisAVdissociation,anindicationofaventricular
sourceofarrhythmia,asthesinusPwavesmarchthroughtherhythmstripatthebottom.By
the6thbeat,thePwaveisnotresponsiblefortheventricularcomplex(thePRintervalistoo
short),andbythe11thbeat,youcanseethePwaveasasmallnegativedeflectionattheendof
theQRScomplex(seeleadV1).Thelastrecordedbeatcomesinearly,butappearstobecoming
fromthesamefocusastheAIVR.Notethatitisfollowedimmediatelybywhatappearstobea
retrogradePwavetuckedinontheendoftheQRScomplex(bestseenasapositivedeflection
inrhythmleadV1,andanegativedeflectioninrhythmleadsIIandaVF).Thereisthena
compensatorypausewithlackofregularsinusactivity(blockedbytheretrogradePwave).In
AIVR,theventricularrateisoftenbetween60and110bpm,frequentlywithin10beatsofthe
sinusrhythmwithshiftbetweenthetwooccurringfrequentlyastheycompetefordominance.
Fusionbeats(4thand5thbeats)areQRScomplexeswithmorphologybetweentheventricular
beatsandthenormallyconductedsinusbeatsasimpulsesfrombothfusetoformthecomplex,
anothermarkerindicatingaventricularsourceforthearrhythmia.AIVRismostoftenseen
afteranacutelyoccludedcoronaryarteryisopenedwiththrombolyticsorballoonangioplasty
withorwithoutstentingandisduetoenhancedautomaticity.Itordinarilyrequiresno
treatmentandsubsidesonitsown.LastlyAIVRcanalsobeseenindigitalistoxicity,whichcan
betreatedwithdigoxinimmuneFab(immunoglobulinfragments)acutely.

B,C,D,E.Ordinarilythepatientisasymptomaticandrequiresnofurthertreatment.
Notes

5.Youareaskedtoseea52yearoldwomanwithnopriormedicalhistoryadmittedtothe
neurosurgeryservicewithanontraumaticsubarachnoidbleed.Shewasjustplacedona
ventilator,andanurgentcraniotomyisplanned.BasedontheaboveECGdone2hoursearlier,
whatwouldbethenextbeststepbeforeproceedingtosurgery?
A.Echocardiogramtoassessforrightventricularhypokinesis
B.RepeatECG
C.Lookatthechestxraytoconfirmacongenitalheartabnormality
D.Stresstesttoassessforsignificantischemia
E.blocker

5.TheanswerisB:RepeatECG.ThisECGdemonstratessinustachycardiawithapparent
rightaxisdeviation.However,makenotethatthepredominantlypositiveprecordialPwaves
appeartosupportasinusmechanismfromthehighrightatrium,butthenegativePwavesinI
andflatPwavesinIIdonot.Thissuggestslimbleadreversal.Withrightarmleftarmlead
reversal,asinthiscase,leadIisinverted,leadsIIandIIIarereversed,leadsaVRandaVLare
reversed,andleadaVFisunchanged.Rightarmleftarmleadreversalisoneofthemore
commonreasonsforrightaxisdeviation,andassessingthePwavemorphologyasexplained
helpsdistinguishthis.Thisabnormalityneedstobedistinguishedfromraredextrocardia,
whichcanalsoproducetheselimbleadfindings.However,theprecordialleadswouldshow
lackofnormalRwaveprogressionastheheartwouldbeintheothersideofthechest.
A.Despitewhatappearstoberightaxisdeviationatfirst,thereisnoevidenceofrightventricular
hypertrophyorenlargementonthisECG,andthereforeechocardiogramisnotindicated.
C.AstheECGpatternismostconsistentwithlimbleadreversal,thechestxrayisunlikelytobe
helpful,unlesstherewaslackofprecordialRwavetransitioninwhichdextrocardiawouldbemore
likely.
D.Thereisnoroleforastresstestinthispatientwhohasanurgentnoncardiacsurgicalindication.
E.Therearenofindingstosuggestthatblockertherapyisindicated.
Notes

6.A24yearoldfemaleheroinabuserpresentswithfeverandmalaise.Shehasbeen
complainingofmildpalpitations.Sheisnormotensive,hasanormalpulseoximetryonroom
air,buthasaholosystolicmurmurheardattherightlowersternalborder.AnECGis
performed.Whatwouldbethebestnextstep?
A.RepeatECGnow
B.Echocardiogram
C.Give1mgofIVatropine
D.Havetranscutaneouspacingpadsplacedonthepatient,andthepacemakeronstandby
E.CTscanofthechest

6.TheanswerisB:Echocardiogram.Thispatient'sECGdemonstratessinusarrhythmia,
whichcommonlyoccursintheyoungerpatient,andisduetonormal,enhancedvagaltone.The
PPintervalshortensduringinspiration,duetoreflexinhibitionofvagaltone,andlengthens
duringexpiration.Veryrarely,sinusarrhythmiaissymptomaticifthePPintervalis
excessivelylong.Followingbloodculturesandantibioticinitiation,atransthoracic
echocardiogramisthemostreasonablenextstepgiventheIVdrugabusehistory,thefever,and
themurmursuggestiveoftricuspidvalveregurgitation.
A.ThereisnoreasontoimmediatelyrepeatanECG.DailyECGs,however,arereasonableifthereis
concernorconfirmationofaorticvalvevegetations,asthesecouldprogresstoaorticvalveabscess
andAVnodalblock,oftenfirstmanifestasfirstdegreeAVblock.
C,D,E.Thereisnoimmediateindicationforatropine,transcutaneouspacing,orCTscan.
Notes

7.A23yearoldmanwithnopastmedicalhistorypresentstotheemergencydepartmentwith
syncope.Hehasanormalphysicalexam.BasedontheECG,themostappropriatenextstep
wouldbe:
A.Placementofatransvenouspacemaker
B.Referralforcoronaryangiographyandangioplastyifindicated
C.InitiationofIVamiodarone

D.Referralforneurologicevaluationandelectroencephalography(EEG)
E.Referralforelectrophysiologystudy

7.TheanswerisE:Referralforelectrophysiologystudy.ThisECGshowssinus
bradycardia,adeltawaveconsistentwithpreexcitationfromabypasstract(shortPRinterval,
slurringoftheinitialQRS),andapseudoinfarctpatterninferoposteriorly(QwavesandST
elevationinII/aVF).Thepresenceofapreexcitationpatternwithdocumentedarrhythmias
suchasAVRTisknownasWolfParkinsonWhitesyndrome.GiventhisECGpatternanda
historyofsyncope,thereishighconcernformorelethalarrhythmiasthatinvolvethebypass
tract,suchasatrialfibrillationthatcouldconductsoquicklydownthebypasstracttothe
ventriclesastocauseVT,ventricularfibrillation,andsuddendeath.Anelectrophysiologystudy
isindicatedwithpossibleradiofrequencyablationofthebypasstract.
A.ThereisnoindicationfortransvenouspacemakerbasedonthisECG.
B.Coronaryangiographyisnotindicated.TheQwavesseeninferiorlywithevidenceofposterior
involvement(prominentearlyRwaveinV2)are,infact,preexcitationdeltawavesindicativeofthe
bypasstract(pseudoinfarctpattern).
C.AmiodaronewouldbeagoodchoiceiftherewasevidenceofrapidatrialfibrillationorAVRT,
neitherofwhichhavebeendocumentedyetinthispatient.
D.NeurologicevaluationwithEEGiscertainlyareasonablestepinmanypatientswithsyncope.
However,withthisECGpattern,cardiacsyncopeismorelikely.

Notes

8.A25yearoldmancomplainsof2yearsofmildlyprogressivedyspneaonexertion.He
presentsforfurtherevaluation,mostlyattheinsistenceofhisfiance.Hisexamisnotablefora
prominentS2heartsoundandasystolicejectionmurmurattheleftuppersternalborderwith
asubtleheavenotedonpalpationoftheprecordium.HisECGshows:
A.Sinusarrhythmia,rightaxisdeviation,rightatrialenlargement,rightventricular
hypertrophywithstrainpattern
B.Sinusrhythmwithprematureatrialcomplexes,leftaxisdeviation,leftatrial
enlargement,rightbundlebranchblock
C.Sinusarrhythmia,leftaxisdeviation,leftatrialenlargement,leftventricular
hypertrophywithstrainpattern
D.Wanderingatrialpacemaker,rightbundlebranchblock
E.Sinusrhythm,leftaxisdeviation,ischemicTwaveabnormalities

8.TheanswerisA:Sinusarrhythmia,rightaxisdeviation,rightatrial
enlargement,rightventricularhypertrophywithstrainpattern.Thispatternis
consistentwithrightventricularhypertrophy.ThereisrightaxisdeviationdefinedasaQRS
axis>100degrees:notetheSwaveisdeeperthantheheightoftheRwaveinleadIwith
predominantlypositiveQRScomplexesintheinferiorleads.Rightatrialenlargementisdefined
asatallPwave>0.25mV(>2.5smallboxes)inleadII(seenhere),and/oratallPwaveinlead
V1>0.15mV(>1.5smallboxes).RightventricularhypertrophyisseenwithaprominentRwave
thatistallerthanthesmallSwaveinleadV1,relativelydeepSwavesinleadsV5andV6,atall
dominantRwaveinleadaVR,withdeepTwaveinversionsinleadsV1toV4consistentwitha
strainpattern.Onechocardiogram,thispatientwasfoundtohaveanenlargedrightatrium,a
hypertrophiedanddilatedrightventricle,andapulmonaryarterysystolicpressureof80mm
Hg.Hewasdiagnosedwithcorpulmonalesecondarytoidiopathicpulmonaryarterial
hypertension.
B,C,D,E.Therhythmisirregularduetosinusarrhythmia,notprematureatrialcontractionsor
wanderingatrialpacemaker.Theaxisisnotleftwardgiventhepredominantlynegativedeflectionin
leadI.Sometimescorpulmonalecanbeassociatedwithanincompleteorcompleterightbundle
branchblock.However,asintheECGabove,thereisnotruerSR,andtheQRSisnarrow,excluding
rightbundlebranchblock.ThenotchingofthedownslopeoftheRwaveinleadV1isconsistentwith
adelayedintrinsicoid(delayedtimeofthedownslopeaftertheRwave)whichisalsoseeninright
ventricularhypertrophy.ThoughthePwaveinleadV1appearstobedeeperthan0.1mV(>1small
box)itisnotquiteaswideas40ms(>1smallbox),makingthisborderlineleftatrialenlargement.
TheprecordialTwaveabnormalitiesaremoreconsistentwitharightventricularstrainpatternfrom
therightventricularhypertrophyinthisECGratherthanischemia.
Notes

9.A60yearoldmanwithhyperlipidemiatakingoverthecounterfishoilsupplements,and
withafamilyhistoryofayoungerbrotherwhohadcoronaryarterybypasssurgery,presented
toanemergencydepartmentafterexperiencingjawpainandupperchesttightnessthatstarted
attheendofaroutine20milebikeride.ECGAwasdoneatpresentation(withchestpain),
whileECGBwasdone5minuteslateraftersublingualnitroglycerinresolvedthepain.Based
ontheseECGs,themostlikelydiagnosisis:
A.Highgradeleftanteriordescendingarterystenosis
B.Highgraderightcoronaryarterystenosis
C.Hyperkalemia
D.Hyperventilation
E.Acutepulmonaryembolus

9.TheanswerisA:Highgradeleftanteriordescendingarterystenosis.ECGA
showssinusrhythmwithbroadbasedprominentTwavesintheprecordialleads(called
hyperacuteTwaves),inferiorleadSTsegmentdepression,andSTsegmentelevationinlead
aVR,allconsistentwithdiffusesubendocardialischemia,likelyfromahighgradeleftanterior
descendingarterystenosis.ComparingECGAtoapriorECG,orinthiscase,anECGdone5
minuteslater(ECGB)oncechestpainresolvedisimportantinidentifyingthesedynamicECG
changes.ECGBshowsresolutionoftheTwaveandSTsegmentchanges.Thepatientwenton
tothecatheterizationlabanda70%midleftanteriordescendingarterystenosiswith
associatedthrombuswassuccessfullystented.Hisprecatheterizationtroponinwasmildly
elevated,andhedidhavemidtodistalanteroseptal,anterior,andapicalakinesison
echocardiogramduetomyocardialstunningduringischemia.Whenischemiaislikelyby
clinicalhistoryandhyperacute,negative,orbiphasicTwavesarepresentintheprecordial
leads,thissuggestshighgradeleftanteriordescendingarterystenosisandshouldbetreated
aggressively.
B.ThoughthereareSTsegmentdepressionsintheinferiorleads,thedepressiondoesnotcorrelate
withterritoryofischemiaastheSTsegmentelevationdoes.Theserepresentreciprocalchanges:ST
depressionininferiorleadsduringanteroseptalischemiaorSTdepressionintheanteriorleads
duringinferoposteriorischemia.HyperacuteprecordialTwavesarenottypicallyseenwithright
coronaryarterystenoses.

C.Themorebroadbased,roundedTwaves(hyperacuteTwaves)herearenotconsistentwith
hyperkalemia,whichusuallywouldhavesharpsymmetricallypeakedTwaves,shorteningoftheQT
interval,andsometimesSTsegmentelevation.
D,E.HyperventilationandacutepulmonaryemboluscancauseSTsegmentandTwavechanges,
butnottypicallyhyperacuteTwaves.
Notes

10.Youarecalledtoseeyourpatientintheemergencydepartment.Heisa54yearoldman
withahistoryofabnormalcalciumscoreonapriorcardiacCT,hypertensionontreatment,and
afamilyhistoryofprematurecoronaryarterydiseaseinhisyoungerbrother.Hepresentswith
severechestpainandupperbackpaindescribedasaclampthatbegan30minutesearlier.
Hisbloodpressureis198/96mmHgintherightarm,and170/76mmHgintheleftarm.His
examisotherwiseunremarkableexceptfordiaphoresisandanxiety.AnECGisperformed.
Whichwouldbethemostappropriatenextimmediatestep?
A.V/Qscan
B.IVthrombolytictherapy
C.StartIVheparinnow,followedbyIVeptifibatideifhistroponiniselevated
D.Achestxray
E.IVnifedipine

10.TheanswerisD:Achestxray.ThisECGrevealssinusrhythmwithSTelevation
consistentwithanacuteinferiorinjurypatternandreciprocallateralSTdepression.Inthevast
majorityofcases,thereshouldbenodelayinopeningtheoccludedvesselwitheitherIV
thrombolytics(under30minutesfrompresentation)orcoronaryangioplastyandstent(under
90minutes).Beforeproceedingemergently,however,itisalwaysimperativetoexcludeany
contraindications.Inthispatientwithchestandbackpainandamorethan20mmHg
differenceinarmbloodpressures,ascendingaorticdissectionneedstoberapidlyexcluded.Ifa
widenedmediastinumisnotedonchestxray,thenemergentconfirmatoryimagingwitheither
CTangiographyoftheaortaortransesophagealechocardiographycouldbedonewhile
cardiothoracicsurgeryisemergentlyconsultedforalifethreateningaorticdissection.
Alternatively,anaortogramcouldbeperformedatthebeginningofacardiaccatheterization.If
themediastinumwasnotwidenedonchestxray,buttheclinicalsuspicionremainshigh,
furtherdiagnosticstudiesshouldberapidlyperformedbeforegivingthrombolytics,
anticoagulation,orIVantiplateletagents.Inthiscase,anaorticdissectionwasconfirmedon
CTangiogram,andthepatientwastransportedtoahospitalwithacardiothoracicsurgeon.
Ascendingaorticdissectioncarriesamortalityratethatincreasesapproximately2%perhour
afterpresentation.Aorticdissectionscansometimespresentwithanacuteinjurypatternsuch
asthisifthedissectionflapoccludesflowintotherightcoronaryartery(whichoccursmore
commonlythanintotheleft).
A.ThereisnoroleforaV/Qscantolookforpulmonaryembolusinthisscenario.
B,C.IVthrombolyticsareabsolutelycontraindicatedinsuspectedaorticdissection,andIVheparin
couldworsenthedissectionacutely.

E.IVnifedipinewouldnotbeagoodchoiceasthiscouldactuallycauseareflexincreaseincardiac
contractility,possiblyworseningtheextentofanaorticdissection.AcontinuousIVblockerwould
bethefirstagentofchoicetorapidlycontrolthebloodpressurewhilediagnostictestsareperformed.
Notes

11.A37yearoldmalewithahistoryofseizuressinceage15andmigraines,presentstoyour
officewithcomplaintsofpalpitationsthatawokehimfromsleepthenightbeforeand
associatednearsyncope.Thisoccurredoffandonoverafewhours.Hehashadsimilar
symptomssporadicallyinthepastseveralyears,butthisepisodewasmoreintense.He
describesthemasdifferentfromhisseizures.Hisexamisnormal.BasedontheECGabove,the
mostappropriaterecommendationis:
A.24houroutpatientcontinuousECGmonitoring
B.Electrophysiologystudy
C.Followupwithhisneurologist
D.Stresstest
E.Admissiontothehospitalforemergentcoronaryangiographyandangioplastyif
indicated

11.TheanswerisB:Electrophysiologystudy.Anelectrophysiologystudyisindicatedin
thisparticularpatientwithnearsyncopebecausetheECGshowssinusrhythmwithST
elevationinleadsV1andV2consistentwithaBrugadapattern.Brugadasyndromewiththis
ECGpatternisassociatedwithventriculararrhythmiasandsuddenarrhythmicdeath.
ClassicallyitischaracterizedbyaprominentcovedSTelevationgreaterthan0.2mV(>2small
boxes)endingwithanegativeTwaveinleadsV1andV2.Alternatively,andmoresubtly,this
STelevationmorethan0.2mV(>2smallboxes)canthensaginthemiddleoftheSTsegment
>0.1mVor<0.1mV(>or<1smallbox)abovebaseline,endingwithapositiveorbiphasicT
wave(termedsaddlebackvariations).TheprevalenceintheUnitedStatesisnotwell
established,butinAsiathissyndromeisthemostlikelycauseofsuddendeathinmenyounger
than50yearsofage.Itisatleasteighttimesmoreprevalentinmenthaninwomen.Syncope
andsuddendeatharecommon.Thefamilyhistoryisimportant,thoughthesyndromecan
occursporadically.ItisduetoamutationintheSCN5Agenethatencodesforavoltagegated
sodiumchannel.Treatmentisanimplantablecardioverterdefibrillatortopreventsudden
death.
A,C.GiventhepresyncopewiththisconcerningECGpattern,electrophysiologystudyiswarranted
before24hourcontinuousECGmonitoringorneurologyfollowup.
D,E.Thereisnoindicationforastresstestorcoronaryangiography,asthehistoryandST
abnormalitiesarenottypicalofischemia.
Notes

12.A49yearoldpatientwithahistoryofsicklecelldiseaseandcocaineabusewasadmitted
withintestinalischemiaandunderwenturgentexploratorylaparotomyandresectionofthe
terminalileum.Twodayspostoperatively,anECGisperformed.Whatwouldbethemost
appropriatenextbeststep?
A.Urgentcoronaryangiography
B.IVcalciumgluconate
C.IVmagnesiumsulfate
D.IVsodiumphosphate
E.PulmonaryembolusprotocolCT

12.TheanswerisB:IVcalciumgluconate.ThisECGdemonstratessinustachycardia,
borderlineintraventricularconductiondelay,andpeakedTwavesconsistentwith
hyperkalemiafromlacticacidosisandrenalfailure.Calciumgluconateantagonizestheeffects
ofpotassiumonmyocardium,rapidlyreducingthelikelihoodofarrhythmias,butitdoesnot
actuallylowerthepotassium.Othermeasuressuchasinsulinanddextroseinfusions,sodium
bicarbonate,andhighdoseinhaledagonistswilllowerpotassiumbydrivingpotassiumfrom
theextracellularspaceintocells.Sodiumpolystyrenesulfonatebindspotassiuminthe
intestinaltractbuttakesthelongesttoact.Lastly,inrenalfailure,hemodialysisissometimes
indicatedifmedicaltherapyfails.HyperkalemiahasatypicalECGprogression.Withpotassium
levelsbetween6to6.5mEq/L,therearepeakedTwaveswithaverypointypeakasinthis
example.From6.5to7mEq/L,thereisPRprolongationwithflatteningthenlossofPwaves.
Greaterthan8mEq/L,thereiswideningoftheQRSprogressingtoasinewavemorphology
followedbyventricularfibrillationandasytole.
A,C,D,E.Thereisnoroleforcoronaryangiography,astheJpointelevationseeninleadsV1
throughV3islikelyduetothehyperkalemia.Additionally,theSTsegmentisnotconvexupaswould
bemoretypicalofanacuteinjurypattern.IVmagnesiumsulfate,potassiumphosphate,ora
pulmonaryembolusprotocolCTarenotindicated.
Notes

13.A47yearoldwomanwithhypertensiontreatedwithatenololpresentstoyouroffice
complainingoffatigueanddyspneawithexertionthatshenoticesonhermorninghikes

throughthewoodsoverthelastweek.Shehasnocomplaintsatrest.Onreviewofsystems,she
doesreportrashesinthepast,butshecouldnotrecalliftheywerebull'seyeshaped.Shedoes
reportfrequentlyremovingticksfromherdog.Onexam,herpulserateisslow,butthereareno
rashes.BasedonherECG,whatwouldbethebestrecommendation?
A.Decreasetheatenololdoseandfollowupin1week
B.Admissiontothehospitalforplacementofapermanentpacemaker
C.Admissiontothehospitalforobservationandpossibleplacementofatransvenous
pacemaker
D.CheckLymetiterswithcloseoutpatientfollowup
E.Admissiontothehospitalforurgentcoronaryangiogramwithangioplastyifindicated

13.TheanswerisC:Admissiontothehospitalforobservationandpossible
placementofatransvenouspacemaker.TheECGshowssinusrhythmwithleftatrial
enlargement,thirddegreeAVblock,andafascicularescaperhythminthe30s.Giventhehigh
gradeAVblockwithanescaperhythmwellbelowtheAVnode,sheisathighriskforsyncope
andsuddendeath.Thereforeadmissiontothehospitalforconsiderationofatransvenous
pacemakerisappropriate.
A.Thisisanunstablerhythm,andthoughstoppingtheatenololisappropriate,allowingfor
outpatientfollowupisinappropriate.

B,D.Givenherhistory,checkingLymetitersonadmissiontothehospital(notasanoutpatient)
wouldbereasonable,becauseifthisisheartblockrelatedtoLymedisease,withantibiotictreatment
shemaynotrequireapermanentpacemaker.
E.Althoughcoronaryarterydiseaseandischemiacanleadtoheartblock,thereisnothinginher
historyorECGthatwarrantsurgentcoronaryangiography.
Notes

14.A28yearoldmanpresentsforacheckupafterhis25yearoldbrotherdiedsuddenlywhile
divingintheCaribbeanIslands.Yourpatient'shistoryisnotableforonesyncopalepisode4
yearsearlier,whichwassuddenandbrief,withoutassociatedtrauma.Hedidnotseekmedical
careandthoughtitwasduetoheatexhaustion.Hetakesnomedicationsandhasnosignificant
medicalorsocialhistory.Hisexamisnormal.HisECGisshownabove.Whichofthefollowing
canbeassociatedwiththisECGfinding?
A.Hypercalcemia
B.Hyperkalemia
C.IVlidocaine
D.Erythromycin
E.Metoprolol

14.TheanswerisD:Erythromycin.TheECGshowsnormalsinusrhythmwithamarkedly
prolongedQTinterval.QTprolongationisassociatedwithpolymorphicventriculartachycardia
(torsadesdespointes)anddeath.Giventhepatient'sageandfamilyhistoryofpossiblesudden
cardiacdeath,congenitallongQTsyndromeisprobable.Thereareotheracquiredcausesof
longQT,suchasmedicationsincludingerythromycin,clarithromycin,haloperidol,and
methadone.Foracompletelistrefertowww.torsades.org(http://www.torsades.org).Inaddition,
otherconditionsmaycauselongQTsuchasischemia,hypothyroidism,hypocalcemia,
hypokalemia,andseverebraininjury.ThenormalcorrectedQTintervalisconsideredtobe
<460msforwomenand<440msformen.Visually,theQTintervalshouldbelessthan50%of
theRRinterval.CongenitallongQTsyndromemaybeassociatedwithafamilyhistoryof
suddendeathorsyncope,especiallybeforetheageof40.LongQTsyndromesurvivorsof
suddencardiacdeathwarrantanimplantablecardioverterdefibrillator.Otherwise,blockers
arethemainstayofpreventioninmostcases.
A,B,E.Asabove.
C.IVlidocaineandoralmexiletineareamongafewantiarrhythmicmedicationsthatdonotprolong
theQTinterval,unlikequinidine,procainamide,sotalol,andamiodarone.
Notes

15.A56yearoldwomanpresentstoherprimarycarephysiciancomplainingof3weeksofnew
fatigue.Shehasrarelyexperiencedasenseofdyspneawithexertion,butdeniesorthopnea,
chestpain,presyncope,orsyncope.Shetakesconjugatedestrogensforhotflashesafteran
uncomplicatedhysterectomy10yearsago.Herexamisnotableforabloodpressureof150/88
mmHg,butisotherwiseunremarkable.AnECGisdoneintheoffice.Thebestnextstepwould
be:
A.Electrophysiologyconsultationforelectrophysiologystudy
B.LowerextremityDopplerultrasound
C.Initiateatenolol
D.Tilttabletest
E.Stresstest

15.TheanswerisA:Electrophysiologyconsultationforelectrophysiologystudy.
TheECGshowssinustachycardiajustabove100bpmwithseconddegree2:1AVblock.TheP
wavesthatarenotconductingcanbeseenattheendofeveryTwave(mostevidentherein
leadsI,V1,andV6).Becausetheblockis2:1,youcannotdifferentiatebetweenMobitzIor
MobitzIIseconddegreeAVblock.MobitzIseconddegreeAVblock,withprogressivePR
lengtheningbeforedroppingaQRS,isgenerallyconsideredtobeintranodal(withintheAV
node),typicallystable,andrarelyrequiresapacemaker(onlyifverysymptomatic).MobitzII
seconddegreeAVblockhasastablePRintervalbutmanifestsasrandom,unpredictable
droppingoftheQRScomplexesandisinfranodal,usuallyunstable,andoftenrequiresa
pacemaker.Anelectrophysiologystudywouldbeindicated,givenhervaguecomplaints,to
differentiatetheexactlevelofblockandneedforpacemaker.
B.Thoughthepatienttakesconjugatedestrogens,whichcanincreasethepossibilityof
venothromboembolicdisease,theECGmandatesfurtherelectrophysiologyevaluationfirst.
C.Thoughsheishypertensive,atenololiscontraindicatedinthesettingofseconddegreeAVblock.
D.Tilttabletestingisusedincertainpatientswithsyncopeorpresyncope,butthispatienthas
neithercomplaint.
E.Thoughheartblockanddyspneacanbeassociatedsometimeswithischemicheartdisease,the
nextbesttestinthissituationisanelectrophysiologyevaluation.
Notes

16.A44yearoldmanpresentstotheemergencydepartmentwith60minutesofnewchest
fullnessanddiaphoresisfollowedbyabrieflossofconsciousnessthatbeganwhilehewas
carryingdrywallonajobsite.Intheabsenceofanyabsolutecontraindications,thebestnext
stepwouldbe:
A.IVfibrinolyticsunder30minutesfromarrivaltotheemergencydepartment
B.PulmonaryembolusprotocolCT
C.Coronaryarteryangiographywithangioplastywithorwithoutstentingunder90
minutesfromarrivaltotheemergencydepartment
D.Electrophysiologystudy
E.Antiinflammatorymedications

16.TheanswerisC:Coronaryarteryangiographywithangioplastywithorwithout
stentingunder90minutesfromarrivaltotheemergencydepartment.ThisECG
showsanacuteanteroseptalinjurypatternwithinferolateralreciprocalSTdepression,
consistentwithananteroseptalSTelevationmyocardialinfarction.Thebasicrhythmissinus
rhythmwithleftatrialabnormality,leftventricularhypertrophy(Rwave>0.11mV[>11small
boxes]inleadaVL)withassociatedleftaxisdeviation,earlyprecordialR/Stransition(asthe
QRScomplextransitionsfromnegativetopositiveearlierthanleadV3),prolongedQT,a
prematurejunctionalbeat(13thbeat),andwithslightaberrancy(incompleteRBBBpatternby
thersRpatterninV1,lessthan120ms[<3smallboxes]).Coronaryarteryangiographywith
angioplastywithorwithoutstentingwithin90minutesofarrivaltotheemergencydepartment

isthetreatmentofchoice.
A.Asmostemergencydepartmentsdonothaveaccesstoacardiaccatheterizationlab,IV
fibrinolyticsarethesecondbestoption,andshouldbegivenwithin30minutesofarrivaltothe
emergencydepartment.
B,D,E.ThesymptomsandSTchangesarenotindicativeofpulmonaryembolus,chronic
arrhythmias,orpericarditis,andthereforepulmonaryembolusprotocolCT,electrophysiologystudy,
andantiinflammatorymedicationsarenotindicated.Syncopemayberelatedtotransientlife
threateningVT,oftenassociatedwithanacuteinfarction.
Notes

17.A70yearoldwomanwithahistoryofcoronaryarterydiseasetreatedwithpriorcoronary
arterystentsandknownrightbundlebranchblockpresentstoyourofficeforaroutinevisit.
Shecomplainsofnewmildfatiguefor1week.Herpulserateis130bpm,bloodpressureis
104/76mmHg.HerexamisotherwisenotableforcanonAwaves,variableS1,anda2/6
holosystolicmurmurattheapex.HertachycardiapromptsanECGthatshowsthefollowing:
A.Sinustachycardiawithrightbundlebranchblockandleftposteriorfascicularblock
B.VT
C.Atrialflutter2:1conduction,withrightbundlebranchblockandleftposterior
fascicularblock

D.AVRT
E.Sinustachycardiawithleftbundlebranchblock

17.TheanswerisB:VT.TheECGdemonstratesawidecomplexregulartachycardia
consistentwithVT.Therearetwoothermajorcausesofwidecomplextachycardias:any
supraventriculartachycardia(includingsinustachycardia)withbundlebranchblockaberrancy
andAVRT(alsoknownasatrioventricularreciprocatingtachycardia).Clinicallythehistory
ofcoronaryarterydiseasemakesanywidecomplextachycardiamorelikelytobeVT.
Importantly,hemodynamicstabilitydoesnotexcludeVT.Threeimportanttelemetrystripor
ECGfindingsthatconfirmVToversupraventriculartachycardiawithaberrancyareAV
dissociation,fusionbeats,andcapturebeats.AVdissociationisnotedherewith
supraventricularPwavesseenbetweenthe12thand13thbeatsandthe14thand15thbeatsare
slightlymorenegativedeflectionsthanonrhythmleadV1.NotethesePwavesareseeninlead
V2,aswell.Withcalipers,youcanmarchthesePwavesbothforwardandbackwardtofind
otherconfirmatorydissociatedPwavesatthesameatrialrate(about70bpm)whichisslower
thantheVT.AVdissociationleadstoavariablesoundingS1asthemitralandtricuspidvalves
havevariabledegreesofopeningdependingonthetimingofdissociatedatrialcontractions.
FusionbeatsareQRScomplexeswithmorphologybetweensupraventricularconductionand
ventricularconductionasthetwowavefrontsmeettoformonecomplex(notseenhere).
CapturebeatsoccurwhenadissociatedPwavefortuitouslyfindstheentireventricleavailable
fordepolarizationbetweenVTbeats,creatinganormalnativecomplex(notseenhere).Other
supportingfindingstosuggestVTare:anaxisbetween90degreesand180degrees
(northwestaxisorfarleftaxis)monophasicRwaveinV1(seenhere)rSRinV1withRwave
tallerthanrdeeperSwavedepththanRwaveheightorQSwaveinV6(seenhere).Thereare

alsoothermoreadvancedcriteriacalledBrugadacriteriaandleadaVRcriteria.
A.TheAVdissociationexcludessinustachycardiawithrightbundlebranchblockandleftposterior
fascicularblock,nottomentionthemonophasicRwaveinV1ismoreconsistentwithVTthana
typicalrightbundlebranchblock.
C.Theabsenceofflutterwavesexcludesatrialflutter.
D.WidecomplexAVRToccurswhenthereisconductionusingabypasstract,inwhichthereentry
circuitgoesdownthebypasstractanduptheAVnode,orlesscommonlydowntheAVnode,
conductingwithabundlebranchblockaberrancyandupthebypasstract.Thecriterianotedin
answerBfavorVTandnotAVRT.
E.ThepositivedeflectioninV1excludesleftbundlebranchblock.
Notes

18.A46yearoldwomanwithahistoryofsystemiclupuserythematosuspresentswithacute
onsetchestpainbeginning5hoursago.Itbeganatrest,andshereportsbeingunabletolieflat.
Heryoungerbrotherhadamyocardialinfarction1yearearlier.Herexamisnotableformild
distress,bloodpressure112/78mmHg,andpulseoximetry93%onroomair.Thereisafaint
malarrash,slightlydecreasedbreathsoundsatthebasesofherlungs,witharegularheart
rhythmwithoutmurmursorgallops.Thereisnolowerextremityedema.BasedonherECG,the
bestnextstepis:
A.Aspirinandurgentcoronaryangiographywithpossibleangioplastyandstenting

B.IVheparinandpulmonaryembolusprotocolCT
C.AspirinandIVthrombolytics
D.Aspirinandanechocardiogram
E.IVheparinandeptifibatide

18.TheanswerisD:Aspirinandanechocardiogram.Highdoseaspirinoribuprofenis
thetreatmentofchoiceforpericarditis.Inpatientswithafirstepisodeofpericarditis,
regardlessofcause,theadditionofcolchicineappearstoreducethedurationofsymptomsand
therecurrencerate.Thereissomesuggestionthatsteroidsareassociatedwithahigher
recurrencerate,thoughtheymayneedtobegiventotreatanunderlyingautoimmunedisease
whenpresent,orwhenaspirinornonsteroidalantiinflammatorymedicationsare
contraindicatedorineffective.Anechocardiogramisreasonabletoassessforapericardial
effusioninthispatientwithlupus.TheECGshowsdiffuseSTsegmentelevationwithout
reciprocalchanges.ECGchangesinacutepericarditisoccurinfourstages,butsomemaybe
missed.StageoneconsistsofconcaveupwardSTsegmentelevationandPRsegment
depressioninalmostallleadsexceptaVR.StagetwoiswhentheSTsegmentreturnstobaseline
withtheonsetofTwaveflattening.StagethreeconsistsofTwaveinversions.Stagefourisa
returntothenormalECG.Classicallythepatienthasworseningofsymptomswhensupineand
relativereliefwhenleaningforward,becausethehearthangsfromitsattachmentwithless
approximationoftheheartwiththepericardialsacwallsinthisposition.Apericardialfriction
rubisoftenheardasaraspingorscratchingsound,butmaybeabsentorintermittent.

A,C,E.Thoughthispatienthasriskfactorsforcoronaryarterydisease,includinglupusandthe
prematurefamilyhistory,theSTsegmentelevationisdiffuse,notconvexup,andnotassociatedwith
reciprocalSTsegmentdepression.Therefore,thereisnoroleforcoronaryangiography,IV
thrombolytics,IVheparin,oreptifibatide.
B.Thoughthepatienthaschestpainwitharelativelylowpulseoximetry,theECGpointstoward
pericarditisasaspecificcause,thereforethereisnoroleforpulmonaryembolusprotocolCTatthis
point.
Notes

19.A51yearoldmanwithnoninsulindependentdiabetes,hypertension,hyperlipidemia,
andchronicobstructivepulmonarydisease(COPD)presentsforaphysicalinordertoapplyfor
alifeinsurancepolicy.Hismedicationsincludemetformin,atenolol,hydrochlorothiazide,and
tiotropiuminhaler.AnECGisperformed.Thebestnextstepis:
A.Aspirin
B.Increaseatenolol
C.Dronedarone
D.Amiodarone
E.Warfarin

19.TheanswerisE:Warfarin.TheECGshowsatrialflutterwith4:1conduction.Warfarin
isindicatedgivenhishistoryofdiabetesandhypertension.Atrialflutterisareentrantatrial
arrhythmiawhichtypicallyhasanatrialrateofbetween250and350bpm.Ifthispatientwas
notalreadytakingatenolol,theatrialfluttermayhaveconducted2:1,resultinginaheartrateof
about150bpm,whichmaymakeseeingtheflutterwavesmoredifficult.Themostcommon
formofatrialflutterhasdominantnegativeflutterdeflectionsintheinferiorleadsandpositive
flutterdeflectionsinleadV1duetocounterclockwiserotationwithintherightatrium,passing
throughthecavotricuspidisthmus.Generallyatrialflutterisacutelytreatedlikeatrial
fibrillation.Ifunstable,cardioversionisindicated.Iftachycardic,IVororalratecontrolling
agentscanbeofferedsuchasnondihydropyridinecalciumchannelblockers,blockers,and
lesscommonlydigitalis.Anticoagulationdecisionsaresimilartothoseforatrialfibrillation,
thoughtherearelessdata.Lastly,catheterbasedablativetherapiescanbeoffered,particularly
whentheatrialflutteriscounterclockwiseinvolvingthecavotricuspidisthmus,suchasthis.
A.Aspirinwouldbeinsufficientbecausehehastworiskfactorsforthromboembolism:diabetesand
hypertension(CHADS2scoreof2).
B.Increasinghisatenololdosewouldnotbenecessarygivenheisalreadyadequatelyratecontrolled
atrest.
C.Amiodaroneordronedaroneareantiarrhythmicagentswhichshouldnotbestartedwithout
anticoagulationinthispatient.
Notes

20.A77yearoldwomanwithparoxysmalatrialfibrillation,diabetes,andbreastcancer
recentlyhadherdiltiazemincreasedanddigoxinadded3weeksearlierforrapidventricular
rates.Overthelastseveralweeks,shehashadprogressivefatigue,anorexia,andseveralnear
syncopalepisodes.AnECGisperformedintheoffice.Whichofthefollowingstatementsis
mosttrueregardingherdiagnosis?
A.Thereiscompleteheartblock.
B.Coronaryangiographyshouldbeurgentlyperformed.
C.Ventriculararrhythmiasareuncommon.
D.Visionchangesarecommon.
E.Glucagonshouldbegiven.

20.TheanswerisD:Visionchangesarecommon.ThisECGrevealssinusbradycardia
witharatebelow40bpm,withSTsegmentchangesconsistentwithdigitaliseffect.The
constellationoffatigue,anorexia,andpresyncopecombinedwiththisECGrevealingsignificant
bradycardiaraisestheconcernfordigitalistoxicity.Visionchangesarerelativelycommon,
includingcolorvisioncomplaints,scotomasand,rarely,blindness.Withchronictoxicity,serum
digitalislevelsdonotcorrelatewithtoxicityasasymptomaticpatientscanhaveelevatedlevels,
andtoxicpatientscouldhavetherapeuticlevels.Medicationssuchasdiltiazem,verapamil,
amiodarone,andquinidinecanincreasedigitalislevels,ascanrenalfailure.Arrhythmiasare
common,includingbradyarrhythmias(includingcompleteheartblock)duetoexcessive
parasympathetictone,junctionalarrhythmias,ventricularectopy,andVTs,including
biventriculartachycardia,andclassically,atrialfibrillationorflutterwithAVblock.STsegment
changesfromdigitaliseffectareanormalJpoint(thetransitionpointbetweentheQRS
complexandtheSTsegment)followedimmediatelybyasaggingorscoopedSTsegment.ECG
digitaliseffectalonedoesnotindicatetoxicity.
A.Thoughcompleteheartblockisassociatedwithdigitalistoxicity,thisECGshowsPwavesbefore
eachQRScomplex,excludingcompleteheartblockhere.
B.TheSTsegmentchangesarenottypicalofischemia,butratherdigitaliseffect.
C.Ventriculararrhythmiasarecommonwithdigitalistoxicity,especiallyinpatientswithpreexisting
heartdisease.
E.Glucagoncanbeusedasanantidoteforblockerandcalciumchannelblockertoxicity.However,
theconstellationofsymptomsandECGfindingsismoreconcerningfordigitalistoxicity,therefore
digoxinimmuneFabshouldbegiven.

Copyright2015Elsevier,Inc.Allrightsreserved.

BOOKCHAPTER

Radiology
AmberS.TullyMDandJamesS.StuddifordMD,FACP
USMLEImagesfortheBoards,SectionIII,175175

Notes

1.A69yearoldwomanwithahistoryofpoorlycontrolledhypertensionandtype2diabetes

mellituspresentedwithahistoryofleftsidedweaknessandsensorydeficitpredominantly
involvingthelowerlimb,withhypobulia.Thecomputedtomography(CT)scanisabove.The
mostlikelycauseoftheabovefindingis:
A.Hypertensiveintracranialhemorrhage
B.Atheroscleroticocclusionoftherightmiddlecerebralartery(MCA)
C.Atheroscleroticocclusionoftherightanteriorcerebralartery(ACA)
D.Ischemicinfarctionsecondarytohypoperfusion

ACAInfarct

1.TheanswerisC:Atheroscleroticocclusionoftheanteriorcerebralartery(ACA).
Thereishypodensity(arrows)withmildmasseffectinvolvingtheparamedianrightfrontal
lobe.ThisisconsistentwithanACAterritoryacuteischemicinfarct.Themostcommoncause

ofanischemicACAterritoryinfarctisatheroscleroticocclusion,typicallyattheA2origin.The
patient'shistoryofhypertensionandtype2diabetespredisposehertoatheroscleroticdisease.
A.Thepatient'shistorydoesputheratriskforhypertensiveintracranialhemorrhage.However,this
wouldpresentasahyperdensity,mostcommonlyinvolvingthebasalganglia,followedbythe
cerebellumandpons.
B.Theappearanceisconsistentwithanischemicinfarctduetoarterialocclusion.However,the
territoryinvolvedisnotconsistentwiththeterritorycoveredbytheMCA.
D.Theappearanceisconsistentwithanischemicinfarction.However,infarctsrelatedto
hypoperfusiontypicallypresentattheborderzonesbetweenvascularterritories.
Notes

2.A20yearoldwomanwithahistoryofasthmapresentstotheemergencydepartment
complainingofchestandneckdiscomfort.Herchestxray(left)andneckxray(right)are
shownabove.Whichofthefollowingisthecorrectdiagnosis?
A.Pneumomediastinum
B.Pneumothorax
C.Pneumoperitoneum
D.Hyperinflationwithoutextrapulmonaryair
E.Pneumothoraxandpneumomediastinum

PneumomediastinumwithAsthma

2.TheanswerisA:Pneumomediastinum.Spontaneouspneumomediastinummayoccur
inanypatientwithcoughingorstraining,butitismorecommonamongasthmatics.Thecause
isbelievedtobespontaneousruptureofasmallblebalongthemedialpleuralsurface,closeto
themediastinum.Thispatient'ssymptomsofsuddenonsetofchestdiscomfortwithchestand
necktightnessisatypicalclinicalmanifestationofspontaneouspneumomediastinum,which
requiresnotreatmentinthissituation.Achestxrayisdonetobecertainthereisno
pneumothorax,whichmayrequiretreatmentwithachesttube.
Thechestxrayshowsfaintlucentstreaksofairalongthebordersofthesuperiormediastinum
(yellowarrows).Theneckxrayshowsmoreobviouslucentstreaksextendingfromthesuperior
mediastinumintothesofttissueplanesoftheneck(redarrows).Inthiscase,achestCT(aboveright)
wasalsoperformed,confirmingmediastinalairdissectingintotheneck,althoughCTevaluationis
notmandatoryinthisbenignclinicalsituation.Hadthepatientbeenvomitingorhadsheswallowed
somethingsharp,therewouldbereasontosuspectanesophagealtear,andaGastrografinswallow
andchestCTwouldberequired.
B,E.Nopneumothoraxisvisibleonthechestxray.Thelungapicesarealsowellvisualizedonthe
neckxray,withnoevidenceofpneumothorax.
C.Thereisnoevidenceofpneumoperitoneum,whichtypicallyappearsasalucentzoneofair
beneaththerighthemidiaphragm.Theairseenbeneaththispatient'slefthemidiaphragmrepresents
thenormalgastricairbubble.
D.Thereisnoevidenceofhyperinflationdespitethehistoryofasthma.Thelungvolumesappear
normalwithnohyperlucencyandnoflatteningofthediaphragm.
Notes

3.Amotherreportstothepediatricianwithher9montholdsonwithachiefcomplaintof
refusaltofeedtodayandincreaseddrooling.Theinfantisinconsolablycrying,making
auscultationofthelungsdifficult,butappearsdyspneic,andbetweencriesthephysicianhears
stridor.Thechildistransferredtothenearestemergencydepartmentandanteroposterior(AP)
andlateralchestradiographsareobtained.Whichofthefollowingstatementsistrue?
A.Basedontheradiographicfindings,abronchoscopyshouldbeperformedimmediately.
B.Therightlungishyperinflatedrelativetotheleft.
C.Inthesettingofasuspectedforeignbodywithintheairway,ifradiographsarenormal,
bronchoscopyisnotindicated.
D.Dyspneaandstridorcanoccurinchildrenwhenaforeignbodyiswithinthe
esophagus.

E.LowdoseCTofthethoraxisnothelpfulinevaluationforforeignbodieswithinboth
theesophagusandairways.

ForeignBodyinEsophagus(Coin)

3.TheanswerisD:Dyspneaandstridorcanoccurinchildrenwhenaforeignbody
iswithintheesophagus.Thisinfantpresentswithfairlytypicalsignsandsymptomsof
foreignbodyingestion/aspiration.Theradiographsconfirmthepresenceofacircularmetallic
foreignbodyatthelevelofthethoracicinlet,posteriortothetracheaonthelateralview,andin
acoronalorientation(arrows)thus,impactedwithintheesophagus.Acluethatthepresumed
coinisintheesophagusontheAPradiographisitscoronalorientation(enface),becausea
coinshapedobjectwouldfavorasagittalorientation(enprofil)withinthetrachea(duetothe
incompleteposteriorcartilaginousringsofthetrachea).Foreignbodieswithintheesophagus
canhavemasseffectontheposteriortracheaandinducedyspneaandstridor,especiallyatthe

thoracicinlet,oftenconfoundingtheclinicalpresentation.
Foreignbodyaspiration/ingestionisacommoncauseofrespiratoryandgastrointestinal(GI)malady
inthepediatricpopulation.Potentialcomplicationsofesophagealforeignbodiesincludeimpaction,
laceration,andperforation.Aspiratedoringestedforeignbodiescanmanifestwithbothacuteand
chronicsymptomsandsigns.Thepreferredmethodofextractionofimpactedesophagealforeign
bodiesisthroughendoscopy,withoverallaminimalcomplicationrate.
A.Hadtheforeignbodybeenwithinthetracheaonthelateralview,animmediatebronchoscopyfor
retrievalwouldhavebeenindicated.
B.Aforeignbodythatisaspiratedcanoftenobstructanairwaydistaltothetracheaandcreatea
ballvalveeffect,resultinginhyperinflationoflungdistaltotheobstruction.Ofcourse,neitherlung
ishyperinflatedinthiscase.However,ifaspiration,asopposedtoingestion,werestilla
consideration,additionallateraldecubitusviewscouldbeobtained.Normallyinthelateraldecubitus
position,thesidedownwillbelessinflatedduetotheweightofthebodybut,intheballvalve
obstructiontypescenario,thesidedownwiththeobstructionmaybehyperinflated.
C.Coinsareprobablythemostcommonlyswallowedforeignbody,butanythingsmallisapotential
ingestion/aspirationhazardforchildren,especiallyundertheageof2,andmostairwayforeign
bodiesarenotradiopaque.Therefore,inachildwithacuterespiratorysymptomsorwhoisunstable,
bronchoscopyshouldbeperformedirrespectiveofnegativeradiographs.
E.Ifthechild'ssymptomsareofamorechronicnature,lowdoseCTofthethorax,withpotentialfor
CTbronchography,iscertainlyindicatedtoidentifythelocationofapotentialesophagealorairway
foreignbodyanddirecttheendoscopist/bronchoscopist.
Notes

4.A23yearoldmanpresentstotheemergencydepartmentwithfever,nausea,andvomiting
andisfoundtohavediffuseabdominaltendernessandaleukocytosis.ACToftheabdomenis
performed.WhichofthestatementsregardingtheCTfindingsiscorrect?
A.Thepericecalfatisnormalindensity.
B.Theappendixisabnormallydilated,anditswallisabnormallyenhancing.
C.Thisappendixislikelytobecompressibleonultrasound.
D.Thereisanappendicolith.
E.Thereisstrongevidenceofappendicealperforation.

AcuteAppendicitis

4.TheanswerisB:Theappendixisabnormallydilatedanditswallisabnormally
enhancing.Thisisafairlytypicalpresentationforappendicitis.Ontheimagesabove,thereis
adilated,retrocecalappendix(redarrows)withavidwallenhancementandmarked
periappendicealfatinfiltration,allfindingssupportingadiagnosisofacuteappendicitis.OnCT
theappendixshouldbelessthan7mmindiameter.Afluidfilledappendixwithout
intraluminalairorcontrast(ifadministered),andcecalinflammationcanaddtothesuspicion.
Acuteappendicitisisduetoluminalobstruction,usuallyattheostia(byanappendicolithor
hypertrophiedPeyerpatches)andasuperimposedinfection.Associatedcomplicationsinclude
perforationwithpneumoperitoneumandperiappendicealabscessformation.Treatmentissurgical
resection,ifthelesionisnotperforatedorisminimallyperforated,andpercutaneousdrainageifa
wellformedabscessdevelopsinthesettingofperforation.Incertainpatientpopulations,suchas
pregnantandpediatricpatients,ultrasoundisapreferredinitialimagingmodalityinsuspected
appendicitistoavoidirradiation.
A.Asabove,thepericecalfatisabnormalindensity.
C.Additionalsonographicfindingssuggestiveofappendicitisincludeanoncompressibleappendix,
tendernesswhilescanningtheappendix,andappendicealwallhyperemiaonDoppleranalysis.
D.Anappendicolithisacalcifiedconcretionthatformswithinthelumenoftheappendix,andwhen
presentishighlysuggestiveofappendicitis.Inthiscase,thereisnoappendicolith.
E.Ontheselectedimages,thereisnopneumoperitoneumorwellformedperiappendicealabscess
tosuggestperforation.
Notes

5.A14yearoldboypresentstotheemergencydepartmentwithsevereleftscrotalpain.While
hehadbeenexperiencingpainoffandonforaboutaweek,thepainisnowmuchworseandhas
beenconstantforabout3hours.Anultrasoundisordered.Whichofthefollowingstatementsis
true?
A.Bothtestesaresymmetricinsize.
B.Whilethereisnowaveforminthesampledareaofthelefttesticle,thereisevidenceof
bloodflowbasedonthepowerDoppleranalysis.
C.Thesonographicfindingsaresuggestiveofneoplasm.
D.Iftheinvolvedtesticleisfoundtobeviable,thepreferredsurgicaltreatmentis100%
effectiveinpreventingrecurrence.
E.Nofurtherimagingisnecessary.

TesticularTorsion

5.TheanswerisE:Nofurtherimagingisnecessary.Thesonographicimagesare
diagnosticoflefttesticulartorsion,andnofurtherimagingshoulddelaypromptintervention.
Thelefttesticleismarkedlylargerthantheright,andthereisnobloodflowdemonstratedon
bothpowerandspectralDoppleranalysis(arrows).
Testiculartorsionoccurswhenthetestisandspermaticcordtwist,eitherspontaneouslyorsecondary
totrauma,causingthevascularpedicletotwistaswellandresultinginvascularcompromise.
Viabilityofthetestisdependsonthedegreeoftorsionandthetimeofonsetofpaintointervention,
withan80%chanceofsalvagewithin6hoursanda0%chanceafter12hours.Thereisabimodal
peakageofincidenceininfantsandadolescents.Upto50%ofpatientsreportahistoryofsimilar
symptomsresolvingspontaneously.
A.Thelefttesticleismarkedlylargerthantheright.
B.ThereisnobloodflowdemonstratedonbothpowerandspectralDoppleranalysis.
C.Thereisnoneoplasmbysonographicfindings.
D.Whilemanualdetorsioncanbeattempted,surgeryisthedefinitivetreatment,andifthetesticleis
foundtobeviable,orchidopexyishelpfulinpreventingrecurrenttorsionbutnotfailsafe.Ifthe
testicleisnotviableatsurgery,itshouldberemoved,becausethebloodtestisbarriermaypotentially
breakdown,allowingthepatienttodevelopimmunityagainsthisownsperm,thusloweringhis
fertility(thesocalledantispermantibodytheory).
Notes

6.A60yearoldwomanpresentstoherprimarycarephysicianwithcomplaintsofintermittent
mildbonepain,especiallyintheback.Shehaslostweightsinceherlastvisit.Sheisfoundtobe
mildlyanemicandtohavemildrenalinsufficiencyonlaboratorytests.Furthertestingofthe
patient'surinerevealsthepresenceofBenceJonesproteins.Sheissentformagneticresonance
imaging(MRI)ofthespine.SheinformstheMRItechnologistthatshelongagohadapieceof
metalstuckinhereyeafteracaraccident.Alateralradiographoftheskullisperformedto
evaluateformetallicforeignbodyintheorbitspriortotheMRI.Whichofthefollowing
statementsistrue?
A.Thepatientlikelyhaselevatedserumcalcium.
B.Thereisintraorbitalmetal,anabsolutecontraindicationtoanMRI.
C.Findingsontheradiographsuggestaprocessinvolvingoveractiveosteoblastsand
suppressedosteoclasts.
D.Ifthisdiseaseinvolvesthelumbarspine,MRItobeperformedisnotsensitivein
identifyinglesions.
E.Thepatient'sprognosisisfavorable.

MyelomaInvolvingtheCalvarium

6.TheanswerisA:Thepatientlikelyhaselevatedserumcalcium.Thelateral
radiographoftheskullshowsmultiple,welldefined,lyticlesionsthroughoutthecalvarium
(redarrows),withapunchedoutappearance,which,especiallywiththeclinicalinformation
provided,arehighlylikelyduetomyeloma.Therearemultiplelesionsthereforethestageof
thediseaseismorelikelytobeadvanced,andthepatientlikelyhaselevatedserumcalciumdue

totheosseousdestruction.Myelomaisamonoclonalplasmacellmalignancyarisingfromthe
bonemarrow.Myelomaisthemostcommonprimaryboneneoplasm.Commonsitesof
involvementofmyelomaincludethespine,ribs,pelvis,skull,femur,andhumerus.Myelomais
primarilytreatedwithchemotherapy.
B.Therearenometallicforeignbodieswithintheorbitsontheradiographprovided,sowithoutany
otherobviouscontraindications,thepatientissafetohavetheMRI.
C.Theneoplasticplasmacellsinmyelomareleasecytokinesthatincreaseosteoclasticactivityand
suppressosteoblasticactivity,ultimatelyleadingtothelucentlesionswithlittletonoassociated
sclerosis(i.e.,newboneformation).
D.MRIisthebestimagingtoolwhenevaluatingforanymarrowreplacingprocess,suchasmyeloma
ormetastases,thetwomostcommonbonemalignanciesinadults.ThereforetheMRItobe
performedinthiscasewouldbehighlysensitivefordiseaseinvolvingthespine.
E.Despitetherapeuticadvancements,myelomacontinuestobeafataldisease,especiallyatan
advancedstage.
Notes

7.A48yearoldmanpresentedtohisprimarycarephysicianwitha6monthhistoryof
graduallyincreasedawarenessofsuddenrandommuscularjerking.Hiswifereportsthathis
thinkingseemstobemorecloudedandtherearepersonalitychanges.TheMRIdemonstrated
symmetricsignalabnormalitywithinthecerebralcortex,caudateheads,andputamenonthe
T2anddiffusionweightedimages.Themostlikelydiagnosisis:
A.Multiplesclerosis
B.Encephalitis
C.CreutzfeldtJakobdisease(CJD)
D.Multiinfarctdementia

CreutzfeldtJakobDisease(CJD)

7.TheanswerisC:CreutzfeldtJakobdisease(CJD).Thepatternofhyperintensesignal
onT2(topleft)anddiffusionweightedimaging(bottomleft,topright)withinthebilateral
putamenandcaudatehead(straightarrows)aswellasthebilateralparamediangyri(curved
arrows)isclassicforCJD.ThethreemajorcategoriesofCJDaresporadic,hereditary,and
acquired.Thesporadicformisthemostcommon,accountingforatleast85%ofcases.The
hereditaryformwillhaveafamilyhistoryofthediseaseandtestpositiveforageneticmutation.
Theacquiredformdemonstratesseveralvariantsubtypes,withvariablesymptomsandcourse
ofthedisease.ThesporadicandvarianttypesofCJDaremorelikelytodemonstratesignal
abnormalitywithinthethalamus,thesocalledpulvinarsign.
A.MultiplesclerosisshoulddemonstrateT2signalabnormalityinvolvingtheperiventricularand
subcorticalwhitematter,notthegraymatterasseenhere.
B.EncephalitisshoulddemonstrateT2signalabnormalityinvolvingboththegrayandwhitematter
andshouldnotbesymmetric.
D.Multiinfarctdementiashoulddemonstratemultipleareasofoldinfarctswithvolumelossand
gliosis.Whiletheremaybefocalareasofdiffusionrestrictionrepresentingacuteinfarcts,itwould
notdemonstratethesymmetryseenhere.
Notes

8.An18yearoldwomanpresentstotheemergencydepartmentcomplainingofchest
discomfort.Onphysicalexamination,sheismildlytachypneicandtachycardic,withafeverof
101F.Herpastmedicalhistoryissignificantforaseizuredisorder.Herchestxrayisabove.
Whatisthemostlikelydiagnosis?
A.Fungalpneumonia
B.Squamouscellcarcinomaofthelung
C.Lungabscess
D.Pulmonaryembolus
E.Wegenergranulomatosis
9.Whatadditionalstudiesareindicatedatthistime?
A.CTscanofthechest
B.CTscanofthechest,abdomen,andpelvis
C.MRIscanofthechest
D.MRIscanofthebrain
E.Ventilation/perfusionscanofthelungs

LungAbscess

8.TheanswerisC:Lungabscess.Thechestxrayshowsa6cmmasslikestructureinthe
leftlung.Themasshastypicalfeaturesofanabscess,includinganareaofcentralcavitation
withanairfluidlevel(arrow).Thepatient'sunderlyingseizuredisorderputsheratsignificant
riskofaspiration,whichwasthecauseofthisinfection.
A,B,E.Thedifferentialdiagnosisofcavitarylungmassesalsoincludesfungalinfections,squamous
celltumors,andWegenergranulomatosis,amongotherlesslikelyetiologies.However,thepatient's
profile,includingheryoungageandherhistoryofseizuredisorder,rendertheseotherdiagnoses
unlikely.
D.Pulmonaryembolusmaypresentwithchestdiscomfort,tachypnea,andtachycardiabutwould
notproducealargecavitarymass.Mostpatientswithpulmonaryembolushavenormalchestxrays,
butthemostcommonabnormalitieswouldbefocalatelectasisandsmallpleuraleffusion.

9.ThecorrectanswerisA:CTscanofthechest.ChestCTisindicatedtoconfirmthex
rayfindings,toevaluateforpossibleadditionalabnormalities,andtoestablishabaseline
appearanceforfollowupcomparison.ArepresentativeimagefromtheCTstudy(above)
confirmsalargemasswithcentralnecrosisandairfluidlevel(arrow)inthesuperiorsegment
oftheleftlowerlung.Noadditionalabscessesarepresent.
B,D.Thereisnoindicationtoperformadditionalimagingoftheabdomen,pelvis,orbrainatthis
time.

E.Asthereisnoreasontosuspectpulmonaryembolism,thereisnoindicationfor
ventilation/perfusionscanning.
Notes

10.A50yearoldmanpresentstotheemergencydepartmentwithabdominalbloating,nausea,
vomiting,andnobowelmovementin4days.Pastsurgicalhistoryissignificantfor
appendectomyasayoungadult.Onphysicalexam,theabdomenisdistended,andthereisa

paucityofbowelsounds,butthereisnoreboundtenderness.Thepatient'svitalsignsarestable
andlaboratorytestsarenormal.Whilewaitingforasurgicalconsult,abdominalradiographs
areperformed.Whichofthefollowingstatementsiscorrect?
A.Thereisalargebowelobstruction.
B.Thereisasmallbowelobstruction.
C.Therearesignsofvascularcompromise.
D.CToftheabdomenisunnecessary.
E.Tumoristhemostlikelyetiology.

SmallBowelObstruction

10.TheanswerisB:Thereisasmallbowelobstruction.Thepatientpresentswith
symptomsconcerningforbowelobstruction.Theradiographsaresupportiveofahighgrade
smallbowelobstruction.Thecolonisnotvisualized,andthedilatedloopsareclearlysmall
bowel(arrows),asevidencedbythepresenceofvalvulaeconniventes.
Radiographicfindingssupportiveofsmallbowelobstructionincludedilatedsmallbowelloops,
presenceofvalvulaeconniventes,multipleairfluidlevelsonerectordecubitusviews,andarelative
absenceofairandstoolthroughoutthelargebowel.Thefirstcourseofactionshouldbe
decompressionviaplacementofanasogastrictube.Giventhefluidfilledstomach,thepatientwill
verymuchappreciatethisprocedure.Complicationsofsmallbowelobstructionincludebowel
strangulation,perforation,andsepsis.Prognosisofanuncomplicatedobstructionisgood.Atrialof
conservativemanagementwithbowelrest,decompression,andhydrationmayprovehelpfulinlow
gradeobstructions.
A.Thecolonisnotvisualized,andthedilatedloopsareclearlysmallbowel,asevidencedbythe
presenceofvalvulaeconniventes.
C.Therearenoobviousradiographicfindingsofpotentialvascularcompromise(e.g.,volvulus,
pneumatosis,pneumoperitoneum).
D.CToftheabdomenisindicatedinthisscenarioinordertohelpidentifythecause,gradethe
severity,andofferinformationforsurgicalplanning.
E.Themostcommoncauseofsmallbowelobstructionisadhesions,followedbyhernias,andthen
tumors.
Notes

11.A23yearoldmanpresentstotheemergencydepartmentwithseverecolickyrightflank
pain,whichradiatestothegroin.Heisafebrile,andhisvitalsignsarestable.Aurinalysis
revealshematuria.ACToftheabdomen(above)andpelvis(p.196)isobtained.Whichofthe
followingstatementsistrue?
A.Intravenous(IV)contrastshouldhavebeenadministered.
B.Thecauseofpainismostlikelymusculoskeletalinorigin.
C.Thisdiseaseprocessismorecommoninwomen.

D.Radiographsareconsideredsufficienttomakethisdiagnosis.
E.Thereisobstructiveuropathyontheright.

ObstructiveUrolithiasis(additionalfiguresonp.198)

11.TheanswerisE:Thereisobstructiveuropathyontheright.TheCTimages
provideddemonstrateanobstructingcalculusintherightureterovesicularjunction(red
arrow),under10mminsize.Thereisresultinghydroureter(bluearrow),hydronephrosis

(yellowarrow),enlargementoftherightkidney,andrightperinephricfatstanding.
Urolithiasisshouldbeconsideredverylikelyinpatientswhopresentwithacutecolickyflankpain
thatradiatestothegroin.Distalureteralcalculilessthan10mminsizecanpassspontaneously.
Ultrasoundisthepreferredinitialimagingtestinsuspectedurolithiasisinpregnantpatients.Inthe
absenceofcomplicationssuchasrenalfailure,anuria,orurosepsis,hydrationandanalgesiawith
oralpainmedicationsisusuallytheinitialtreatmentinpatientswithsmallstones.

A.Inadultswithsuspectedurolithiasiswithacutesymptoms,CToftheabdomenwithoutcontrastis
thepreferredstudyofchoice.ThesensitivityandspecificityofanoncontrastCTinidentifying
ureterolithiasisare95%and98%,respectively.BecauseamajorityofcalculiarehyperdenseonCT,a
urographicCTphasewithIVcontrastisrarelynecessaryandusuallyreservedforspecialscenarios,
suchasidentifyingradiolucentstonesinpatientsonidinavir(aproteaseinhibitor).
B.Besidestheureteralstonebeinganobviouscauseofthepatient'ssymptoms,thereisnoevidence
providedontheCTimagesthatthereisamusculoskeletalabnormality.
C.Urolithiasisisthreetimesmorecommoninmen.
D.Despiteoldteachingsthat90%ofrenalandureteralcalculicanbeseenonradiographs,andthe
factthatmoststonesarecomposedofcalcium,comparisonstudieswithCThaveshownthatthe
sensitivityofradiographsisactuallyabout60%.
Notes

12.A65yearoldwomanpresentstoherprimarycarephysicianaftertrippinginherdriveway
andiscomplainingofleftwristpainandswelling.Sheisreferredtothelocalimagingcenterfor
radiographsoftheleftwrist,andshereturnstoherdoctorwithoneradiographfromthestudy.
Whichofthefollowingstatementsistrue?
A.Nootherradiographicviewsarenecessaryinthiscase.
B.Thisisnottheexpectedlocationofinjury,giventhepatient'sage.
C.Whenthepatientfell,shelikelylandedonthepalmofherdorsiflexedhand.
D.Mostoftheseinjuriesrequiresurgery.

E.Anunderlyingadvancedinflammatoryarthropathypredisposedthepatienttothis
injury.

DistalRadialFracture

12.TheanswerisC:Whenthepatientfell,shelikelylandedonthepalmofher
dorsiflexedhand.Thispatienthassufferedafractureofherleftdistalradius(redarrow).
Thesingleposteroanterior(PA)radiographshowsthecomminutedimpactedfracture,which
hasanintraarticularcomponentextendingtotheradiocarpaljoint.
Inpeopleapproximately40yearsoldandolder,themostcommonfractureatthewristthatresults
fromafallonanoutstretchedhand(FOOSH)whenthehandisdorsiflexedisthedistalradial
fracture.Therearemanytypesofdistalradialfractures,themajoritygiveneponyms,andthemost
commonbeingtheCollesfracture,inwhichthereisatransversefracturethroughthedistalradius
andapexvolarangulation.Collesfracturestendtooccurinosteoporoticbone.Distalradialfractures
arelargelyduetoaxialcompressionwithvariabletensileforces,especiallyinthesettingofFOOSH.
A.Whilethereappearstobenodisplacementorangulationonthissingleview,atleastalateralview
isnecessarytofullycharacterizethefractureandguidetherapy.
B.Inpeopleapproximately40yearsoldandolder,themostcommonfractureatthewristthat
resultsfromaFOOSHwhenthehandisdorsiflexedisthedistalradiusfracture.
D.Mostdistalradialfracturesaretreatedwithreductionandimmobilization,andsurgeryisnot
needed.
E.Thereisnoevidenceontheradiographofanunderlyingadvancedinflammatoryarthropathy,
suchasjointspacenarrowing,erosions,orjuxtaarticularosteopenia.

Notes

13.A47yearoldwomanpresentedtoherprimarycarephysician'sofficewitha2weekhistory
ofincreasingweaknessofherrightfaceandnoticedyesterdaythatshecouldnotfullycloseher
eyelid.Imagingwasobtained.Themostlikelyetiologyforthepatient'ssymptomsis:
A.Facialnerveschwannoma
B.Bellpalsy(facialneuritis)
C.Vestibularschwannoma

D.Acuteerosiveotomastoiditis

BellPalsy

13.TheanswerisB:Bellpalsy(facialneuritis).PostcontrastT1weightedimaging
demonstratedenhancementoftherightfacialnervethatincludedthetympanicsegmentand
geniculateganglion(curvedarrow)withextensionintotheinternalauditorycanal(IAC)
(straightarrows).Thefacialnerveisbrokendownintoanatomicsegments:themeatalsegment
isfromthebrainstemtotheIACthelabyrinthinesegmentisfromthefundusoftheIACto
facialhiatusthetympanicsegmentisfromthegeniculategangliontothepyramidaleminence

themastoidsegmentisfromthepyramidalprocesstothestylomastoidforamenandthe
extratemporalsegmentisfromthestylomastoidforamentothepesanserinus.
Thefacialnervemaydemonstratemildenhancementnormallyuptoandwithinthegeniculate
ganglionandtympanicsegment,butenhancementshouldneverextendmoreproximallythroughto
thelabyrinthinesegmentandintotheIAC.Theappearanceofthinwispyenhancementextending
intothefundusoftheIACisconsistentwithBellpalsy.
A.Facialnerveschwannomacanbeacauseofslowlyprogressivefacialweaknessratherthanthe
acutepresentationseeninthispatient.Ittypicallypresentsasanodularenhancingmassanywhere
alongthenervecourse.Thesymptomswoulddependonthesegmentofnerveinvolved.
C.Vestibularschwannomatypicallypresentsinapatientwithsensorineuralhearingloss
demonstratingnodularenhancementwithintheIACandfrequentlyextensionintothe
cerebellopontineanglewithwideningoftheporusacusticus.
D.Acuteerosiveotomastoiditisshouldpresentwithadditionalsymptomsofpain,fever,andpossibly
conductivehearingloss.Thereshouldbeabnormalsignalandenhancementwithinthemastoidair
cells,notseenintheseimages.
Notes

14.A23yearoldmanpresentstotheemergencydepartmentwithsymptomsofmalaise,dry
cough,anddyspneaforseveralweeks.Physicalexaminationrevealstachypnea,tachycardia,

andfever,withcracklesonauscultation.Onfurtherquestioning,thepatientadmitstoIVdrug
abuse.Thechestxrayfindings(top)promptthecliniciantoorderachestCTstudy,fromwhich
isarepresentativesectioninthecoronalplaneisshowninthebottomfigure.Whatisthemost
likelydiagnosis?
A.Pneumocystispneumonia(PCP)
B.Pneumococcalpneumonia
C.Miliarytuberculosis(TB)
D.Cytomegalovirus(CMV)pneumonia
E.Pulmonaryedema

HumanImmunodeficiencyVirusPneumocystisPneumonia(HIVPCP)

14.TheanswerisA:Pneumocystispneumonia(PCP).ThechestxrayandchestCT
showadiffusegroundglasspatternthroughoutthelungs.Thetermgroundglassreferstothe
edgeofamicroscopeslidethatcanbewrittenonwithpencil,anditmeansthatalthoughthe
arearemainstransparent,onecannotseedetailthroughit.Thispatternischaracteristicof
pneumocystispneumonia,commonlyknownasPCP.ThepatienttestedpositiveforHIV
infection,likelyacquiredthroughthesharingofcontaminatedneedles,andheisseverely
immunosuppressed.SputumobtainedduringbronchoalveolarlavagewaspositiveforPCP.

PCPiscausedbyPneumocystisjirovecii,anendemicfungusthatisharmlesstoimmunocompetent
individuals,butitisthemostcommoncauseofopportunisticinfectioninpatientswithHIV,typically
occurringwhenCD4countsfallbelow200/L.Althoughtheorganismisclassifiedasafungus,
treatmentiswithantibiotics,trimethoprimsulfamethoxazole,for21days.Despiteantibiotic
treatment,mortalityratesrangebetween10%and20%.
B.Pneumococcalpneumoniatypicallyappearsasadensezoneoflungconsolidation,ratherthana
hazygroundglasspattern.Pneumococcalpneumoniamayoccupyanentirelunglobebutisseldoma
diffuse,bilateralprocess.Patientswithoutaspleenaremoresusceptibletopneumococcalinfection,
becausetheorganismisencapsulatedandusuallyopsonizedinthespleen.PatientswithHIV,but
stillwithoutclinicalacquiredimmunedeficiencysyndrome(AIDS),haveahigherincidenceof
pneumococcalinfectionthanthegeneralpopulation.
C.MiliaryTBcharacteristicallyappearsasadiffusemicronodularpatterninthelungs,ratherthana
groundglasspattern.Thedescriptivetermmiliaryisderivedfromtheappearanceoftinymillet
seedswhichareusedasbirdfeed.MiliaryTBisamanifestationofTBbacteremiaandcanoccurin
bothprimaryTBinfectionandinreactivationorsecondaryTB.TBhasbeenconsideredanAIDS
definingillnessinpatientswhoareHIVpositive.
D.CMVpneumoniaisanopportunisticcytomegaloviralinfectionthatoccursinimmunosuppressed
individuals,butittypicallyappearsasamicronodularpattern,similartomiliaryTB.
E.Pulmonaryedemaoftenappearsasagroundglasspatternonchestxray,butgravitydependence
ischaracteristic,withrelativesparingoftheupperlungs.Patientsinpulmonaryedemashould
manifestotherevidenceoffluidoverloadorcongestiveheartfailure,suchasvascularcongestionand
pleuraleffusions.
Notes

15.A33yearoldwomanpresentstotheemergencydepartmentwithalteredmentalstatusand
severeheadachefollowingaminormotorvehicleaccident(MVA)inwhichshewasthedriver.
Thepatienthasnomemoryoftheincident.TheCTscanisabove.Themostlikelyexplanation
oftheabovefindingis:
A.Posttraumaticsubduralhemorrhage
B.Posttraumaticsubarachnoidhemorrhage
C.Incidentalcalcificationwithinthesubarachnoidspace
D.Subarachnoidhemorrhage(SAH)secondarytoruptureofananeurysm

Subarachnoidhemorrhage(SAH)

15.TheanswerisD:Subarachnoidhemorrhage(SAH)secondarytoruptureofananeurysm.
Theappearanceisconsistentwithhemorrhageinthesubarachnoidspace(arrow).The
distributionwithintheleftsylvianfissurepointstoarupturedaneurysminvolvingtheleftMCA
region.CTangiography(CTA)ordirectangiographyshouldbeperformedurgentlytoidentify
thelocationoftheaneurysmandtreatitwitheitherangiographicallydirectedcoilingor
placementofavascularclip.
A.Posttraumaticsubduralhemorrhageisincorrect,asthelocationofthehemorrhagewithinthe
sylvianfissureisconsistentwithasubarachnoid,notsubdural,location.
B.WhileSAHmaybepresentfollowingsignificanttrauma,theMVAwasdescribedasminor.The
distributionofbloodinthesylvianfissureismoretypicalforananeurysmruptureresultinginaloss
ofconsciousnessleadingtoherminorMVA.

C.Idiopathiccalcificationcanoccur.However,calcificationisalmostalwaysrelatedtoaknown
previousinsult,andthedensityistypicallymuchgreaterthanthatseenonthisimage.
Notes

16.A25yearoldwomanisinvolvedinalowspeedMVAandistakentotheemergency
department.Sheisfoundtobestable,butiscomplainingofdiffuseabdominalpain,and
thereforeaCToftheabdomenisobtained.Whichofthefollowingstatementsregardingthe
imageprovidedistrue?
A.Thereisactivehemorrhageinthepelvis.
B.Ifleftuntreated,thesalientabnormalitycanresultinseverecomplications.
C.Thereisnomalignantpotentialoftheidentifiedabnormality.
D.Thisabnormalityhasdevelopedacutely.
E.Thisabnormalityisfoundonlyinthepelvis.

OvarianTeratoma

16.TheanswerisB:Ifleftuntreated,thesignificantpelvicabnormalitycanresult
inseverecomplications.TheCTimageprovideddemonstratesaroundmassintheright
hemipelvis,composedofcentralcalcification,surroundingsofttissue,andlessdensefat

layeringanteriorly(arrow).Thesefindingsarediagnosticofanovarianteratoma.Despitebeing
currentlyasymptomatic,thisteratomacouldpotentiallyrupture,causingachemicalperitonitis
and/oradhesions,actasaleadpointinovariantorsion,causeinfertilityduetomasseffecton
thefallopiantubeoruterus,orrarelydegenerateintoamalignantteratoma,necessitating
surgicalresectionwithhopetopreservetheovary.
Amatureovarianteratomaisabenignneoplasmdevelopingfromatleasttwoofthethreeprimordial
germlayers.Teratomasarethemostcommonovarianneoplasmsinwomenyoungerthan20years
old.Ovarianteratomascanbebilateralinupto15%ofpatients.Ultrasound,CT,andMRIareall
potentiallyusefulmodalitiesindistinguishingteratomasfromotherovarianneoplasms.
A.Thereisnoevidenceofhemorrhageontheimageprovided.
C.Anovarianteratomamaydegenerateintoamalignantteratoma,thoughitisrare.
D.Thisteratomawaslikelypresentinthispatientsincebirth,albeitsmalleratthattime.
E.Inorderoffrequency,teratomascanarisefromthesacrococcygealregion,gonads(ovarianand
testicular),mediastinum,retroperitoneum,andintracranially(e.g.,pinealgland).
Notes

17.A55yearoldwomanwithahistoryofbreastcancerisfoundtohaveanelevatedalkaline
phosphataseonrecentbloodworkandissentforabonescan.Basedonthefindingsofthe
bonescan,radiographsareobtained.Basedontheimagesprovided,whichofthefollowing
statementsistrue?
A.Thereisasolitarysuspiciousfindingonthebonescan.
B.Theradiographsaremostcompatiblewithanoldhealedfracture.
C.Bonescansareasensitivemodalityforbreastcarcinomametastases.
D.Thereisabnormalactivityinthepelvisonthebonescan.
E.Thefindingsonthebonescanandradiographsaredisconcordant.

LyticBoneMetastasisinBreastCancer

17.TheanswerisC:Bonescansareasensitivemodalityforbreastcarcinoma
metastases.Theimagefromthebonescanaboveshowsincreasedradiotraceruptakeinthe
distalrighthumerusaswellasintheproximallefttibia(redarrows).Giventhehistoryof
breastcancerandanelevatedalkalinephosphatase,thesefociareveryconcerningfor
metastases.Bonescansareverysensitiveforosseousmetastases,particularlyosteoblastic
metastases,andareusedasscreeningtoolsinpatientswithcertaincancers,suchasbreast,
prostate,andlung.
Breastcancercommonlymetastasizestobone,andcanbeeitherosteoblasticorosteolyticinnature.
The10yearsurvivalrateofstageIV(metastatic)breastcancerisapproximately10%.Inpatients
withsuspiciousfindingsonabonescanscreeningformetastases,radiographsand,ifnecessary,MRI

areappropriateimagingstudiesforfurtherevaluation.Bonescansareveryusefulinmonitoring
responsetotherapyinwidespreadmetastaticdiseasetothebone.
A.Asmentionedabove,therearetwosuspiciouslesionsonthebonescan:onthedistalright
humerusandtheproximaltibia.Thereisnoabnormalactivityelsewhereinthebones.
B.Apathologicfracturemaybepresentonthelateralview,butisdifficulttoevaluateforwithabsent
cortex.Thereisnohealedfracture.
D.Thebonescanradiotracer(Tc99mMDP)isexcretedviathekidneysthusactivityfromurine
withinthebladderandurethraandontheperineumisnormal.
E.Theradiographsshowapermeative,expansile,lyticprocessdestroyingthedistallefthumerus
(whitearrows),concordantwiththebonescanfindingsandcompatiblewithmetastasis.
Notes

18.A25yearoldmanpresentstotheemergencydepartmentwithahistoryofelbowtothe
headduringabasketballgame.Helostconsciousnessforafewminutes.Uponregaining
consciousness,hewasalertandoriented,complainingofaheadache.Bythetimehearrivedvia
ambulance,hecomplainedofsevereheadacheandwasvomiting.ACTscanoftheheadwas
orderedanddemonstratedtheabovefindings.Thereisahyperdense,biconvexcollectioninthe
leftparietalregionwithasubtle,nondisplacedfractureoftheleftparietalbone.The
appearanceaboveismostconsistentwith:
A.Anacutesubduralhematoma
B.Asubacutetochronicsubduralhematoma
C.Anacuteepiduralhematoma
D.Anacuteepiduralabscess

EpiduralHematoma

18.TheanswerisC:Anacuteepiduralhematoma.Thehistoryoftraumacausinglossof
consciousnesswithalucidintervalfollowedbyarapiddeteriorationinhisclinicalcourseis
typical.Epiduralhematoma(straightarrow)isusuallycausedbyfocaltraumatothe
temporoparietalregionresultinginafractureandtearingofthemiddlemeningealarteryor
oneofitsduralbranches.OnCT,thebloodishyperdense,lenticular,orbiconvexinshapeand
doesnotcrosssuturelines.Thereisalmostalwaysafractureassociatedwithit,asinthiscase
(curvedarrow).Itisasurgicalemergencyand,ifuntreated,isassociatedwithahighmortality
rate.
A.Anacutesubduralhematomaisfrequentlyassociatedwithtrauma.Theclinicalpresentationofa
lucidintervalisnottypical.Thesourceofthehematomaisvenousratherthanarterial,sothereis
usuallyamoreindolentclinicalcourse.Onimaging,itfrequentlycrossessuturelinesasacrescentic
hyperdensemass.
B.Asubacutetochronicsubduralhematomawouldappearasanisodensetohypodensecrescentic
massalongtheinnertableofthecalvarium.Itistypicallyassociatedwitheitheraremotetrauma
historyornoknowntraumahistory.
D.Anacuteepiduralabscesswouldnotpresentwiththishistory.Itisrareintheabsenceof
intracranialsurgeryandtypicallyhasaninsidiousonset.OnCT,anabscessshouldbehypodense
ratherthanhyperdense.
Notes

19.A70yearoldwomansuffersafallathomeandisbroughttotheemergencydepartmentby
aneighbor.Sheisunabletowalkandcomplainsofpaininherlefthip.Radiographsare
obtainedwhilesheisinthewaitingroom.Whichofthefollowingstatementsistrue?
A.Thisinjuryistheleastcommonatthisanatomiclocation.
B.Thisinjuryisoftenrelatedtooveruse.
C.Thereisapathologicfracture.
D.Thereisanincreasedriskofavascularnecrosissecondarytothisinjury.
E.Thisinjurydoesnotrequiresurgery.

SubcapitalFemoralNeckFracture

19.TheanswerisD:Thereisanincreasedriskofavascularnecrosissecondarytothisinjury.
Thispatienthassufferedadisplacedimpactedsubcapitalfractureoftheleftfemoralneck(red
arrows).Femoralneckfracturesareintracapsularwithanincreasedriskofavascularnecrosis
frompotentialdisruptionofbloodsupplytothefemoralhead.
Femoralneckfracturesarecommonlysustainedbyelderlypeople,oftenrelatedtoseeminglymild
falls.Thepropensityforthehiptoflexandinternallyrotatewhenthefemoralneckisfracturedcan
obscuresubtlefracturelinesonradiographs.Ifafractureissuspectedandradiographsarenegative,
MRisthenextmostappropriateimagingstudytoperform.CTismuchlesssensitivefor
nondisplacedfemoralneckfractures,usuallybecauseoftheunderlyingosteoporoticbone.Evenwith
surgicalfixation,theprognosismaybepoorduetocomorbiditiessuchasvenousthromboembolic
diseaseandpneumonia.
A.Femoralneckfracturesaredescribedbylocation:subcapital,midcervical,andbasicervical.The
subcapitalfractureisthemostcommon.Anintertrochantericfractureisconsideredaproximal
femoralfracture.
B.Insufficiencyfracturescommonlyoccurintheelderly,whereasoverusefracturescommonlyoccur
inyoung,extremelyactivepeople.
C.Thisfractureissimple(i.e.,twofragments)withnoevidenceofanunderlyingpathologicprocess,
suchasmetastasis.

E.Thetreatmentforthisfractureisinternalfixation.
Notes

20.A60yearoldwomanpresentstoherprimarycarephysicianwithweightlossandiron
deficiencyanemia.Fecaloccultbloodtestsarepositive.Adoublecontrastbariumenemais
performedforfurtherevaluation.Whichofthefollowingstatementsistrueregardingthis
disease?
A.Commonriskfactorsincludeahighfiber,lowfat,andanimalproteindiet,
inflammatoryboweldisease,andapositivefamilyhistory.
B.Thislocationisthemostcommonsiteforthisentity.
C.Thetreatmentrequiressurgicalresection,possiblywithadjuvantchemotherapy.
D.Theselesionsarenotassociatedwithbenignpolyps.
E.Thisconditiondoesnotusuallypresentwithobstruction.

AdvancedColonCarcinomaAnnularConstrictingLesion(i.e.,Apple
CoreLesion)

20.TheanswerisC:Thetreatmentrequiressurgicalresection,possiblywith
adjuvantchemotherapy.Thisvignettedescribesaclassicpresentationofcoloncancerwith
thepeakageatpresentationof50to70yearsoldandthemostcommonpresentations
includingweightloss,changeinbowelhabits,melena/hematochezia,andirondeficiency
anemia.ColoncanceristhemostcommonGImalignancyandthesecondmostcommoncause
ofcancerdeaths.Coloncancercanbeidentifiedonabariumenemaasasessileplaque,a
pedunculatedlesion,alargepolypoidlesion,asemiannularorsaddlelikelesion,anannularor
applecorelesion,orwallthickeningresemblingacarpetlikelesion.
Thespotradiographsabovedemonstratecircumferentialnarrowinginthedistaltransversecolon,
withshelflikeborders,resemblinganapplecoreandrepresentinganadvancedannularconstricting
tumor(arrows).Thebariumenemahasa90%95%detectionrateforcoloncancer,whichis
comparablewiththatofbothcolonoscopyandCTcolonography.Theadvantagetothebariumenema

overcolonoscopyinscreeningisthenoninvasiveness.Theadvantageofcolonoscopyinscreeningis
theabilitytoremovepolypsduringtheprocedure.BothCToftheabdomenandpelvisandwhole
bodypositronemissiontomography(PET)CTareusefulmodalitiesininitialstagingofcoloncancer,
particularlyinevaluatingforlymphnodeandlivermetastases.
A.Commonriskfactorsincludealowfiber,highfat(notlow),andanimalproteindiet
inflammatoryboweldiseaseandapositivefamilyhistory.Geneticallybasedsyndromesassociated
withanincreasedriskofcoloncancerincludefamilialadenomatouspolyposis,Gardnersyndrome,
andhereditarynonpolyposis.
B.Thesigmoidcolonisthemostcommonlocation,followedinorderbytherectum,transverse,
descending,andascendingcolon,andcecum(thelattertwobeingmorecommoninadvancedages).
D.Mostcommonly,coloncanceristheresultofmalignantdegenerationofabenignadenomatous
polyp.
E.Fiftypercentofcolonicobstructionsarecausedbycoloncancer,withotheracutelifethreatening
complicationsbeingintussusceptionandperforation.
Notes

21.Anasymptomatic,50yearoldwomanpresentsforpreoptestingpriortogeneralanesthesia
forkneereplacementandhasthechestxrayshownabove.Whatisthemostappropriatenext
step?
A.Bariumswallow

B.CToftheneckandchestwithcontrast
C.CToftheneckandchestwithoutcontrast
D.Ultrasoundoftheneck
E.Noadditionalimagingisneededbeforekneereplacementsurgery.

ThyroidGoiter

21.TheanswerisC:CToftheneckandchestwithoutcontrast.Thenextmost
appropriatestepwouldbeCToftheneckandchestwithoutcontrasttoevaluatealarge
substernalgoiter.
Thechestxrayshowsalargemass(M)occupyingthesuperiormediastinumandextendingintothe
neck(arrowheads).Themassdisplacesthetracheatotheright(arrow)andnarrowsthetracheal
lumenalongmostofitslength.Theappearanceistypicalofalargesubsternalgoiter.Further
evaluationoftheenlargedthyroidanditsrelationshiptothetracheaandgreatvesselsisnecessary
priortoelectivesurgeryundergeneralanesthesia.Ideallythepatientshouldbeeuthyroidatthetime
ofsurgery,soserumthyroidfunctiontestsandconsultationwithanendocrinologistarealso
indicated.
A.Abariumswallowmaybehelpfulinevaluationoftheneckandesophagusinpatientswhoare
experiencingdifficultyswallowing.However,abariumswallowwouldnotprovidedetailed
evaluationofthethyroidtissueortheextentofanyencroachmentonthetracheaandgreatvessels.

B.CTimagingisidealforevaluatingtheextentandthecharacterofthethyroidmass,thepatencyof
thetrachea,andanyencroachmentonthegreatvesselsinthesuperiormediastinumandtheneck.
However,theuseofiodinatedcontrastisrelativelycontraindicatedwhenevaluatingthethyroid
becausethedyeloadsaturatesthethyroidglandwithiodine.Thiswouldinterferewiththeuptakeof
radioactiveI131forseveralweeksifthereisasubsequentneedforI131thyroidscanningorI131
radiationtherapy.
D.Ultrasoundwouldprovideinsufficientimagingofthislarge,substernalgoiter.Ultrasoundofthe
neckismostappropriateforevaluationofamildlyenlargedglandthatdoesnotextendintothe
mediastinumorofapalpablethyroidnoduletodetermineifitiscysticorsolid.Solidnodulesmay
thenbefurtherevaluatedwithI131scanning.Benignadenomastakeupiodineandappearhoton
I131thyroidscans,whereasmalignantnodulesdonottakeupiodineandappearcold.
E.Furtherimagingpriortoelectivesurgeryisnecessarytoevaluateanyeffectthisveryenlarged
thyroidglandmayhaveonthepatient'sairwayandgreatvessels.Iftherewereaneedforemergency
surgery,ratherthanelectivesurgery,anawakeendoscopicintubationmaybenecessarytoensurea
secureairwaypriortoinductionofanesthesia.
Therearemanycausesofthyroidgoiter,andsomeofthemorecommonetiologiesaredetailed
below:
Themostcommoncauseofgoiterworldwideisdietaryiodinedeficiency.However,the
commonpracticeofiodinesupplementationinsaltmakesthisalesslikelycauseintheUnited
States.
Gravediseaseisadiffuse,benignenlargementofthethyroidgland.Gravediseaseiscaused
byautoimmuneoverstimulationofthethyroidthatleadstothyroidhypertrophy,
overproductionofthyroidhormone,andsymptomsofhyperthyroidism.
Hashimotodiseaseiscausedbyautoimmunedamagetothethyroidthatresultsindecreased
thyroidhormoneproductionandsymptomsofhypothyroidism.Lowserumthyroidhormone
levelscauseincreasedthyroidstimulatinghormoneproductionbythepituitarywhich,inturn,
stimulatesthyroidgrowth.
Multinodulargoitersarecausedbythepresenceofmultiple,benignsolidandcysticnodules
throughouttheglandthatresultinoverallenlargement.
Thyroiditisisaninflammatoryconditionofthethyroidthatcausespainandgeneralized
swelling.
Severaltypesofthyroidmalignancyoccur,butthesetypicallymanifestasasolitary,hard
noduleratherthanasdiffuseglandularenlargement.Amalignantnodulewillappearasasolid
noduleonultrasoundthatiscold,ornonfunctioning,onI131thyroidscanning.CTimagingof
malignantlesionsmaybehelpfulforevaluationofregionallymphnodesandstaging.

Notes

22.An86yearoldwomanistransferredformanagementofarecentstroke.Sheisbreathing
spontaneouslybuthasdifficultyswallowing.Aportablechestxrayisobtainedfollowing
placementofaDobhofftubeforentericfeeding.Whichofthefollowingstatementsaretrue?
A.TheDobhofftubeisinsatisfactorypositioninapatientwithsitusinversus.
B.TheDobhofftubeisinsatisfactoryposition,afterpassingthroughagastricpull
throughandthenbelowthediaphragm.
C.TheDobhofftubeisinsatisfactoryposition,afterpassingthroughalargehiatalhernia
andthenbelowthediaphragm.
D.TheDobhofftubeisinthepleuralspaceandshouldberemoved.

FeedingTubeinPleuralSpacewithPneumothorax

22.TheanswerisD:TheDobhofftubeisinthepleuralspaceandshouldbe
removed.
TheDobhofftubedoesnotfollowtheexpectedanatomiccourseoftheesophagus.Instead,itpasses
intotherightmainbronchus,thentherightlowerlobebronchus,thenperforatestherightlung.The
tipisdeepintherightsulcus,inthepleuralspace,anditshouldbepromptlyremoved.
Aswouldbeexpected,thereisalsoarightpneumothorax.Thevisceralpleuralline(redarrows)is
visibleabout2cmbelowtherightupperribs,withnolungmarkingsbeyond.Therefore,prompt
chesttubeplacementisalsoindicated.
A.Thereisnositusinversus,asthegastricairbubbleisvisiblejustbelowtheleftdiaphragm,andthe
liveroccupiestherightupperabdomen.
B,C.Sometimesanasogastric(NG)tubeorDobhofftubemayfollowaserpentinecourseifthereisa
dilatedesophagus,agastricpullthroughfollowingesophagectomy,alargehiatalhernia,or
intrathoracicstomach.However,thispatientshowsnoevidenceoftheseentities.Thegastricair
bubbleisintheexpectedlocationbelowtheleftdiaphragm,andtherearenoclipsinthechestor
abdomentosuggestprevioussurgery.Thereisalsonoevidenceofhiatalhernia,andtheDobhoff
tubetipterminatesoutsidetheanatomiclocationofthestomach.
Notes

23.A35yearoldHispanicmanpresentedtotheemergencydepartmentwithahistoryof
worseningheadache.Whileintheemergencydepartment,thepatientsufferedawitnessed
simplepartialseizure.ACTscanoftheheadwasobtained.Whatisthemostlikelyetiologyfor
theaboveCTfindings?
A.Congenital
B.Infectious
C.Inflammatory
D.Metabolic
E.Toxic

Neurocysticercosis

23.TheanswerisB:Infectious.Thefindingoffocalcalcifications(arrowheads)inboththe
leftfrontallobe(imageonright)andrightparietallobe(imageonleft)isaclassicpresentation
forneurocysticercosis.Thereisasurroundinghypodensity(straightarrows)andmasseffect
involvingthecalcificationintherightparietallobe,representingedema.Thiscalcificationwith
accompanyingedemarepresentsamoreacutestageofdiseasedemonstratinganactivehost
responsewhiletheleftfrontalcalcificationrepresentsalater,inactivestage.Neurocysticercosis
iscommonindevelopingcountriesandisbeingseenmoreoftenindevelopedcountrieslikethe
UnitedStateswiththecontinuedimmigrationfromCentralandSouthAmerica.The
presentationisnonspecific,andseizureisnotuncommon.
A.Ifitwerecongenital,onewouldexpecttofindmigrationalanomalieswithgraymatter
heterotopia,whichisnotseenintheseimages.
C.Theimageontheleftdemonstratesedema,whichisexpectedinaninflammatorycondition
however,thesecondimagedemonstratesasinglecalcificationwithoutanyinflammatorychanges.
D.Mostmetabolicprocessesaffectthebrainmorediffuselyandsymmetrically.
E.Aswithmetabolicprocesses,thereistypicallyamorediffuseandsymmetricappearancetothe
abnormality.
Notes

24.A77yearoldmanhasanormalpreoperativechestxray(top,AandB)priortocoronary
arterialbypassgraft.Hishospitalcourseproceedsuneventfully,andheisdischargedfor
outpatientrehabilitation.Athis6weekpostoperativecheckup,hereportspersistentdyspnea
withoutchestpainorpalpitations,especiallywithexerciseorwhenlyingdown.TheECGshows
nochanges,andthelungssoundclearonexam.Afollowupchestxray(bottom,AandB)is
obtainedforfurtherevaluation.Whatisthemostlikelydiagnosis?
A.Congestiveheartfailure
B.Dresslersyndrome
C.Subpulmoniceffusion
D.Rightlowerlungcollapse
E.Rightphrenicnervepalsy

ParalyzedDiaphragm

24.TheanswerisE:Rightphrenicnervepalsy.ThephrenicnervesderivefromtheC3,
C4,andC5cervicalrootsbilaterallyandpasscaudallythroughtheneckandmediastinumto
innervatethetwoleavesofthediaphragm.Phrenicnervepalsycanbecausedbyinadvertent

surgicalinterruptionofthenerveduringmediastinaldissection,asinthiscase.Otherpossible
etiologiesincludetraumaor,mostcommonly,malignantinvasion.
Themostcommonsymptomisdyspneawhichisexacerbatedwithexerciseandrecumbent
positioning.Thechestxrayshowselevationoftheaffecteddiaphragm(arrows).Diagnosiscanbe
confirmedbyfluoroscopicobservationofrespiratorymotion.Theaffecteddiaphragmdoesnot
descendliketheopposite,normaldiaphragmoninspiration.Asnifftestmaneuver,inwhichthe
patientabruptlysniffs,causessharpinspirationofthenormaldiaphragm,whichincreases
abdominalpressurethatinturncausesparadoxicalelevationoftheparalyzeddiaphragm.
A.Patientswithcongestiveheartfailuremaypresentwithdyspnea.However,thechestxraywould
showevidenceofpulmonaryedemaandpleuraleffusions.
B.Dresslersyndromeisaselflimitedautoimmunephenomenonthatoccursseveralweeksafter
myocardialexposurefrombypasssurgeryormyocardialinfarction.Leftsidedchestpainrelievedby
aspirinisthecharacteristicsymptom.Thechestxraycharacteristicallyshowsasmallleftsided
pleuraleffusion.
C.Thispatienthasnopleuralfluid.Pleuraleffusionsusuallypoolinthecostophrenicangleswitha
characteristicmeniscusshape.Occasionallyfluidcollectsbelowthelung,elevatingthelungbase,and
thisisknownasasubpulmoniceffusion.DetectionmaybedifficultonthePAchestxray,although
thediaphragmcontourissubtlyabnormal,andameniscusisusuallyvisibleposteriorlyonthelateral
view.Adecubitusviewallowsthefluidtopoolalongthelateralchest,confirmingdiagnosis.
D.Thispatient'srightlowerlungremainsinflatedeventhoughtherightdiaphragmiselevatedfrom
phrenicnervepalsy.Incasesofrightlowerlungcollapse,theairlesslobeappearsasafocalincreased
densityatthemedialrightlungbaseonthePAviewandattheposteriorbaseonthelateralview.The
rightdiaphragmedgeistypicallyeffacedonthePAviewbecausethereisnorightlowerlobeairto
outlineit.
Notes

25.A33yearoldhealthymanisintheoperatingroombeingpreparedforinguinalhernia
repair.Intubationisunexpectedlydifficult,andafteratraumaticintubation,thepatientis
notedtobehypoxic.Astatportablechestxrayisobtained.Whatisthediagnosis?
A.Hyperaeration
B.Leftpneumothorax
C.Rightpneumothorax
D.Righttensionpneumothorax
E.Bilateralpneumothorax

DeepSulcusPneumothorax

25.TheanswerisE:Bilateralpneumothorax.Traumaticintubationmayleadtotearof
thetrachea,piriformsinus,oresophagus,andpneumothoraxand/orpneumomediastinum
mayresult.Oncethepatientisstabilized,furtherevaluationoftheintegrityoftheairwayand
esophagusshouldproceed.CTscanning,Gastrografinstudies,andendoscopicevaluationmay
allbehelpful.
A,B,C,D.Thethoraxishyperinflated,notfrompulmonaryhyperaeration,butfrombilateral
pneumothoraces.
Theportablechestxrayshowscompletecollapseofthedense,airlessrightlung,whichhasthe
appearanceofaspinnakersail(narrowyellowarrows).Alargerightpneumothoraxsurroundsthe
collapsedlung,anditisobviousthattherearenolungmarkingsintheperipheryofthechest.
Perhapslessobviousisthetelltaledeepsulcussign(narrowredarrow)whichappearsas
hyperlucencyattherightlungbase,withanunusuallydeepcostophrenicangle,becauseinthe
supinepositionapneumothoraxmaysometimesbemostvisibleatthelungbase,ratherthanatthe
apex.
Themediastinumisshiftedtowardtheleft,indicatingthattherightpneumothoraxisindeedunder

tension,withimminentriskofcardiovascularcompromise.Therefore,promptchesttubeplacement
torelievethepneumothoraxisimperative.
Oncloserinspection,thexrayalsoshowsevidenceofamoderatesizedleftpneumothorax(wide
yellowarrows).Theedgeoftheleftlungisdisplacedabout2cmfromtheribs,andthereisadeep
sulcussignattheleftbase,aswell(wideredarrow).
Observationoftheadditionalleftsidedpneumothoraxreinforcestheimportanceofdoingacomplete
andorganizedsearchoftheimage,evenifanobviousabnormalityispresent.Failuretoobserve
additionalabnormalitiesissometimescalledhappyeyesyndrome,whichdescribesthetendencyto
quitlookingoncethefirstobservationprovidessatisfactionofsearch.
Notes

26.Whichofthefollowingbestdescribestheinjury?
A.Toddlerfracture
B.Greenstickfracture
C.Stressfracture
D.Buckle/torusfracture
E.Plasticfracture

Buckle/TorusFractureofDistalRadiusandUlna

26.TheanswerisD:Buckle/torusfracture.Theradiographsshowbucklefracturesofthe
distalradialandulnarmetaphyses(redarrows).Thewordbuckledenotesthebucklingofthe
cortex.Thecortexcanbuckleinorout,andwhenbuckledout,itappearsasaprotuberance
alongtheotherwisesmoothcortex,ofteninacircumferentialfashion,asseeninthecaseabove.
Thewordtorusdenotesacircularprotuberance,whichhasbeenattributedtoanelevatedintact
periosteumduetosubperiostealhemorrhage.
Abuckle,ortorus,fractureisoneofmanyfracturesthatcancommonlybeseeninskeletally
immaturepatientsinwhomtheincreasedelasticityofthebone(relativetoanadult)resultsin
deformationratherthanabreak.Theballbearingruleimpliesthataballbearingwouldjumpat
thesiteofthefractureifrollingdownthecortex.Thesefracturestendtohealcompletelywith
immobilization(casting)andonlyrarelydevelopasubperiostealcorticaldefect.Bucklefracturesare
usuallyduetoaxialloading,sometimeswithanangularcomponent,andwheninthedistalradius,
especiallythemetaphysis,areoftenthemanifestationoftheskeletallyimmaturefallingonanout
stretchedhand.
Thebucklefractureisoftenincludedinthecategoryofincompletefractures(notinvolvingtheentire
cortex)othersuchfracturesincludeplastic,greenstick,stress,andtoddler.
A.Atoddlerfractureisafractureoccurringinthelegsorfeetofthosechildrenwhoarejust
beginningtoambulatewithoutothertrauma.Acommontoddlerfractureinvolvesthedistaltibia,
ofteninaspiralconfiguration.
B.Inagreenstickfracture,thecortexisusuallydisruptedorbowedononesideofthebone,theside
thatexperiencedthecausativetension,resemblingabrokengreentwig.
C.Inanearlystressfracture,thebonesubjectedtorepetitivestressmayappearnormalon
radiographs,oritmayhavebarelyvisiblehairlinefractures.
E.Inaplasticfracture,thebonebendswithoutacorticaldeformity.
Notes

27.A55yearoldwomanwithnosignificantpastmedicalhistorypresentstotheemergency
departmentwithsevere,intermittent,diffuse,abdominalpain,worseningoverthecourseofa
fewdays.Physicalexaminationispertinentforabdominaltendernessintheleftlower
quadrant.Thereisnobloodperrectum.Vitalsignsarestable,andlaboratoryvaluesare
normal.ACToftheabdomenisobtained.Basedontheimagesprovided,whichofthefollowing

statementsistrue?
A.Thesalientabnormalityiswithintherightlowerquadrant.
B.Apredisposingconditionresponsibleforanacuteprocessisidentified.
C.Thereisasmallbowelobstruction.
D.Nonsurgicalmanagementispreferred.
E.Theemergencydepartmentphysicianshouldimmediatelyconsulttheurologyservice.

IntussusceptionColocolonicwithLeadPointMass

27.TheanswerisB:Apredisposingconditionresponsibleforanacuteprocessis
identified.Thefindingsontheimagesprovidedarepathognomonicforintussusception,
colocolonicinthiscase.Onthefirstaxialimageprovided,thereisaclassictargetsigninthe
leftlowerquadrant,withabowelwithinbowelappearanceperipherally,morecentral
mesentericfat,andamesentericvesselasthebull'seye(redarrow).Onthesecondaxialimage,
aroundedsofttissuemassisseeninthemoredistaldescendingcolon(yellowarrow),whichis
actingasaleadpoint,pullingwithitasegmentofmoreproximaldescendingcolonandits
mesentery.
Intussusceptionoccurswhenaproximalsegmentofboweltelescopesintoamoredistalsegment,
oftenpullingthemesenterywithit(whitearrow).Complicationsofintussusceptioninclude
obstructionandcompromiseofthevascularsupplytothebowelthatresultsinbowelischemia,
perforation,andsepsis.Longsegmentintussusceptionisrelativelyrareinadultsandisalmost
alwaysduetoaleadpointmassorMeckeldiverticulum.Intussusceptionismorecommoninchildren
thanadults,andismostcommonlyileocolicandidiopathic.Insuchcases,theinitialpreferred
treatmentispneumaticreductionwithanairenema.
A.Thesalientabnormalityiswithintheleftlowerquadrant.
C.Thereisnoevidenceofasmallbowelobstruction,withthevisualizedsmallbowelloopsnormalin
caliber,withoutwallthickening,andcontainingintraluminalcontrast.
D.Inadults,longsegment,nontransientintussusception,particularlywithaleadpointidentified,
usuallyrequiressurgery,especiallyifthereisevidenceofvascularcompromise.Themassinthiscase
wasfoundatsurgerytobeacolonpolyp.Shortsegmentintussusception,especiallyinthesmall
bowel,tendstobeatransientphenomenon,notrequiringlongtermfollowuporintervention.
E.Thereisnogenitourinarysurgicalissuenecessitatingaurologyconsult.
Notes

28.A58yearoldconstructionworkerundergoingroutinephysicalexaminationforlife
insuranceevaluationisfoundtohavedecreasedbreathsoundsattheleftlungbase,andachest
xrayisorderedforfurtherevaluation.Basedonthexrayinterpretation,whichofthefollowing
diagnosesmustbeconsidered?
A.Metastaticlungcancer
B.TB
C.Mesothelioma
D.Alloftheabove

Mesothelioma

28.TheanswerisD:Alloftheabove.Thechestxrayshowsasmallleftpleuraleffusion
(redarrows)withnovisiblelungabnormality,andallthreediagnoses,metastaticlungcancer,
TB,andmesothelioma,mustbeconsideredaspossiblecauses.
Thispatienthadaprimarymesothelioma,andafollowupchestxray6monthslatershoweda
significantlylargerlefteffusion.PETCT(below)doneforfurtherevaluationatthattimeshowedthe
largeeffusion(e)andametabolicallyactive,irregularsofttissuemassalongthediaphragmatic
surface(whitearrow),consistentwithpleuralmalignancy.

Mesotheliomaisaprimarypleuralmalignancywithpoorprognosis.Pericardialandperitoneal
mesotheliomaalsooccurbutwithlessfrequency.Occupationalexposuretoasbestosisanalmost
uniformunderlyingriskfactor,althoughpatientsseldomhavetrueasbestosis,whichisanasbestos
relatedinterstitiallungdisease.
Theonsetofmalignancyistypicallydecadesfollowinginitialexposuretocrocidolitetypeasbestos
fibers.Clinicalsymptomsofmesotheliomaincludedyspnea,cough,chestdiscomfort,weightloss,
andanemia.Characteristicchestxrayfindingsincludeeffusion,pleuralthickening,andpleural
basedmassesornodularity.Althoughsomepatientsundergothoracotomyforsurgicalexcision,
prognosisremainspoor,andpalliativetreatmentisappropriate.
Thedifferentialdiagnosisofunilaterallefteffusionislengthyandincludesbothbenignand
malignant,aswellasacuteandchronic,etiologies.Malignanteffusionsmaybecausedbyprimary
mesotheliomaorbypleuralmetastasisfrommanypossibleprimarycancers,includinglung,breast,
GI,ovarian,lymphoma,andmanyothers.InfectiouscausesofunilateraleffusionincludeprimaryTB
andbacterialempyema,whichtypicallyproducesaloculatedeffusion.Certainautoimmunediseases
suchasrheumatoidarthritisandlupusmayproducesmalllefteffusion.Dresslersyndromeisaself
limited,autoimmuneprocessthatoccursaftermyocardialinfarctionoraftercardiacsurgeryand
presentsasasmalllefteffusionassociatedwithleftchestpain.Inpatientswithhistoryoftraumaor
surgery,aunilateralhemorrhagicorchylouseffusionmayoccur.Abdominaldiseaseprocessessuch
aspancreatitis,subdiaphragmaticabscess,orascitesmayalsogiverisetounilateraleffusion.
Cirrhosistypicallyproducesalarger,rightsidedeffusion.Othercommoncausesofeffusion,suchas
congestiveheartfailure,aremostoftenbilateralandusuallyshowunderlyingpulmonaryedema.
Notes

29.AnHIVpositive,54yearoldmanpresentstotheemergencydepartmentcomplainingof
coughandmalaise.Onfurtherquestioning,headmitstoIVdrugabuse,andhehasexperienced
intermittentsweatsandweightlossoverthelast4months.Achestxray(top)andchestCT
(bottom)wereobtained.Whatisthemostlikelydiagnosis?
A.Septicemboli
B.Pneumococcalpneumonia
C.PCP
D.Metastasis
E.TB

TBinHIV

29.TheanswerisE:TB.Thepatienthascharacteristicsymptomsandxrayfindingsof
activeTB,andpromptrespiratoryprecautionsareindicated.ClinicalsymptomsofactiveTB
canbemyriadandincludecough,malaise,nightsweatswithintermittentfevers,andweight
loss.Asinthispatient,xraystypicallyshowpatchy,heterogeneousconsolidations,
predominantlyinthebetteraeratedupperlobezones.Theremaybecavitation(arrows),
sometimeswithairfluidlevels.Otherpossiblefindingsincludepleuraleffusion,pericardial
effusion,andhilarormediastinaladenopathy,especiallyinchildren.ChestCTcommonly
demonstratesevidenceofendobronchialspreadofinfectionwithinfectiousmaterial
distributedinthedistalairways.
ThecausativebacteriumisMycoplasmatuberculum,whichisendemicinunderdevelopedcountries
andoftenseenamongrecentimmigrantsaswellasimpoverishedindividuals.TBhasbeen
consideredanAIDSdefiningillnessinHIVpositivepatients,whoareparticularlysusceptible.The
infectionisspreadthroughinhaledmicrodropletsproducedduringcoughingaswellasduring
talking.Therefore,thepatient'smouthandnoseshouldbecovereduntilantibiotictreatmentrenders
sputumsamplesfreeofacidfastbacilli,andallclosecontactsofthepatientshouldbescreenedfor
TB.
A.SepticembolimaybesuspectedinpatientswithfeverandhistoryofIVdrugabuse.Theyappear
asfaint,roundednodulesonchestxray,sometimeswithevidenceofcentralnecrosis.ChestCT
showsmultiplecavitaryandsolidnodulesshoweredthroughouttheperipherallungs,withnoupper
lobepredominance.
B.PneumococcalpneumoniaoccurswithgreaterfrequencyinHIVpositivepatientsthaninthe
generalpopulation.However,thetypicalchestxrayandCTfindingswouldshowdenselobar
consolidationwithairbronchograms,whichrepresentlucentairwaysbranchingthroughthezoneof
infection.

C.PCPisanopportunisticinfectioncommonlyseenamongHIVpositivepatients,particularlywhen
theCD4countfallsbelow200.ThetypicalchestxrayandCTfindingsareofavague,groundglass,
infiltrativepatternwithscatteredairbronchograms.Thedistributiontendstobediffusebut
heterogeneous.
D.Pulmonarymetastasesmayhaveavarietyofpatterns,includinglungnodules,cavitarylesions,
andinfiltrativeinterstitialpatterns.However,theytendtobewidelydistributedwithoutupperlobe
predominance.
Notes

30.A53yearoldmanpresentstotheemergencydepartmentwithchestpainandshortnessof
breath.TheECGshowselevatedSTsegments.Achestxrayisobtainedwhichshowsdiffuse
coarseningoftheinterstitialmarkings,withKerleyAlines(straightyellowarrow),KerleyB
lines(redarrows),andsmallbilateralpleuraleffusions(curvedyellowarrows).Whatisthe
significanceoftheKerleyAandBlinesonthechestxray?
A.Thepatienthaspulmonaryinterstitialedema.
B.Thepatientisdevelopingacuterespiratorydistresssyndrome(ARDS).
C.Thepatienthasmetastaticlungcancer.
D.Thepatienthasasbestosis.
E.ThepatienthasMycoplasmapneumonia.

AcuteMyocardialInfarctionwithKerleyLines

30.TheanswerisA:Thepatienthaspulmonaryinterstitialedema.Dr.PeterKerley,
aradiologistfromIreland,describedA,B,andClinesonradiographsasrepresentingdistended
pulmonarylymphaticsinpatientswithfluidoverloadorcongestiveheartfailure.KerleyAlines
(yellowarrow)arelonglinesradiatingfromthehilatotheperiphery,andtheyrepresent
lymphaticdistentionintheintralobularseptae.KerleyBlines(redarrows)areshort,peripheral
lineardensitiesthatrepresentdistendedlymphaticsintheinterlobularseptae.KerleyClines
appearonthechestxrayasafinenetworkofcrisscrossinglinesandrepresentsuperimposedA
andBlines(notapparentonimageshown).
Thechestxrayshowsacentralandbibasilargroundglasspatternassociatedwithsmalleffusions
andKerleyAandBlines.Intheclinicalsettingofchestpain,dyspnea,andECGchanges,thechestx
rayfindingsarediagnosticofpulmonaryedema.AchestCT(shownabove),obtainedforother
reasons,confirmsthefindings.
B.InpatientswithARDS,thepulmonarycapillariesliningthealveoliaredamaged,andfluidseeping
intothealveolarairspacescreatesadiffusegroundglasspattern.However,sincethereisnofluid
overload,andnodistentionofthelymphatics,noKerleylinesandnopleuraleffusionsshouldbe
present.
C.Lungcancermetastasisinthechestmaymanifestasmultiplepulmonarynodulesaspleural
metastasiswithmalignanteffusionashilarandmediastinaladenopathy,whichmayobstructcentral
vesselsandaslymphangiticspreadoftumor.Whenthereiscentralvenousobstructionfrom
adenopathyorwhenthelymphaticchannelsareinvaded,therewillbelymphaticobstructionand
distention,withappearanceofunilateralorbilateralKerleylines.Althoughlymphangitic
carcinomatosisisinthedifferentialdiagnosiswhenKerleylinesareobserved,thispatient'sclinical
scenarioishighlycharacteristicofanacutemyocardialinfarctionwithpulmonaryedema.
D.Asbestosisisapneumoconiosiscausedbyinhalationofasbestosdust,whichmayincite
pulmonaryinterstitialfibrosis.However,theincreasedinterstitiallungmarkingsinpatientswith
asbestosistendtobecoarserintexturewithmoredistortionofthelungarchitecture.Calcified
pleuralplaquesoftenaccompanytheinterstitialabnormalitiesinpatientswithasbestosis.
E.Mycoplasmapneumoniaisacommoncommunityacquiredpneumoniathattypically
demonstrateslunginfiltratesandinterstitialprominence.PatientswithMycoplasmahaveclinical
evidenceofupperrespiratorytractinfection,butnoclinicalorchestxrayevidenceoffluidoverload
orvascularcongestion.
Notes

31.An82yearoldmanpresentstotheemergencydepartmentwithacomplaintofpersistent
headacheforthepastweekfollowingafallfromstanding.Cranialnerveandmentalstatus
examsarenormal.TheCTscanisshownabove.Thefindingabovemostlikelyrepresents:
A.Subarachnoidhemorrhage
B.Subduralhemorrhage
C.Epiduralhemorrhage
D.Epiduralabscess

SubduralHemorrhage

31.TheanswerisB:Subduralhemorrhage.Intheelderlypatient,becauseofthecerebral
volumelossandstretchingofthecorticalveins,minortraumasuchasafallfromstandingmay
causetheseveinstotear,withbloodaccumulatinginthesubduralspace.Theappearanceofa
hyperdensecrescenticcollection(straightarrow)crossingsuturelines(curvedarrow)isclassic
foranacutesubduralhematoma.
A.Subarachnoidhemorrhagemayoccurasaresultoftrauma.Itsdistributionistypicallywithinthe
sulci,anditdoesnotdisplacebrainparenchyma.
C.Epiduralhemorrhagemaydisplacebrainparenchymabuttypicallyisassociatedwithsevere
traumaresultinginfractureofthecalvarawithresultanttearingofanepiduralartery.Itistypicallya
lenticularshapedcollectionthatdoesnotcrosssuturelines.Duetoitsarterialsupply,itcanrapidly
expand,resultinginherniationanddeath.Itisasurgicalemergency.

D.Epiduralabscessistypicallyahypodensecollection.Thepatientusuallypresentswithother
symptomssupportinginfectionandalteredmentalstatus.
Notes

32.A38yearold,asymptomaticwomanpresentsforapreoperativechestxray(top)priorto
bunionectomy.ThechestxrayfindingspromptfurtherexaminationwithchestCT(bottom).
Whatisthemostlikelydiagnosis?
A.Pericardialcyst
B.Lymphoma
C.Thymoma
D.Teratoma
E.Diaphragmatichernia

Thymoma

32.ThecorrectanswerisC:Thymoma.Thereisalargesofttissuemass(T)intheright
anteriormediastinum,withasolid,heterogeneousappearancemostsuggestiveofthymoma.
Incidentallyfoundthymomasarenotuncommon,althoughsymptomssuchaschestdiscomfort
maybepresent.Ofpatientswiththymoma,30%50%haveassociatedsymptomsofmyasthenia
gravis,butonly15%20%ofpatientswithmyastheniahaveanunderlyingthymictumor.The
generaldifferentialdiagnosisofanteriormediastinalmassesincludeslymphoma,othercauses
ofadenopathy,thymiclesions,germcelltumorssuchasteratoma,andthyroidmassessuchas
substernalgoiter.
A.Pericardialcystsarefluidfilledlesionswithathinmesotheliallining,usuallyalongtherightheart
border.Thislesionisclearlysolidwithnofluidcomponent.
B.Lymphomamaypresentasanasymmetricmediastinalmassandisstatisticallymorecommon
thanthymoma.However,amoretypicalappearancewouldbeahomogeneous,solidmatof
adenopathyintheprevascularspaceofthesuperiormediastinum.Manyothercausesofbulky
adenopathy,suchassarcoidormetastaticdisease,canalsooccurintheanteriormediastinumbutare
lesslikelyinanasymptomaticpatient.
D.Teratomasandothergermcelltumorsincludeallthreegermcelllayersandtypicallyfeaturefatty

componentsthatareobviouslyradiolucentonCTstudies.Teratomasoftenshowcalcificationsand
sometimespartiallyformedteethorbones.
E.Thispatient'sdiaphragmisintactwithnoevidenceofruptureorherniation.Herniationisusually
leftsidedbutmayoccurontherightandmaycontainliver,colon,and/oromentum.
Notes

33.A41yearoldwomanintheintensivecareunitwithcongestiveheartfailureshows
persistentevidenceofoxygendesaturationdespiteintubationforrespiratorydistress.After
interpretingherportablechestxray,whatisthenextappropriateaction?
A.NGtubeplacement
B.Chesttubeplacement
C.AdministrationofIVfurosemide
D.Endotrachealtube(ET)tubeadjustment
E.Aleftlateraldecubituschestxray

LungCollapse

33.TheanswerisD:Endotracheal(ET)tubeadjustment.Thechestxrayshows
pulmonaryedemaintherightlungandcompleteopacification,orwhiteout,oftheleft
hemithorax.Thecausesofanopacifiedhemithoraxincludecompletelungcollapse,large
pleuraleffusion,denselobarpneumonia,andpriorpneumonectomy.Acluetothepresenceof
lobarcollapsemaybenoticeableshiftofthemediastinumtowardtheaffectedside,causedby
volumelossintheairlesslung.Inlargeeffusions,themediastinumusuallyremainsmidline.
Lungconsolidationfromlobarpneumoniaisseldomthisdenselyopaque,andlucentbranching
airways,knownasairbronchograms,aretypicallyvisibleinzonesofdenselungconsolidation.
Patientswithpriorpneumonectomyshowtypicalpostsurgicalchangessuchasmetallicclips
andsometimessurgicalribdefects.
Inthiscase,theETtubeisbelowthecarinaintherightmainstembronchus,andonlytherightlung
isbeingventilated.Theleftlunghasabsorbedallavailableoxygenfromtheairwaysandhas
collapsed,producingcompleteopacificationofthelefthemithorax.IftheETtubeiswithdrawn5cm,
itwillbeabovethecarina(arrow),andtheleftlungwillreexpand.
A.Thereisevidenceofmoderategastricdistentionintheleftupperabdomen,likelyfromair
swallowingduringintubation.Thisplacesthepatientatsomeriskofaspiration,butplacementofan
NGtubeissecondary,relativetotheimperativeofETtubeadjustment.
B.Achesttubewouldbeindicatedtodrainalargepleuraleffusion,butthispatienthasacollapsed
lungandnoeffusion.
C.Therightlungshowsevidenceofpulmonaryedema,andthepatientmaybenefitfromIV
furosemide.However,ETadjustmentisthefirstpriority.
E.Aleftlateraldecubitusxrayishelpfulindeterminingthepresenceoffreeflowingleftpleural
effusion.However,adecubitusviewoffersnoclarificationwheneffusionsaresolargeastoproduce
completeopacificationofthehemithorax.Furthermore,thecauseofthispatient'swhiteoutislung
collapse,noteffusion.
Notes

34.A38yearoldwomanpresentstoherphysician'sofficewithacomplaintofacuteonsetof
blurryvisionandpaininherlefteye.Hermedicalhistoryisotherwiseunremarkable.TheMRI
demonstratedenhancementoftheleftintraorbitalsegmentoftheopticnerveoncoronal
postcontrastT1imagingcorrespondingtoopticnerveedemademonstratedonthecoronalT2
imaging.Axialfluidattenuatedinversionrecovery(FLAIR)imagingofthebraindemonstrated

multiplehyperintenselesionspredominantlyperiventricularinlocation.Herblurryvisionis
mostlikelyrelatedto:
A.Ischemicopticneuropathy
B.Orbitalpseudotumor
C.Opticneuritis
D.Orbitalcellulitis

OpticNeuritisMultipleSclerosis

34.TheanswerisC:Opticneuritis.Theclinicalsymptomsdescribedareatypical
presentationofthedisease.Opticneuritisisaninflammatorydemyelinizationthatisassociated
withanincreasedriskofdevelopingmultiplesclerosis.Thereisenhancementoftheopticnerve
(straightarrow),withopticnerveedema(curvedarrow).Themostsignificantpredictorforthe
subsequentdevelopmentofmultiplesclerosisisthepresenceofwhitematterlesionsonFLAIR
imagingofthebrain(arrowheads).Theirperpendicularorientationtothelateralventriclesis
typicalformultiplesclerosis.
A.Ischemicopticneuropathysecondarytoatheroscleroticdiseasetypicallypresentswithpainless
visionlossandistypicallyinanolderagegroup.Theimagingcharacteristicsarerelatively
nonspecific.
B.Orbitalpseudotumortypicallypresentswiththeacuteonsetofpain,swelling,proptosis,and
decreasedvision.Intheabsenceofothersignssuggestinginfection,thediagnosisisfrequentlymade
clinically,andthepatientisputonsteroids.
D.Orbitalcellulitispresentswithproptosisandophthalmoplegia.Itisfrequentlyaccompaniedby
othersignsandsymptomsofinfectionincludingfever,pain,decreasedvision,andchemosis.Itis
usuallyapparentclinically.
Notes

35.A63yearoldhealthymanwithnosmokinghistoryhasalungnoduleincidentally
discoveredonapreoperativechestxrayforkneearthroscopy.ChestCTconfirmsa9mm
solitarypulmonarynodule(arrow)intherightmiddlelobewithnootherabnormalities.What
isthenextappropriatestep?
A.Purifiedproteinderivative(PPD)placement
B.PETscanning
C.CTguidedlungbiopsy
D.FollowupCTscanin1year
E.Nofurtherevaluationisneeded

SolitaryLungNodule
35.TheanswerisE:Nofurtherevaluationisneeded.TheCTscanshowsthatthe
solitarylungnodulecontainslucentfatdepositscharacteristicofabenignhamartoma.
Therefore,notreatmentorfollowupimagingisneeded.
Hamartomasarethemostcommonbenignneoplasmsofthelungandarethethirdmostcommon
causeofasolitarylungnodule.Theyaremorecommoninmenthaninwomenandtypicallypresent
asanincidentalchestxrayfindinginmiddleagedpatients.Pathologicallytheyrepresentbenign
neoplasmsthatcontainavarietyoflungtissuesincludingfat,benignepithelialcells,andoften
cartilage,whichmayexhibitcalcification.Therefore,theCTappearanceisoftendiagnostic,especially
ifthelesioncontainsfatorpopcorntypecalcification.
Thedifferentialdiagnosisofanonspecificappearing,solitarypulmonarynoduleisextensive,
includingbothbenignandmalignantetiologies,although,fortunately,mostarebenign.Themost
commonetiologiesareactiveordormantinfectionandprimaryormetastaticmalignancies.
Indicationsforfurtherevaluationofanonspecificsolitarynoduledependonthepatient'sriskfactors
formalignancyandonthesizeandappearanceofthenodule.Factorswhichincreasethepatient's
riskofmalignancyincludesmokinghistory,exposuretoasbestosorradon,pasthistoryof
malignancy,advancedage,andalargersizedorspiculatednodule.Factorswhichincreasethe
likelihoodofbenignetiologyincludelackofsmokinghistory,positivePPD,youngage,smallnodule,
andthepresenceoffatorcalcificationinthenodule.Thepresenceofcalcificationisreadily
detectablewithCTandvastlyimprovesthelikelihoodofbenignetiologysuchasagranulomafrom
TBorhistoplasmosis.

Table31(t0010)summarizesgeneralrecommendationsandshowsthatonemayconclude,for

example,thata25yearoldnonsmokerwitha3mmnoduleneedsnofurtherevaluationhowever,a
65yearoldheavysmokerwitha9mmnodulemaywanttoconsiderbiopsy,orcertainlyfurther
evaluationwithPETCT.
Table31

Nodule

LowRiskPatient

HighRiskPatient

Size
Nodule Nofollowuprequired

FollowupCT@12months

4mm

ifstable,stop

Nodule= FollowupCT@12months

FollowupCT@612months

46mm

ifstable,thenCT@1824months

ifstable,stop

Nodule> FollowupCT@612months

FollowupCT@36months

68mm

ifstable,then912months

ifstable,thenCT@1824months

ifstable,then24months
Nodule> FollowupCT@3,9,24months

FollowupCT@3,9,24monthsdynamic

8mm

CTw/contrast,PET,and/orbiopsy

dynamicCTw/contrast,PET,and/or
biopsy

A.PPDplacementisnotnecessaryinthiscase,becausethelungnoduleisobviouslyabenign
hamartoma.However,PPDstatuswouldbehelpfulinweighingriskfactorsforanonspecific,
noncalcifiednodule.
B.PETCTisnotneededforevaluationofthisbenignlesion.However,PETCTmaybehelpfulin
determiningthelikelihoodofmalignancyofanambiguousnodulethatmeasuresatleast1cm.
C.Biopsyisnotindicatedforthisbenignlesion.However,biopsyofa9mm,nonspecificnodulemay
beconsidered,especiallyinahighriskpatient.
D.Nofollowupisindicatedinthiscase.However,a9mm,nonspecificnodulewithoutcalcification
orfatwouldrequireclosefollowupifbiopsywerenotperformed,eveninalowriskpatient.Multiple
followupscanswouldbeneededtodocumentshorttermstability,beginningat3months,andlong
termstability,endingat2years.Aonethirdincreaseindiameter,from9mmto12mm,would
representavolumedouble,andifthisoccurredovera6monthperiod,itwouldrepresenta
suspiciouspatternofgrowththatwouldpromptbiopsy.
Notes

36.A20yearoldwomanpresentstotheemergencydepartmentwithcrampylowerabdominal
painandsomevaginalbleeding.Herlastmenstrualperiodwasapproximately2monthsago
however,hercyclesareirregular.AserumbetaHCGlevelis50,000mIU/mL.Whichofthe
followingstatementsistrueregardingtheimagesoftherightadnexaaboveobtainedfroma
transvaginalultrasoundwithcolorDoppleranalysis?
A.Agestationalsacispresent,anditiswithintheuterus.
B.Whileagestationalsacispresent,thereisnoembryooryolksacidentified.
C.ColorDoppleranalysisdemonstratesnosignificantabnormalityintherightadnexa.
D.Arightovaryappearsseparatefromthegestationalsac.
E.Theapproximateageofthisfetusis18weeks.

EctopicPregnancy

36.TheanswerisD:Arightovaryappearsseparatefromthegestationalsac.The
imagesoftherightadnexaobtainedfromatransvaginalultrasoundwithcolorDoppleranalysis
demonstrateanectopicpregnancyinthefimbrial/infundibularregionoftherightfallopian
tube.Onthefirstprovidedimage(aboveleft),therightovary,containingmultiplehypoechoic
subcentimeterfollicles,isseentotherightandslightlyanteriortotherightovary(white
arrow),andabutsthegestationalsac(redarrow)atanacuteanglewithathinamountof
echogenicadnexaltissueintervening.
Anectopicpregnancyisdefinedasanygestationoccurringoutsideoftheuterusorwithinthecornua
oftheuterus,andcanleadtohighmorbidityandmortalityifdiagnosisisdelayed.Ninetyfive
percentofallectopicpregnanciesaretubal.Essentialsonographicfindingsfordiagnosinganectopic
pregnancyinwomenwithapositivebetaHCGincludenointrauterinepregnancyandan
adnexal/tubalmass.Ifthereisnointrauterinepregnancyandanadnexalmassisnotidentified,
ectopicpregnancycannotbeexcluded,andthepatientrequiresclosefollowupwithserialbetaHCG
levelsand,ifneeded,repeatultrasounds.Treatmentoptionsforectopicpregnancyincludemedical
management(e.g.,withoralmethotrexate),surgicalmanagementwithsalpingectomy,orultrasound
guidedpercutaneousinjectionofthegestationalsacwithpotassiumchloride.
A.Whileimagesoftheuterusarenotprovidedtoexcludeaconcurrentintrauterinepregnancy(so
calledheterotopicpregnancy),thisgestationalsaciscertainlynotwithintheuterus.
B,E.Nomeasurementsareprovidedtoaccuratelydatetheembryohowever,thecrownrump
lengthoftheembryoisslightlymorethan10mmbasedonthescaleprovidedonthesideofthe
image(eachdotrepresentingasonometer),andthereisstillayolksac,makingthispregnancy
clearlywithinthefirsttrimester.
C.OncolorDoppler(aboveright),thereisasemicircleofincreasedbloodflowsurroundingthe
gestationalsac,whichiscompatiblewiththesocalledringoffiresignattributedtoanectopic
pregnancy(yellowarrow).Caremustbetakentoprovethattheectopicpregnancyisseparatefrom
theovary.Anovarianectopicpregnancyisextremelyrarehowever,acorpusluteumwithintheovary
canalsohavearingoffireappearance,leadingtofalsepositives.Iftherewerefurtherquestionasto
whetherthemass/gestationalsacwasovarianinorigin,theultrasoundprobecouldbeusedtoapply
pressuretotheovary.Iftheovarymovesindependentlyfromthegestationalsac,thentheyaretruly
separatestructures.
Notes

37.A60yearoldmanwithhypertensionpresentstotheemergencydepartmentwithtearing
backpain.ACTangiography(CTA)ofthethoraxisobtained.Classically,howisthisdisease
processmanaged?
A.Watchfulwaiting

B.Endovasculargraftplacement
C.Medicalmanagement(i.e.,lowerbloodpressure)
D.Aorticbypass
E.Aorticvalvereplacement

AorticDissectionTypeB

37.TheanswerisC:Medicalmanagement(i.e.,lowerbloodpressure).Thispatient
ispresentingwithanacutethoracicaorticdissection,typeB.Aorticdissectionoccurswhenan
intimaldisruptionresultsinasubintimalhematomathatcanexpandtocompressthetrue
aorticlumen(redarrows).Thefalselumencanpropagateineitheraproximalordistal
directionwithpotentialtoobstructaorticbranchvessels.
Underthepreferredclassificationsystem,theStanfordsystem,therearetwotypesofdissections:
typeAoriginatesintheascendingthoracicaorta,andtypeBoriginatesdistaltotheleftsubclavian
artery.TypeBdissectionsareclassicallytreatedwithmedicalmanagementifmajorbranchvessels,
suchasthesuperiormesentericartery,communicatewiththetrueaorticlumen,asinthiscase.The
mortalityrateofthosewithtypeBdissectionstreatedwithmedicalmanagementis10%.
Aorticdissectionisalifethreateningemergencywithaveryhighmortalityrate,necessitatingrapid
diagnosiswithCTAinsuspectedpatients.OnCTA,anintimalflapwithacontrastenhancing
intramuralhematoma(i.e.,falselumen)isdiagnostic.InpatientswhohavecontraindicationstoIV
contrast,MRangiographyandtransesophagealechocardiographyarealternativeimagingmodalities.
Inpatientswithchestpain,ischemicheartdiseaseisatleast1000timesmorecommonthanaortic
dissection.
A.Untreateddissectionshaveamortalityrateofapproximately1%2%perhourforthefirst48
hours,makingwatchfulwaitinganinappropriateresponse.
B,D.TypeAdissectionsaretypicallytreatedwithendovasculargraftplacement,withamortality
rateof30%.
E.TypeAdissectionscancompromisetheoriginsofthecoronaryarteriesandtheaorticvalve,which
maynecessitatevalvereplacement.
Notes

38.A50yearoldmanwithnopastmedicalhistorypresentstotheemergencydepartmentwith
newyellowingofhisskinandeyes.Hedeniesanyabdominalpain.Asidefromelevated
bilirubin,hisserumchemistriesarenormal.Basedontheimagesfromthecontrastenhanced
CToftheabdomenabove,whatisthediagnosis?
A.Sclerosingcholangitis
B.Cholangiocarcinoma
C.Adenocarcinomaofthepancreatichead
D.Obstructivecholedocholithiasis
E.Portalveinthrombosis

ObstructiveJaundiceCholedocholithiasis

38.TheanswerisD:Obstructivecholedocholithiasis.Thispatientispresentingwith
painlessjaundice.Theclassicetiologyofpainlessjaundiceinamiddleagedindividualis
pancreaticcarcinoma,andthisCToftheabdomenwasobtainedtoevaluateforsuch.However,
itwasfortunateforthispatientthattheCTclearlydemonstratedanobstructingcalculusinthe
distalcommonbileduct(redarrow),withinthepancreatichead,withresultingmarked
extrahepaticandintrahepaticbiliaryductaldilatation.Thispresentationissomewhatatypical
forobstructivecholedocholithiasis,becausepatientsoftenexperiencepain.Thedilatationof
thecommonbileductleadingtothecommonhepaticductandlefthepaticductisseenwellon
thecoronalimage.Theaxialimagedemonstratesthemarkeddilatationofcentralintrahepatic
biliaryradicals(ayellowarrowidentifiesonesuchradicalintherighthepaticlobe).
Causesofcommonbileductobstructionincludecholedocholithiasis,pancreaticorampullary
neoplasms,benignstricture,andparasiteinfection.ThedensityofgallstonesisvariableonCT,with
calculirangingfromlessdensethanwater,isodensetowater(thereforeinvisible),orcalcifiedasin
thiscase.MRcholangiopancreatography(MRCP),ultrasound,andhepatobiliaryscintigraphy,often
provemoresensitiveimagingmodalitiesinsuspectedcholedocholithiasis.Inanacutepresentation
ofobstructivejaundice,CToftheabdomenwithcontrastisanappropriatefirstlineimaging
modalitytoevaluateforneoplasticandpotentiallyinfectiousorautoimmunecauses.Endoscopic
retrogradecholangiopancreatography(ERCP)withsphincterotomyandcalculusretrievalisa
preferredinitialtherapyinobstructivecholedocholithiasis.

A.Whilesclerosingcholangitisisacauseofobstructivebiliarydilatation,themostcommonfinding
onCTisscattereddilatedintrahepaticbileductsthatdonotclearlycommunicatewiththebiliary
system,and,again,thereisamoreclearlyidentifiableetiologyontheprovidedimages.
B.Likewise,thereisnointraluminalorintrahepaticmassontheseimagestosupportadiagnosisof
cholangiocarcinoma.
C.Ontheprovidedcoronalimage,nopancreaticheadmassisseen,andfurthermore,aclearcauseof
thebiliarydilatationisidentified.
E.Thevisualizedportionsoftheportalveinsystem,seenadjacenttothedilatedintrahepaticbiliary
ductsandcommonbileductonthecoronalimage,areenhancingnormally.

ImagesforQuestion39onpage254
Notes

39.FollowingahighspeedMVA,anunrestrained47yearoldwomanpassengeristransported
byairtothenearesttraumacenter.Thepatientrequiredintubationattheaccidentscene,but

respondstocommandsandisfoundtobeneurologicallyintactonexamination.ACTofthe
cervicalspineisperformed.Basedonthefindings,thepatientistakenforastatMRIandthen
takenforsurgery.Anintraoperativelateralradiographofthecervicalspineisobtained(shown
above).Basedontheimagesprovided,whichofthefollowingstatementsistrue?
A.Thistypeofcervicalspinefractureisconsideredstable.
B.Spinalcordedemaand/orhemorrhagewilldefinitelybeseenontheMRI.
C.CervicalMRangiographyshouldbeperformedatthesametimeastheMRI.
D.ThistypeofinjuryhasbeennamedaJeffersonfracture.
E.Themechanismofinjuryproducingthisfractureclassicallyishyperflexion.

Hangman'sFracture(explanationonp.256)

39.TheanswerisC:CervicalMRangiographyshouldbeperformedatthesame
timeastheMRI.Thepatienthassufferedtraumaticspondylolisthesisoftheaxis(TSA),

commonlyknownasthehangman'sfracture.ThesagittalCTimagesaboveshowbilateral
traumaticspondylolysisatC2,orfracturesthroughtheparsinterarticularisonbothsidesofthe
posteriorneuralarchofC2(whitearrows).Onthelateralradiograph,althoughsubtle,thereis
traceanterolisthesis,oranteriortranslationofC2onC3(yellowarrow).Thissubluxationwould
likelybeexaggeratedifneckflexionwereallowed.TheaxialCTimageshowsthatthefracture
ontheleftextendsintotheforamentransversarium(redarrow),throughwhichtheleft
vertebralarterytravels.Vertebralarteryinjuryisarelativelycommonpotentialcomplication,
especiallywhenthefractureextendsintotheforamentransversarium,asseenwiththiscase.
CervicalMRangiographyisausefuladditionalsequencetobeperformedwiththeexpected
conventionalMRIofthecervicalspinetoevaluateforvertebralarteryinjury.
Thetermhangman'sfractureiscommonlyused,thoughanachronistic,todescribeTSA.Thisinjury
wasoriginallynamedbasedonhistoricalaccountsofsuchfracturesoccurringduringhangings,when
theknotofthenoosewasplacedunderthechinandviolentlyforcedupwardsundertheweightofthe
victim.Today,ofthosestillhangedforcapitalpunishment,onlyaminorityisfoundtohaveC2arch
fractures,andMVAsandfallsaccountforamajorityofsuchinjurypatterns.Inhighvelocitytrauma
victims,whencervicalspineinjuryissuspected,CTisamoreappropriatefirstlineimagingstudy
thanradiographs.MRIisextremelyusefulinevaluatingfornonosseousinjury,suchastraumatic
discherniation,epiduralhematoma,andligamentousdamage.
A.Whilethefractureisconsideredunstableandrequiressurgicalfixation,neurologicdeficitsoccur
inonlyabout25%ofcases,likelyduetotherelativecapaciousnessofthespinalcanalatC2andthe
potentialforautodecompressionfromthefracture.
B.Cordedemaand/orhemorrhagewouldbealesslikelyfindingontheMRIofthecervicalspineof
thisneurologicallyintactpatientforthereasonscitedinA.
D.Thishasbeentermedthehangman'sfracture.AJeffersonfractureclassicallydenotesafourpart
breakthroughtheanteriorandposteriorarchesoftheC1ring.
E.Themechanismofthisfractureisusuallyforcedhyperextensionwithaxialloading,andisoften
theresultofthechinstrikingthedashboardinamotorvehiclecrash.
Notes

40.A50yearoldwomanisbroughttotheemergencydepartmentwithsevereabdominalpain
thathasworsenedoverthecourseofaboutaweek.Sheisalsodyspneic.Herpastmedical
historyissignificantforinjuriessustainedinanMVA1yearago.Pertinentphysicalexam
findingsincludedecreasedbreathsoundsintherightlungbase,andabdominalguardingand
reboundtenderness.Sheistachycardicwithaborderlinelowbloodpressure.Herserumlactate
iselevated.Whichofthefollowingstatementsistrueregardingthefindingsontheselect
imagesprovidedfromaCToftheabdomen?
A.Findingsaremostconsistentwithcomplicatedpneumoniaintherightlungbase.
B.Thevisualizedbowelisnormal.
C.Thisacuteprocessislikelythesequelaofremotetrauma.
D.Thepatienthasagoodprognosiswithsurgicalmanagement.
E.Therighthemidiaphragmisintact.

DiaphragmaticRupturewithStrangulatedIntrathoracicHerniatedBowel

40.TheanswerisC:Thisacuteprocessislikelythesequelaofremotetrauma.The
CTimagesabovedemonstrateherniationofsmallbowelandmesenteryintothethoraxthrough
atraumaticdiaphragmaticrupture(redarrows).Intheacutesettingoftrauma,diaphragmatic
ruptureisoftendifficulttodiagnoseandmayremainclinicallysilentforalongperiodoftime
untilcomplicationsarise,suchasinthiscase.
Strangulatedherniatedbowelisalifethreateningemergencyrequiringimmediatesurgery.
Diaphragmaticrupturecanresultbothfrombluntabdominaltrauma,duetoincreased
intraabdominalpressure,andfrompenetratingtrauma,fromdirectlacerationfromaprojectileor
stabbingweapon.CTisthebestinitialstudyinapatientwithanacuteabdomen,especiallyif
diaphragmaticruptureiswithinthedifferentialdiagnosis.Nontraumaticdiaphragmatichernias
includeBochdalekhernia(posterolateral),Morgagnihernia(anteromedial),andhiatalhernias.
A.Whilethereislikelycompressiveatelectasisintherightlungbasefromtheherniatedmass,there
isnoconsolidativeprocessinthevisualizedlungs.
B.Theherniatedbowelisdilatedwithfluid,andthebowelwallisthickenedandhypoenhancing,
indicatingobstructionandstrangulationorvascularcompromise.

D.ThemortalityincasesofstrangulatedGIdiaphragmaticherniaisreportedasupto60%.
E.Again,therighthemidiaphragmisobviouslynotintact.

ImagesforQuestion41onpage260
Notes

41.A21yearoldmanisbroughttothenearesttraumacenterafteradivingaccidentfroma
largeheight.ACTofthecervicalspineisperformed(seeimagesonp.259),followed
immediatelybyanMRIofthecervicalspine(shownabove).Basedontheimagesprovided,
whichofthefollowingstatementsistrue?
A.TheaxialCTimageshowsanadditionalosseousabnormality.
B.Thisinjuryisconsideredstable.
C.Thereisnosofttissueinjury.
D.Thereisnodamagetothecervicalspinalcord.
E.Thereishighincidenceoftetraplegiaassociatedwiththeseinjuries.

BilateralInterfacetalDislocationJumpedFacets

41.TheanswerisE:Thereishighincidenceoftetraplegiaassociatedwiththese
injuries.TheCTimagesabovedemonstratebilateralinterfacetaldislocationwithjumped
facets.ThesagittalCTimagesthroughthefacetjointsshowsbothofthesuperiorarticulating
massesdislocatedanteriortotheinferiorarticulatingmasses(redarrows),withlossofthe
normalshinglelikealignment.Additionally,ontheMRimages,aswouldbeexpected,thespinal
canalisseverelynarrowed,withimpingementofthecervicalcordandcordedemaatthelevel
ofdislocation(yellowarrow).Giventheevidenceofcervicalcordinjury,tetraplegiaisquite
likely,asiscommoninsuchinjuries.
Bilateralinterfacetaldislocationisasevereinjuryofthemidtolowercervicalspinecausedby
extremehyperflexion.Inhyperflexioninjuries,facetscanbewidened,perched(inferiortipof
superiorfacetincontactwithsuperiortipofinferiorfacet),orjumped.Otherhyperflexiontype
injuriesincludehyperflexionsprain,simplewedgecompressionfracture,andflexionteardrop
fracture.EvaluationwithbothCTandMRIofthespineisessentialtodeterminetheextentof
osseousandsofttissueinjury,respectively.
A.TheaxialCTimagedemonstratesthesamefinding,andillustratesthesocalledreverse
hamburgersign(whitearrows),wherethefacets,representingthehalvesofahamburgerbun,
appearflippedandupsidedowninrelationtoeachother.
B,C.ThesagittalCTimagethroughthemidportionofthevertebralbodiesshowsgreaterthan50%

subluxationoftheC6vertebralbodyanteriorlyontheC7vertebralbody(greenarrow).Inorderfor
thecervicalvertebratotranslateanteriorly,theremustbedisruptionofnearlyallofthesupporting
ligamentousstructuresofthespineatthatlevel,includingtheanteriorlongitudinalligament,the
intervertebraldisc,theposteriorlongitudinalligament,theligamentumflavum,andtheinterspinous
ligaments,makingitaninherentlyunstableinjury.Edemacanbeseenwithinamajorityofthe
aforementionedligamentousstructuresandintheprevertebralsofttissuesontheMRimages
provided.
D.Thereiscervicalcordinjuryasevidencedbytheimpingementofthecervicalcordandcord
edema.
Notes

42.A63yearoldmanpresentswithahistoryofacuteonsetoflefthemiparesisanddysarthria
12hoursbeforehisarrivaltotheemergencydepartment.ACTscanwasordered.Thebestnext
stepinhismanagementis:
A.Oraladministrationof325mgaspirin
B.Emergentadministrationofintraarterialrecombinanttissueplasminogenactivator
(rtPA)
C.EmergentCTperfusionstudytoassessfortheamountoftissueatrisk

D.Emergentangiographicinterventionwiththrombectomy

MCAInfarct

42.TheanswerisA:Oraladministrationof325mgaspirin.TheCTscanshowsa
completedinfarct(straightarrows)oftheleftMCAterritory.Thethrombusisidentifiedwithin
theleftMCAontheleftimageasthedenseMCAsign(curvedarrow).CurrentAHA/ASA
recommendationspublishedin2007recommendtheoraladministrationofaspirin(initial

doseis325mg)within2448hoursafterstrokeonsetfortreatmentofmostpatients.
B.Selectedpatientswouldhavetopresentwithin6hoursoftheonsetofsymptomsforemergent
administrationofintraarterialrtPAtobeaconsideration.
C.EmergentCTperfusionstudyisindicatedwhenthepatientpresentsinatimeframewhere
neurointerventionalthrombolysisorthrombectomyisaconsideration.Itprovidesthe
interventionalistwithadditionalinformationaboutthesizeofthecoreinfarctandtheamountof
potentiallysalvageabletissuewithintheaffectedvascularterritory.
D.Emergentneuroangiographywiththrombectomymaybeanoptionforselectedpatientswho
presentinlessthan6hoursfromtheonsetofsymptoms.
Notes

43.A50yearoldwomanvisitsherprimarycarephysiciancomplainingofgraduallyincreasing
shortnessofbreath.Shehasahistoryofmildasthmaandhassmokedonepackofcigarettesa
dayfor10years.Onphysicalexamination,shehasfacialswelling,distendedneckveins,and
prominentsuperficialveinsonherchestwall.Achestxrayisobtainedforfurtherevaluation.
Whatisthemostlikelydiagnosis?
A.Asthmaexacerbation
B.Lungcancer
C.Lymphoma
D.TB
E.Pulmonaryembolism

SuperiorVenaCava(SVCSyndrome)

43.TheanswerisB:Lungcancer.Thepatient'ssymptomsofdyspneawithphysicalsigns
offacialedema,distendedneckveins,andprominentcollateralveinsonthechestwallare
characteristicofsuperiorvenacavasyndrome(SVCS),anobstructionoftheSVCmost
commonlycausedbylungcancer.
Thechestxray(top)confirmsalargemediastinalmass,predominantlyrightsided,intheregionof
theSVC(arrows),andhighlysuspiciousformalignancy.AchestCTstudy(bottom)confirmsabulky
mediastinalmass(M)causingnearlycompleteobstructionoftheSVC(V).
SVCSiscausedbymalignantobstructionoftheSVCinabout80%ofcases.Amongmalignantcases,
about85%arecausedbybronchogeniccarcinoma,withabout15%causedbynonHodgkin
lymphoma.PrognosisforSVCSfrombronchogeniccarcinomaispoor,with90%mortalityat30
months,despitetreatmentwithradiationandchemotherapy.Withoutradiation,mostaredead
withinamonth.EndovascularstentplacementtoopentheSVCishelpfulforpalliation.
A.Althoughthepatienthasahistoryofasthma,hersymptomsandchestxrayfindingsaremore
typicalofSVCSfromalargemediastinalmass.Theexpectedchestxrayfindingsinapatientwith
asthmaexacerbationwouldbepulmonaryhyperinflation,sometimeswithpatchyareasofatelectasis
ifthereismucouspluggingfromsecretions.
C.LymphomacancauseobstructionoftheSVCbutlessoftenthanbronchogeniccarcinoma.
D.TBandothergranulomatousdiseases,suchashistoplasmosis,cancausemediastinaladenopathy
andmediastinalfibrosis,whichmayresultinSVCS.TheseetiologiesofSVCSaremorecommonin
developingcountrieswhereTBismoreprevalent.IntheUnitedStates,bronchogeniccarcinomaisa
farmorelikelycause.

E.Pulmonaryembolismmaycausedyspnea,andsomepatientshaverecognizablepulmonaryartery
enlargementonthechestxray.However,thispatienthasabulkymediastinalmassandno
pulmonaryarterialenlargement.
Notes

44.A32yearoldwomanpresentstoherprimarycarephysicianwithpaininherleftelbow
afterfallingwhilerollerblading2daysagoandbracingherfallwithheroutstretchedleftarm.
Onphysicalexam,thereisswellinganddiffusetendernessinvolvingtheleftelbow.
Radiographsareobtained.Whichofthefollowingstatementsistrue?
A.Theanteriorfatpadiselevated.
B.Theposteriorfatpadisnotdisplaced.
C.Thereisnojointeffusion.

D.Thereisnofracture.
E.Asupracondylarfractureishighlylikely.

ElbowJointEffusionandRadialHeadFracture

44.TheanswerisA:Theanteriorfatpadiselevated.Thepatientpresentswitha
FOOSHtypeinjury.Intheadultpopulation,aFOOSHmechanismcanresultininjuryatthe
wrist,elbow,orshoulder.Basedonthelateralradiographshown,thereisdisplacementofboth
theanterior(yellowarrow)andposterior(whitearrow)fatpads,fromtheanteriorand
posteriorolecranonfossae,respectively,indicatingthepresenceofajointeffusion.
Inyoungadults,themostcommonelbowfractureinvolvestheradialhead(50%),isusually
nondisplaced(<2mm),andresultsfromlowenergyaxialloadinginaFOOSHmechanism.Itis
normaltoseetheobliqueanteriorfatpadonalateralradiograph,butwhenelevated,producingthe

socalledsailsign,aneffusionispresent.Itisnevernormaltoseetheposteriorfatpad.A
nondisplacedradialheadfractureisoftenoccult/subtleonradiographsnevertheless,thepresenceof
aneffusionwithoutotherobviousfracture,dislocation,orotherjointderangementhighlysuggests
thepresenceofaradialheadfracture.Treatmentofanondisplacedradialheadfractureisusually
limitedtononsteroidalantiinflammatorydrugsandphysicaltherapy,withearlymobilizationkeytoa
goodprognosis.
B.Thefatpadisdisplacedasnoted.
C.Ajointeffusionispresent.
D.OntheAPradiographshown,thoughsubtle,theverticalcomponentofaradialheadfracture(red
arrow)canbeseeninthemidportionofthearticularsurface.Nevertheless,inthesettingoftrauma,a
jointeffusionindicatesfractureuntilprovenotherwise,andinadultsaradialheadfractureismost
common.
E.Inchildren,themostcommonelbowfractureisasupracondylarfracture.

Copyright2015Elsevier,Inc.Allrightsreserved.

BOOKCHAPTER

AnswerKeys
AmberS.TullyMDandJamesS.StuddifordMD,FACP
USMLEImagesfortheBoards,269272

SectionIBoardImages
Question1:E
Question2:E
Question3:B
Question4:E
Question5:A
Question6:A
Question7:D
Question8:B
Question9:A
Question10:F
Question11:F
Question12:A
Question13:D
Question14:D
Question15:C
Question16:D
Question17:B
Question18:E
Question19:D
Question20:E
Question21:B
Question22:B
Question23:E
Question24:B
Question25:E
Question26:E
Question27:E
Question28:C
Question29:D
Question30:A
Question31:A

Question32:A
Question33:D
Question34:C
Question35:A
Question36:C
Question37:A
Question38:B
Question39:A
Question40:E
Question41:A
Question42:E
Question43:B
Question44:E
Question45:C
Question46:B
Question47:A
Question48:D
Question49:B
Question50:C
Question51:E
Question52:A
Question53:C
Question54:D
Question55:E
Question56:B
Question57:B
Question58:A
Question59:E
Question60:E
Question61:C
Question62:E
Question63:B
Question64:D
Question65:D
Question66:C
Question67:A
Question68:D
Question69:D

Question70:A
Question71:D
Question72:A
Question73:B
Question74:D
Question75:C
Question76:D
Question77:C
Question78:F
Question79:B
Question80:B
Question81:C
Question82:B
Question83:B
Question84:E
Question85:B
Question86:B
Question87:A
Question88:E
Question89:C
Question90:A
Question91:B
Question92:B
Question93:A
Question94:C
Question95:E
Question96:E
Question97:E
Question98:C
Question99:D
Question100:D
Question101:E
Question102:E
Question103:A
Question104:D
Question105:E
Question106:D
Question107:D

Question108:C
Question109:C
Question110:E
Question111:C
Question112:A
Question113:B
Question114:A

SectionIICardiology
Question1:E
Question2:D
Question3:B
Question4:A
Question5:B
Question6:B
Question7:E
Question8:A
Question9:A
Question10:D
Question11:B
Question12:B
Question13:C
Question14:D
Question15:A
Question16:C
Question17:B
Question18:D
Question19:E
Question20:D

SectionIIIRadiology
Question1:C
Question2:A
Question3:D
Question4:B
Question5:E
Question6:A
Question7:C
Question8:C
Question9:A

Question10:B
Question11:E
Question12:C
Question13:B
Question14:A
Question15:D
Question16:B
Question17:C
Question18:C
Question19:D
Question20:C
Question21:C
Question22:D
Question23:B
Question24:E
Question25:E
Question26:D
Question27:B
Question28:D
Question29:E
Question30:A
Question31:B
Question32:C
Question33:D
Question34:C
Question35:E
Question36:D
Question37:C
Question38:D
Question39:C
Question40:C
Question41:E
Question42:A
Question43:B
Question44:A

Copyright2015Elsevier,Inc.Allrightsreserved.

For more high yield stuff for the board exams please check our facebook page;
https://www.facebook.com/USMLEgoodies/
And for watching a wide variety of medical lecture videos please check our youtube channel;
https://www.youtube.com/channel/UCB18YOYrrzR6asXcXNGtKPQ/videos?view_as=public
Thanks

You might also like